Respiratory (UWORLD)

Lakukan tugas rumah & ujian kamu dengan baik sekarang menggunakan Quizwiz!

-Age: 25 years man -Comes to the hospital due to acute-onset shortness of breath. The patient has a history of cystic fibrosis and multiple hospitalizations for recurrent pneumonia. He has a frequent productive cough at baseline. He does not use tobacco -BP (80/50), pulse (110), RR (24). Examination shows mild cyanosis and subcutaneous crepitus. Breath sounds are decreased on the left. Which of the following is most likely responsible for this patient's acute symptoms? a)Alveolar consolidation due to inflammatory exudate b)Bronchial obstruction with alveolar air resorption c)Diffuse constriction of bronchioles d)Increased dead-space ventilation e)Loss of intrapleural negative pressure

Answer: Choice E (Loss of intrapleural negative pressure) -This patient with sudden-onset shortness of breath and a physical examination showing subcutaneous crepitus and unilaterally decreased breath sounds most likely has a pneumothorax. In cystic fibrosis, chronic lung damage, combined with mucus plugging and large alveolar pressure surges (e.g., coughing), predisposes to spontaneous alveolar rupture -Pressure in the lungs is normally equivalent to atmospheric pressure (i.e., 0cm H2O) at end expiration, and the pressure in the intrapleural space is negative (e.g., -5cm H2O) due to the expanding tendency of the chest wall and collapsing tendency of the lungs. Alveolar rupture creates a continuity between the lungs and the pleural space, with pressure equalization and loss of intrapleural negative pressure. Shortness of breath results from inability to expand the ruptured lung, and during rupture, air may be forced into the subcutaneous tissues of the chest wall to cause crepitus -Simple pneumothorax is most common, but tension pneumothorax can develop if the rupture creates a one-way tissue valve that opens during inspiration to allow air into the pleural space and then closes during expiration to trap that air. As a result, intrapleural pressure increases with each breath and leads to contralateral mediastinal shift with vena cava collapse and decreased venous return to the heart. The reduced cardiac output causes hypotension and tachycardia (i.e., obstructive shock) and can rapidly lead to cardiac arrest; therefore, emergent decompression of the pleural space is needed Choice A= Alveolar consolidation with inflammatory exudate occurs with pneumonia, which can cause hypotension (i.e., septic shock). However, increased breath sounds are expected with pneumonia, and it would not explain subcutaneous crepitus Choice B= Bronchial obstruction with alveolar air reabsorption describes atelectasis (e.g., due to a large mucus plug). Shortness of breath and decreased breath sounds are typical, but hypotension and subcutaneous crepitus are not expected Choice C= Diffuse bronchial constriction occurs with asthma exacerbation and is not typical of cystic fibrosis. Hypotension and unilaterally decreased breath sounds are not expected with asthma exacerbation Choice D= Increased dead-space ventilation occurs with pulmonary embolism. Although hypotension (i.e., obstructive shock) can occur, decreased breath sounds and subcutaneous crepitus are not typical

-Age: 72 years man -Brought to the hospital due to a 1-week history of progressive confusion and lethargy. He has also had a persistent cough for the past several weeks with 2 episodes of blood in the sputum. -The patient has hyperlipidemia, well-controlled hypertension, and a 48-pack-year smoking history. On examination, mucous membranes are moist. Lung auscultation reveals wheezing in the left lung. Serum sodium is 123mEq/dl. Chest x-ray reveals a mass in the upper lobe of the left lung. A bronchoscopic biopsy of the mass is performed, and light microscopy of the tissue confirms an aggressive lung cancer. -Which of the following is most likely to be present on further analysis of this patient's biopsy sample? a)Epidermal growth factor receptor gene mutation b)Glial fibrillary acidic protein c)KRAS mutation d)Mucin e)Neural cell adhesion molecule f)Vimentin

Answer: Choice E (Neural cell adhesion molecule) -This patient with an aggressive lung cancer has symptomatic hyponatremia (e.g., confusion, lethargy) despite appearing euvolemic (moist mucous membranes). This presentation suggests small cell carcinoma of the lung complicated by paraneoplastic syndrome of SIADH -Small cell carcinoma, the most aggressive type of lung cancer, typically occurs in patients with a history of heavy smoking. It is characterized by a rapid doubling time and early distant spread; most patients have metastatic disease at the time of diagnosis. Although small cell carcinoma is sensitive to chemotherapy and radiation, most patients relapse within months, and the 5-year survival rate is very low. This malignancy frequently synthesizes hormones or hormone-like substances, resulting in paraneoplastic syndromes (e.g., vasopressin resulting in SIADH, ACTH resulting in Cushing syndrome) -Small cell carcinoma shows evidence of neuroendocrine differentiation. These tumors stain for neuroendocrine markers, such as neural cell adhesion molecule (CD56), neuron-specific enolase, chromogranin, and synaptophysin. Neurosecretory granules can be identified in the cytoplasm of the tumor cells on electron microscopy Choice A= Epidermal growth factor receptor gen mutations are present in some non-small cell lung cancers (NSCLCs), most commonly in adenocarcinoma in nonsmokers. This mutation can predict responsiveness to tyrosine kinase inhibitor drugs (e.g., erlotinib, afatinib). It is not present in small cell lung cancer Choice B= Glial fibrillary acidic protein-positive fibrils are seen with some astrocytomas Choice C= KRAS is an activating mutation that is present in up to 25% of lung adenocarcinomas and is associated with smoking. It is also frequently seen in colorectal and pancreatic cancer Choice D= Mucin is a glycoprotein that frequently provides a protective barrier to epithelial cells. However, the MUC gene is overexpressed in a variety of malignancies (e.g., breast, NSCLC, pancreatic) and is a marker of poor prognosis Choice F= Vimentin is an intermediate filament found within cells of mesenchymal origin and is used in diagnosing sarcomas

-Age: 57 years man -Comes to the office due to unrefreshing sleep. He feels tired during the day and occasionally has to nap during his lunch hours. According to his wife, the patient snores loudly during sleep and frequently gasps for breath. He also has severe claustrophobia -Past medical history is significant for hypertension. The patient takes no sedative medications and is a lifetime nonsmoker. BP (156/94), BMI (30). Physical examination is significant for a bulky tongue and crowded, narrow oropharynx. Electrical stimulation of which of the following nerves may improve the pathophysiologic cause of this patient's symptoms? a)Hypoglossal b)Lingual c)Maxillary d)Phrenic e)Recurrent laryngeal

Answer: Choice A (hypoglossal) -Obstructive sleep apnea (OSA) is characterized by recurrent episodes of upper airway collapse during sleep. Anatomical and neuromuscular mechanisms have been implicated in OSA. Neuromuscular weakness as a pathogenic mechanism in OSA is supported by the fact that apneas occur only during sleep, a time of muscle relaxation. The upper airway dilator muscles weaken during the transition from wake to sleep, leading to airway narrowing and ultimately collapse in individuals with OSA -Stimulation of the hypoglossal nerve using an implantable nerve stimulator causes the tongue to move forward slightly, increasing the anteroposterior diameter of the airway. Studies with these devices have shown a reduction in the number of obstructive events during sleep Choices B and C= The lingual nerve arises from the mandibular division of the trigeminal nerve and supplies sensory innervation to the tongue. The maxillary division of the trigeminal nerve provides sensation to the mid-face area. Neither of these nerves provides motor innervation to the oropharyngeal muscles involved in OSA Choice D= The phrenic nerve supplies the diaphragm. Although diaphragmatic paralysis can cause sleep apnea, OSA with loud snoring and gasping respirations is due to oropharyngeal rather than diaphragmatic dysfunction Choice E= The vocal cords are innervated by the recurrent laryngeal nerve, but sleep apnea is primarily a problem of the pharynx, not larynx

-Age: 32 years man -Comes to the hospital due to 1 week of progressive shortness of breath and cough. He also reports sharp right-sided chest pain that is worsened by deep inspiration. Examination reveals decreased tactile fremitus, dullness on percussion, and decreased breath sounds over the right lower lung. Chest x-ray shows consolidation of the right lower lobe and right-sided effusion. -A thoracentesis is performed, during which the needle is inserted along the upper border of the 10th rib at the right midaxillary line. Which of the following structures is most at risk of being injured as a result of the intervention? a)Hepatic veins b)Intercostal artery c)Intercostal nerve d)Right hepatic lobe e)Right lower lung lobe

Answer: Choice D (Right hepatic lobe) -The visceral pleura covers the surface of the lung, whereas the parietal pleura lines the inner surface of the chest wall and diaphragm. The parietal pleura generally extends approximately 2 ribs below the inferior margin of the lungs. The potential space at the reflection of the costal pleura onto the diaphragmatic pleura is called the costodiaphragmatic recess, where pleural fluid accumulates when the body is erect -Thoracentesis is typically performed between the 6th and 8th ribs along the midclavicular line, the 8th and 10th ribs along the midaxillary line (right image), and the 10th and 12th ribs along the paravertebral line. This allows pleural fluid to be drained without risking lung injury (which can occur at higher insertion sites). However, insertion of a needle below the 9th rib still risks penetrating abdominal structures -In this case, thoracentesis is being performed along the upper border of the 10th rib at the right midaxillary line, putting the patient at risk of liver injury if the needle is inserted too deep Answer: Choice D (Right hepatic lobe) -The visceral pleura covers the surface of the lung, whereas the parietal pleura lines the inner surface of the chest wall and diaphragm. The parietal pleura generally extends approximately 2 ribs below the inferior margin of the lungs. The potential space at the reflection of the costal pleura onto the diaphragmatic pleura is called the costodiaphragmatic recess, where pleural fluid accumulates when the body is erect -Thoracentesis is typically performed between the 6th and 8th ribs along the midclavicular line, the 8th and 10th ribs along the midaxillary line (right image), and the 10th and 12th ribs along the paravertebral line. This allows pleural fluid to be drained without risking lung injury (which can occur at higher insertion sites). However, insertion of a needle below the 9th rib still risks penetrating abdominal structures -In this case, thoracentesis is being performed along the upper border of the 10th rib at the right midaxillary line, putting the patient at risk of liver injury if the needle is inserted too deep

-Age: 63 years man -Admitted to the hospital after suffering a cerebral infarction. A swallow study performed during the acute recovery period shows oropharyngeal dysphagia. Despite being maintained on the appropriate dietary restrictions, the patient experiences an episode of vomiting while lying on his back and subsequently develops pneumonia -Which of the following lung regions is most likely to be affected? a)Anterior segment of the left upper lobe b)Basilar segments of the right lower lobe c)Lingular segments of the left lung d)Middle lobe segments of the right lung e)Posterior segment of the right upper lobe

Answer: Choice E (Posterior segment of the right upper lobe) -Due to gravity, aspiration pneumonia typically develops in the most dependent portions of the lung. Patients who aspirate while lying supine typically have involvement of the posterior segments of the upper lobes and the superior segments of the lower lobes -The right lung is more prone to aspiration than the left lung as the right main bronchus has a larger diameter, is shorter, and is more vertically oriented than the left main bronchus (mnemonic: "inhale a bite, goes down to the right") Choice B= Aspiration in an upright position tends to involve the basilar segments of the lower lobes Choices C and D= Recumbent patients in the prone position (i.e., lying on the abdomen) are more likely to aspirate into the middle lobe and lingula, as these are the most dependent portions of the lung in this position

-Age: 68 years man -Evaluated in the hospital due to a pleural effusion. He has a history of base of tongue squamous cell carcinoma with involvement of multiple left anterior cervical lymph nodes. Two days ago, the patient underwent surgery to remove the tumor and the anterior cervical lymph nodes. Today, he has increasing shortness of breath, and chest x-ray reveals a large left pleural effusion. Which of the following was most likely injured during this patient's surgical procedure? a)Esophagus b)Innominate artery c)Lung parenchyma d)Phrenic nerve e)Thoracic duct f)Trachea

Answer: Choice E (Thoracic duct) -This patient with a pleural effusion 2 days after cervical lymph node removal most likely has a chylothorax due to intraoperative injury of the thoracic duct -The thoracic duct is the largest lymphatic vessel. It originates at about the level of T12 in the abdomen where the lumbar and intestinal lymph trunks converge. It travels superiorly through the mediastinum (posterior to the esophagus) and drains into the junction between the left subclavian and jugular veins in the lower neck. It is most commonly injured during thoracic procedures (e.g., esophagectomy) but can be injured in neck procedures as well (e.g., cervical lymph node removal). Injury can result in leakage of lymph into either the neck or the thorax because a transected duct may retract into the chest -The thoracic duct collects lymph from most of the body, including the abdominal viscera, the viscera of the left hemithorax, all tissue inferior to the umbilicus, and all left-sided tissues superior to the umbilicus (right head, arm and thorax drain into the much smaller right lymphatic duct) Choices A and F= The trachea and esophagus are deeper structures in the central neck that are covered by strap muscles, they are unlikely to be injured during removal of cervical lymph nodes in the lateral neck. In addition, although injury to the esophagus and trachea can cause pleural effusion, such injury is typically accompanied by air accumulation (e.g., crepitus in the neck and/or pneumomediastinum) Choice B= The innominate artery is the first branch of the aortic arch, and it crosses the midtrachea from left to right (in the thorax). It does not travel through the left neck. In addition, injury to the innominate area would cause massive blood loss, resulting in immediate hemodynamic instability Choice C= The apex of the lung can project into the neck and can be injured in some neck surgical procedures. However, injury would cause an acute pneumothorax rather than a slowly accumulating pleural effusion Choice D= The phrenic nerve, which originates from C3-C5 and travels with the internal jugular vein, can be injured during neck procedures. However, such an injury would present with elevation of the hemidiaphragm

-Age: 22 years man -Brought to the ED after being involved in a fight at a nearby bar. The patient's RR (28), BP (88/60), HR (114). Physical examination shows a penetrating right-sided stab wound along the upper surface of the clavicle between the lateral border of the sternum and the midclavicular line. Which of the following structures was most likely injured? a)Accessory nerve b)Ansa cervicalis c)Carotid body d)Inferior thyroid artery e)Lung pleura

Answer: Choice E (lung pleura) -In patients with neck injuries, it is important to remember that the lung apices and cervical pleura extend above the clavicle and first rib through the superior thoracic aperture. Stab wounds immediately above the clavicle and lateral to the manubrium can puncture the pleura and cause pneumothorax, tension pneumothorax, or hemothorax -This patient most likely has a tension pneumothorax as a result of his pleural injury. As increasing volume of air accumulates within the pleural space, the lungs and mediastinum deviate to the opposite side of the chest. Increased pressure within the chest cavity decreases systemic venous return to the heart, lowering cardiac output. Signs and symptoms of tension pneumothorax include tachycardia, hypotension, tachypnea, hypoxemia, and hyperresonance to percussion and absence of breath sounds on the affected side. Treatment is with emergency needle thoracostomy or chest tube Choice A= The accessory nerve (cranial nerve XI) innervates the sternocleidomastoid and trapezius muscle. This nerve may be injured during surgery involving the posterior triangle of the neck (a region bounded by the sternocleidomastoid muscle, trapezius muscle, and clavicle) Choice B= The ansa cervicalis arises from C1, C2 and C3 nerve roots an innervates the sternohyoid, sternothyroid, and omohyoid muscles of the anterior neck. Penetrating trauma to the neck above the cricoid cartilage can injure this nerve Choice C= The carotid body, which contains O2, CO2 and H+ chemoreceptors, lies at the bifurcation of the common carotid artery (just inferior to the hyoid bone) Choice D= The inferior thyroid artery arises from the thyrocervical trunk and courses posterior to the carotid artery and jugular vein to supply the inferior pole of the thyroid gland. Injury to the inferior thyroid artery is commonly associated with hoarseness because it runs adjacent to the recurrent laryngeal nerve.

-Age: 3 years boy -Brought to the office with 5 days of productive cough and fever. Examination shows a tachypneic, ill-appearing child with rales over the right lower lung. Chest x-ray reveals right lower lobe pneumonia. The boy is admitted to the hospital for IV antibiotics. Review of his medical history shows 4 prior pneumonias, and his weight is at the 3rd percentile -Further testing demonstrates high sweat chloride content, and genetic sequencing shows a mutation in a transmembrane protein. Which of the following best describes the dysfunctional transmembrane protein causing this patient's decrease? a)ATP-gated b)Cyclic nucleotide-gated c)Mechanically gated d)Temperature-gated e)Voltage-gated

Answer: Choice A (ATP-gated) -Cystic fibrosis (CF) is an autosomal recessive disorder due to mutations (e.g., delta-F508) in the CF transmembrane conductance regulator (CFTR) protein. The channel pore opens after binding of 2 ATP molecules, allowing transport of chloride ions down the electrochemical gradient. The movement of chloride establishes a membrane potential that draws sodium and water across the membrane as well. By encouraging sodium chloride and water transport across epithelial membranes, the CFTR hydrates mucosal surfaces in the airways and bowel. Mutations in the CFTR gene lead to intracellular protein degradation and decreased activity of the proteins that do reach the membrane. This leads to thick mucus on the lining of epithelial cells, which results in clinical manifestations of chronic cough, recurrent pneumonias, and pancreatic damage -The normal CFTR protein also plays a role in the production of hypotonic sweat. In the eccrine gland, sweat is initially isotonic with the plasma, and normally sodium chloride is removed from the ductal lumen by CFTR. In the absence of functional CFTR, patients with CF cannot remove salt from their sweat and therefore have elevated sweat chloride and sodium levels. As early as the 17th century, increased mortality was recognized for children whose foreheads tasted salty. Today, the sweat chloride test is frequently used to screen for CF Choice B= Cyclic nucleotide-gated ion channels are important for photoreceptor and olfactory receptor neurons. These nonselective channels are directly activated by cyclic nucleotides (e.g., cAMP). In contrast, CFTR channels are modulated by cAMP-dependent phosphorylation, but they are not directly gated by cAMP binding Choice C= Mechanically gated (or stretch-activated) channels are ion channels that open in response to mechanical deformation of a cell membrane. Touch and hearing depend on mechanically activated channel transduction of mechanical force into electrochemical signals Choice D= Temperature-gated ion channels open in response to heat and are responsible for human sensation of external temperature Choice E= Voltage-gated ion channels open and close in response to changes in electrical membrane potential and are permeable to various ions. Congenital long QT syndrome is caused by a mutation in a voltage-gated potassium channel

-Age: 55 years woman -Comes to the office due to a persistent cough, exertional dyspnea, and recent, unintentional weight loss. She has no chronic medical problems and takes no medications. She has never smoked and has no history of exposure to industrial pollutants -She works as an accountant at a large accounting firm. Vital signs are normal. Physical examination reveals decreased breath sounds and dullness to percussion at the left lung base. Imaging studies show an irregular mass in the lower lobe of the left lung and a left pleural effusion. A diagnostic thoracocentesis is performed and the aspirated fluid is sent for cytological evaluation. If a malignancy is diagnosed, it is most likely to be which of the following types? a)Adenocarcinoma b)Bronchial carcinoid c)Mesothelioma d)Small cell carcinoma e)Squamous cell carcinoma

Answer: Choice A (Adenocarcinoma) -The nonsmoking woman with cough and weight loss has a peripheral lung mass with an associated pleural effusion. This presentation is concerning for adenocarcinoma of the lung, the most common lung cancer in the general population, women, and nonsmokers. Although tobacco is the most significant risk factor, the etiology in nonsmokers is thought to be multifactorial. Risk factors include environmental exposures (e.g., radon, second-hand smoke) and underlying lung disease. In addition, certain molecular alterations are seen more commonly in nonsmokers; these "driver-mutations" are though to be responsible for the development of the malignancy and include EGFR mutations and ALK gene rearrangements. KRAS mutations, another driver in the formation of adenocarcinoma, is seen more commonly in smokers -Adenocarcinoma typically presents with cough, weight loss, and hemoptysis. Changes to the distal extremities, including clubbing (increased curvature of the nail plate) and hypertrophic osteoarthropathy (proliferation of the bones, skin, and nails of the fingers), may occur and are thought to be due to tumor-derived growth factors. The tumor tends to be located peripherally (as in this patient) and may present as a nodule, mass, or obstructive pneumonia. Like all malignancies, metastatic disease to the pleura can cause pleural effusions Choice B= Bronchial carcinoid is a rare malignancy that tends to be located centrally, within or adjacent to large bronchi. It typically presents with wheezing due to bronchial obstruction and occasionally also causes carcinoid syndrome (i.e., episodic diarrhea, flushing, bronchospasm) Choice C= Mesothelioma most commonly arises from the mediastinum lining the pleural cavity; tumors are pleural based (i.e., pleural mass with associated pleural effusion, pleural thickening), rather than intraparenchymal. In addition, it is extremely rare in nonsmokers with no history of asbestos exposure Choice D= Small cell carcinoma is strongly associated with smoking; only 1% of cases occur in nonsmokers. It arises from the basal cells of the bronchial epithelium and displays some neuroendocrine features. Small cell carcinoma usually occurs in major bronchi Choice E= Squamous cell carcinoma is also strongly associated with smoking. It usually affects larger bronchi (centrally located) and arises from areas of squamous bronchial metaplasia

-Age: 55 years man -Comes to the ED due to several weeks of productive cough with occasional hemoptysis, weight loss, and low-grade fever. He has lost 7kg over the past 3 months. Medical history is significant for type 2 diabetes and chronic kidney disease, and the patient works as a prison guard. Temp. (37.8C), BP (110/70), pulse (78), RR (18) -Chest x-ray reveals a right apical lung infiltrate and cavitary lesions. He is placed in respiratory isolation, and appropriate empiric therapy is started. The cavitary lesions seen in this patient most likely formed through which of the following pathogenic mechanisms? a)Aggregation of activated leukocytes b)Bacterial toxin-induced cell necrosis c)Exudation and alveolar hepatization d)Intraalveolar bacterial overgrowth e)Obliterative lower airway inflammation

Answer: Choice A (Aggregation of activated leukocytes) -This patient has a productive cough, hemoptysis, fever, and weight loss with an apical cavitary lung lesion, findings characteristic of active TB. Mycobacterium TB is an acid-fast bacillus that replicates within the alveoli. The bacteria are phagocytized by alveolar macrophages, which are initially unable to destroy the pathogen due to microbial adaptations that inhibit the formation of an effective phagolysosome. However, in the weeks after the initial infection, antigen presenting cells (e.g., macrophages, dendritic cells) display mycobacterial antigens to naïve CD4+ T cells in regional lymph nodes, which subsequently differentiate into T helper type 1 (Th1) cells that secrete interferon-gamma -Interferon-gamma activates macrophages, leading to the formation of fully acidified phagolysosomes capable of destroying intracellular mycobacteria. Activated macrophages also aggregate and differentiate into epithelioid histiocytes and multinucleated giant cells, which surround extracellular mycobacteria within granulomas. The release of proteases, nitric oxide, and ROS by these cells helps contain the infection. However, these compounds also cause extensive collateral tissue damage and can result in the formation of cavitary lung lesions Choice B= M. TB produces trehalose dimycolate (cord factor), which inhibits phagolysosome maturation and may contribute to the formation of caseating granulomas. However, cord factor does not directly cause the tissue necrosis associated with cavitary lesions; the release of digestive enzymes by activated macrophages lining the granuloma drives caseation and cavitation Choice C= Lobar pneumonia is characterized by the initial accumulation of an abundant proteinaceous exudate within the alveoli, followed by hepatization of the affected lobe. Lobar pneumonia is typically caused by pathogens such as S. pneumoniae. However, lobar pneumonia presents acutely, not slowly, over several weeks Choice D= Intraalveolar mycobacteria do not directly cause cavitary lesions. Activated macrophages that surround the extracellular mycobacteria are primarily responsible for the tissue damage that leads to cavitation Choice E= Obliterative lower airway inflammation can occur as a feature of cryptogenic organizing pneumonia. In this condition, inflammation causes the proliferation of granulation tissue, which obstructs small bronchioles and leads to alveolar consolation. However, apical cavitary lesions and hemoptysis are uncommon

-Age: 55 years man -Hospitalized with severe abdominal pain associated with nausea and vomiting. Laboratory studies show marked elevation of serum amylase and lipase. He has history of heavy alcohol use. During hospitalization, his condition deteriorates, and he develops severe respiratory distress. There are crackles bilaterally on physical examination and diffuse patchy infiltrates bilaterally on chest x-ray. -The patient fails to improve with mechanical ventilation and 100% oxygen and dies 4 days later due to progressive respiratory failure. Which of the following autopsy findings is most likely in this patient? a)Alveolar hyaline membranes b)Cardiomegaly c)Foci of anaerobic pulmonary infection d)Intraalveolar hemorrhage e)Lung hyperinflation

Answer: Choice A (Alveolar hyaline membranes) -This patient with pancreatitis and subsequent respiratory failure likely has acute respiratory distress syndrome (ARDS). Pancreatitis is a major risk factor for ARDS as it results in the release of large amounts of inflammatory cytokines and pancreatic enzymes into the circulation, which leads to infiltration of neutrophils into the pulmonary interstitium and alveolar spaces. Diffuse injury to the alveolar epithelium and pulmonary microvascular endothelium results in a leaky alveolocapillary membrane and significant pulmonary edema. -ARDS is typically characterized by progressive hypoxemia refractory to oxygen therapy and diffuse interstitial edema in the absence of cardiogenic causes. During the first 1-6 days, interstitial and intraalveolar edema, inflammation, and fibrin deposition cause the alveoli to become lined with waxy hyaline membranes. These membranes consist of fibrin exudate and inspissated protein-rich edema fluid mixed with the remnants of necrotic epithelial cells Choice B= If this patient's rapid-onset dyspnea were due to cardiogenic pulmonary edema, as can occur in decompensated heart failure, or if the patient had chronic heart failure due to alcohol use, chest x-ray would likely reveal cardiomegaly Choice C= This patient is at risk of aspiration; however, aspiration pneumonia due to anaerobic bacteria commonly presents more indolently and is more likely to show an abscess or empyema than diffuse patchy opacities on chest x-ray Choice D= Although disseminated intravascular coagulation and ARDS can results in alveolar hemorrhage, the absence of hemoptysis makes this choice less likely Choice E= Lung hyperinflation may be seen in emphysema, which is a chronic, destructive enlargement of the air spaces distal to the terminal bronchiole

-Age: 60 years woman -Brought to the hospital due to progressive shortness of breath and cough for 2 weeks. The patient was diagnosed with right breast cancer and completed radiation therapy 6 weeks ago. -Temp. (37.3C), BP (110/70), pulse (90), RR (22). Oxygen saturation is 93% on room air. Examination reveals inspiratory crackles over the right lung anteriorly; the lungs are otherwise clear to auscultation. Chest radiograph shows ground-glass opacities in the right lung where radiation was delivered. Laboratory studies reveal no significant changes. -Acute radiation-induced lung injury is suspected. Histopathologic examination would most likely reveal formation of which of the following? a)Alveolar hyaline membranes b)Hemosiderin-laden macrophages c)Necrotizing vasculitis d)Neutrophilic abscess

Answer: Choice A (Alveolar hyaline membranes) Radiation-induced lung injury usually occurs following thoracic irradiation for the treatment of malignancy (e.g., breast or lung cancers, lymphoma). Ionizing radiation causes DNA damage directly (e.g., double-strand breaks) and indirectly through generation of reactive oxygen and nitrogen species that can damage both DNA and other cellular components In the lung, this damage primarily affects the alveolar-capillary barrier, formed from alveolar epithelial cells (i.e., pneumocytes) and vascular endothelial cells, and initiates an inflammatory response with a cascade of cytokines (e.g., IL-1, TNF-alpha) and growth factors (e.g., PDGF, TGF-beta). Manifestations vary based on chronicity: -Acutely: immune-mediated damage results in impaired gas exchange due to exudative alveolitis, with sloughing of pneumocytes and vascular endothelial cells, alveolar/interstitial edema, and hyaline membrane formation. Clinically, this usually manifests as dyspnea and cough 3-12 weeks after initial radiation exposure -Radiation fibrosis is a delayed manifestation of radiation-induced lung injury that can develop 6-12 months after initial radiation exposure. TGF-beta released by immune cells and the surrounding tissue initiates fibroblast proliferation, collagen deposition, and formation of dense fibrous bands. The result is decreased lung volume, dilated bronchi and bronchioles (i.e., traction bronchiectasis), impaired mucociliary clearance, and predisposition to chronic infections Choices B and C= Hemosiderin-laden macrophages (due to diffuse alveolar hemorrhage) and necrotizing vasculitis are both associated with autoimmune vasculitis (e.g., anti-GBM disease). These patients typically have hemoptysis and/or other systemic effects of vasculitis (e.g., glomerulonephritis, arthralgias) Choice D= Although significant neutrophilic infiltrate occurs in patients with radiation-induced lung injury, frank formation of an abscess usually occurs following introduction of anaerobic bacteria to the lung parenchyma due to aspiration (e.g., alcohol use disorder) or septic embolism

-Age: 45 years man -Comes to the ED due to severe dyspnea and chest discomfort that began earlier in the day. The dyspnea has been worsening throughout the day and has been present at rest. He reports no inciting trauma. The patient has no other medical conditions and takes no medications -He has a 20-pack-year smoking history and does not use alcohol or illicit drugs. Temp. (36.7C), BP (110/60), pulse (96). Arterial blood gas drawn on room air shows an arterial partial pressure of oxygen (PaO2) of 54mmHg and an arterial partial pressure of carbon dioxide (PaCO2) of 26mmHg. Which of the following processes is most likely occurring in this patient? a)Alveolar hyperventilation b)Decreased chest wall compliance c)Expiratory air trapping d)Poor respiratory drive e)Respiratory muscle fatigue

Answer: Choice A (Alveolar hyperventilation) -This patient with acute-onset dyspnea and chest discomfort likely has an acute pulmonary embolism (PE). PE leads to increased dead-space ventilation with a consequent ventilation/perfusion (V/Q) mismatch that causes hypoxemia. The acute hypoxemia along with pulmonary parenchymal inflammation triggers an increase in respiratory drive and hyperventilation -Because the rate of CO2 removal is closely tied to ventilation rate, alveolar hyperventilation leads to increased expiration of CO2 with resulting hypocapnia (low PaCO2) and respiratory alkalosis. In contrast, the rate of O2 absorption becomes capped once hemoglobin is saturated, which occurs at relatively low PaO2 levels (e.g., hemoglobin is 85% saturated at PaO2 50mmHg). Therefore, the blood in highly ventilated lung regions cannot absorb extra O2 to compensate for poorly ventilated regions, and hyperventilation does not significantly increase PaO2 -It follows that patients with an acute V/Q mismatch (e.g., due to PE or pneumonia) typically have hypocapnia with respiratory alkalosis and persistent hypoxemia. Without treatment, prolonged hyperventilation can lead to respiratory muscle fatigue (Choice E) with consequent respiratory failure; these patients will have hypoventilation with hypercapnia and respiratory acidosis Choices B and D= Both decreased chest wall compliance, as occurs in obesity hypoventilation syndrome, and poor respiratory drive, as occurs in opioid overdose and some types of stroke, lead to hypoventilation with hypoxemia and increased PaCO2 levels Choice C= Expiratory air trapping occurs in COPD. These patients often have chronic respiratory acidosis (high PaCO2) that may worsen (acute on chronic respiratory acidosis) during exacerbation

-Age: 52 years man -Brought to the ED after being found unresponsive on the street. The patient's medical history is unknown. Temp. (36.2C), BP (108/62), pulse (72). On physical examination, he is unresponsive to verbal and tactile stimuli but moans to deep sternal rub -An arterial blood gas analysis on room air shows a partial pressure of oxygen of 60mmHg. The partial pressure of oxygen in his alveoli is calculated to be 68mmHg. Which of the following is the most likely cause of this patient's symptoms? a)Alveolar hypoventilation b)Dead space ventilation c)Diffusion impairment d)Intrapulmonary shunt e)Left-to-right shunt

Answer: Choice A (Alveolar hypoventilation) -The partial pressure of oxygen in the alveoli (PAO2) is normally 104mmHg, and due to the high rate of O2 diffusion across the alveolar-capillary membrane, the O2 level in the alveolar capillary blood rapidly equilibrates with the PAO2. However, the blood O2 level then drops slightly due to the addition of deoxygenated blood from the bronchial circulation; therefore, the partial pressure of oxygen in arterial blood (PaO2) is normally around 100mmHg. This discrepancy between alveolar and arterial O2 concentration is termed the alveolar-arterial (A-a) gradient. In healthy individuals, a normal A-a gradient is typically between 4 and 15mmHg, with older individuals having higher normal values due to an age-related decline in O2-diffusing capacity -This patient has low PaO2 and PAO2 with a normal A-a gradient (68-60= 8), indicating that his low PaO2 is directly due to low PAO2. Possible causes of hypoxemia in the setting of a normal A-a gradient include alveolar hypoventilation and low partial pressure of inspired (PiO2) (i.e., high altitude). Common causes of alveolar hypoventilation include suppressed central respiratory drive (e.g., sedative overdose) and diseases that decrease inspiratory capacity (e.g., myasthenia gravis, obesity) Choices B and D= Dead space ventilation is one extreme of ventilation-perfusion mismatch that occurs when the alveoli are adequately ventilated, but there is no alveolar perfusion (e.g., pulmonary embolism). An intrapulmonary shunt is the other extreme of ventilation-perfusion mismatch that occurs when blood perfuses alveoli that are not ventilated (e.g., pneumonia, pulmonary edema). The A-a gradient is elevated in ventilation-perfusion mismatch Choice C= Gas diffusion is impaired in diseases that disrupt the alveolar-capillary membrane, such as alveolar hyaline membrane disease (acutely) or emphysema (chronically). Diffusion impairment causes an elevated A-a gradient because O2 cannot be effectively transported into blood Choice E= A left-to-right shunt occurs when oxygenated blood from the left side of the heart is shunted to the right side of the heart (e.g., ASD or VSD). Left-to-right shunts do not cause hypoxemia; however, if left untreated, they may progress into a right-to-left shunt (i.e., Eisenmenger syndrome) with hypoxemia, cyanosis, and an elevated A-a gradient

-Physiologists conduct a study to identify the protein constituents present in normal pulmonary secretions. They recruit 20 healthy men and women who do not take any medications and have no history of tobacco use. -Bronchoalveolar lavage is performed, and the proteins in the fluids are isolated. A specific protein is found that can degrade the elastin present in the extracellular matrix. Which of the following cells are most likely producing this protein? a)Alveolar macrophages b)Ciliated epithelium c)Club cells d)Goblet cells e)Type I pneumocytes f)Type II pneumocytes

Answer: Choice A (Alveolar macrophages) -Elastase is a neutral protease contained in macrophage lysosomes and in the azurophilic (primary) granules of neutrophils. Normally, elastase released from alveolar macrophages and infiltrating neutrophils is balanced by the presence of serum and tissue protease inhibitors. Neutrophil elastase is inhibited by serum alpha-1 antitrypsin, and macrophage elastase is inhibited by tissue inhibitors of metalloproteinases. Neutrophils and macrophage elastases can also degrade each other's (but not their own) inhibitors, augmenting their destructive capacity when both proteases are present. Excess protease activity is a major contributor to the development of both centriacinar and panacinar emphysema Choice B= The ciliated epithelium of the lower respiratory tract sweeps foreign particles and mucus toward the pharynx to promote mucociliary clearance Choice C= Club (formerly Clara) cells are nonciliated cells found predominantly in the terminal portions of the bronchioles. They secrete club cell secretory protein (which protects against airway inflammation and oxidative stress) and surfactant components (which prevent bronchiolar collapse) Choice D= Goblet cells secrete mucin to help with mucociliary clearance. They are present in the epithelial lining of the trachea and bronchi but are not present in the more distal airways (i.e., terminal bronchioles, respiratory bronchioles, alveoli) Choice E= Type I pneumocytes constitute over 95% of the surface area of the alveoli. These end-differentiated squamous cells are not a major source of alveolar fluid secretory products Choice F= Type II pneumocytes secrete the major components of pulmonary surfactant, including phospholipids such as dipalmitoyl phosphatidylcholine and surfactant-associated proteins

-Age: 45 years man -Comes to the office due to progressive dyspnea on exertion and cough for the past several weeks. He has had no fever, chills, chest pain or leg swelling. The patient is a former smoker with 5-pack-year history. Temp. (37.2C), BP (120/70), pulse (82), RR (16) -The patient is hypoxemic at rest, with an oxygen saturation of 88% on room air. Jugular venous pressure is normal. Lung auscultation shows scattered crackles. Heart sounds are normal. -Chest x-ray reveals bilateral midalveolar and lower alveolar opacities. Transbronchial lung biopsy reveals lipoproteinaceous material that is positive on periodic acid-Schiff stain and fills up the terminal bronchioles and alveoli. Electron microscopy of the substance shows lamellar bodies. Impaired function of which of the following most likely caused excessive accumulation of the substance in this patient's alveoli? a)Alveolar macrophages b)Ciliated airway epithelium c)Club cells d)Fibroblasts e)Type 1 pneumocytes

Answer: Choice A (Alveolar macrophages) -This patient's presentation is consistent with pulmonary alveolar proteinosis (PAP), a rare condition characterized by progressive respiratory dysfunction due to the accumulation of surfactant debris within alveolar spaces. Surfactant is a lipoproteinaceous material that appears pink with periodic acid-Schiff staining; it forms concentrically laminated structures (i.e., lamellar bodies) that can be seen on electron microscopy -PAP is caused by an imbalance between surfactant production and clearance. In healthy lungs, surfactant is secreted by type 2 pneumocytes and eventually cleared by alveolar macrophages. In PAP, surfactant removal is impaired due to compromised alveolar macrophage function (e.g., usually because of defects in granulocyte-monocyte colony-stimulating factor (GM-CSF) signaling) -Treatment involves therapeutic whole-lung lavage to wash away surfactant and inhaled GM-CSF replacement therapy Choice B= Ciliated epithelial cells line the conducting airways and are responsible for mucociliary escalator function, a key process in airway clearance. Primary ciliary dyskinesia (e.g., Kartagener syndrome) is characterized by impaired ciliary function and leads to mucostasis, recurrent pneumonia and bronchiectasis Choice C= Club cells (bronchiolar cells) are located at the bronchoalveolar alveolar junction and protect and repair the distal airway (terminal bronchioles). Club cell dysfunction has been implicated in the pathogenesis of COPD Choice D= Fibroblasts produce and maintain the connective tissue matrix of the lung, lending structural support and elasticity. Loss of fibroblast function contributes to degenerative airspace dilation in emphysema Choice E= Type 1 pneumocytes form the alveolar gas-exchanging surface. Rapid destruction and loss of type 1 pneumocytes occurs in early acute lung injury. It does not result in the abnormal accumulation of surfactant products

-Age: 12 years boy -Brought to the clinic for a follow-up examination. The patient was admitted to the hospital 3 months ago after a neonatal drowning event that resulted in severe acute respiratory distress syndrome. He was intubated and mechanically ventilated for 1 week and then discharged 2 weeks later. The patient currently feels well and reports mild dyspnea with exertion -Which of the following parameters is most likely increased in this patient compared to a child with normal cardiopulmonary function? a)alveolar-arterial oxygen difference b)Diffusion capacity of the lung c)Forced vital capacity d)Left atrial pressure e)Lung compliance

Answer: Choice A (Alveolar-arterial oxygen difference) -Acute respiratory distress syndrome (ARDS) refers to inflammatory lung damage triggered by alveolar injury (e.g., aspiration of water) or intense systemic inflammation (e.g., sepsis, pancreatitis). ARDS has a mortality rate of approximately 40%; those who survive often have reduced lung function that persists for months or years and is sometimes permanent -The pathogenesis of ARDS can progress through 3 phases, although most patients move through only the first 2. The exudative phase (phase 1) involves inflammatory disruption of the alveolar-capillary membrane with leakage of protein-rich fluid from the alveolar capillaries into the alveoli and surrounding interstitium. Gas exchange is acutely impaired by alveolar edema and by hyaline membranes that develop on the alveolar surface. After 1-2 weeks, ARDS progresses to the proliferative phase (phase 2), during which endothelial cells, pneumocytes, and fibroblasts proliferate in an attempt to repair damage. Collagen is deposited in the interstitium and may lead to scarring. -The majority of ARDS survivors have a persistent reduction in diffusion capacity that eventually normalizes after several years (Choice B). These patients may remain in a prolonged proliferative phase in which interstitial collagen deposition and remodeling impair gas diffusion across the alveolar-capillary membrane, causing an increased alveolar-arterial oxygen gradient that manifests clinically with mild dyspnea. A small percentage of surviving patients likely enter and remain in the fibrotic phase (phase 3) of ARDS, during which severe interstitial fibrosis develops and manifests as an irreversible reduction in diffusion capacity with marked clinical symptoms (e.g., severe dyspnea) Choices C and E= Collagen deposition during recovery from ARDS typically leads to decreased lung compliance, which in some patients manifests as a restrictive defect with reductions in both forced vital capacity and forced expiratory volume in 1 second on pulmonary function testing. The reductions in expiratory lung volumes typically normalize within 6 months Choice D= The pulmonary edema in ARDS in noncardiogenic, meaning it does not result from heart failure and involves normal left atrial pressure (estimated by pulmonary capillary wedge pressure) during both the acute disease and recovery

-Age: 28 years man -Previously healthy, comes to the urgent care clinic due to shortness of breath. The patient is on a ski vacation in Breckenridge, Colorado, which is situated at an altitude of 2926m (9600ft). On arrival 4 days ago, he experienced mild headache and nausea that resolved spontaneously, but since yesterday, he has had worsening dyspnea and cough -The patient now feels short of breath even with minimal exertion. Temp. (37.3C), BP (134/82), pulse (98), RR (22). Oxygen saturation is 86% on ambient air, which rapidly improves to 95% with supplemental oxygen. Physical examination shows pink mucous membranes, flat neck veins, bilateral inspiratory crackles, and no heart murmurs or pedal edema. Chest x-ray reveals patchy alveolar infiltrates. Which of the following is the most likely cause of this patient's current condition? a)Alveolar-capillary membrane disruption b)Embolic pulmonary arterial occlusion c)Excessive decrease in PaCO2 level d)Impaired left ventricular function e)Increased total red blood cell mass

Answer: Choice A (Alveolar-capillary membrane disruption) -This patient with dyspnea, hypoxemia, and patchy alveolar infiltrates shortly after arriving at high altitude likely has high-altitude pulmonary edema (HAPE). HAPE can occur within several days of arrival at an altitude >2500m (>8000ft), with the risk of increasing as altitude increases. The pathophysiology is driven by the reduced partial pressure of oxygen (PiO2) at high altitude, which leads to hypoxic pulmonary vasoconstriction and consequent increased pulmonary arterial pressure. In some individuals with genetic predisposition, the hypoxic vasoconstriction is unevenly distributed, exposing the capillary beds in relatively less vasoconstricted areas to high perfusion pressure. This causes alveolar-capillary membrane disruption and leads to patchy, bilateral pulmonary edema -HAPE typically presents with dyspnea (progressive over 1-2 days) and cough (sometimes with hemoptysis). Accompanying symptoms of acute mountain sickness (e.g., headache, nausea) are often present, and physical examination reveals bilateral lung crackles. The hypoxemia characteristically improves with supplemental oxygen (likely due to alleviation of pulmonary vasoconstriction), which differentiates HAPE from other causes of diffuse pulmonary edema. Although potential fatal, HAPE usually resolves rapidly upon descent to lower altitudes Choice B= Acute pulmonary embolism causes dyspnea and hypoxemia; however, jugular venous distention and an absence of pulmonary edema (e.g., no auscultatory crackles, clear lungs on chest x-ray) are also expected Choice C= Hyperventilation leading to reduced PaCO2, and respiratory alkalosis is expected at high altitude. An extreme drop in PaCO2 can cause sever alkalosis with associated tingling, numbness, tetany, and possibly seizures, but it does not cause pulmonary edema Choice D= LV function is unaffected in HAPE (the pulmonary edema is noncardiogenic). MI can lead to acute LV dysfunction with pulmonary edema, but it would be expected to present with severe chest pain and to cause jugular venous distention Choice E= In response to the reduced PiO2 at high altitude, the body increases erythrocyte production to increase O2-carrying capacity. However, this secondary polycythemia does not contribute to pulmonary edema

-Age: 56 years man -Comes to the office after 2 episodes of low-volume hemoptysis. He describes it as "steaks of blood" in his sputum. The patient has had a chronic cough over the last several years, most prominently in the morning. He has also had several recent respiratory infections -The patient has smoked a pack of cigarettes daily for the last 40 years. Chest x-ray shows hyperinflated lungs but no infiltrates or masses. Bronchoscopy is performed and several suspicious foci of bronchial mucosa are biopsied. Microscopy of the biopsy sample reveals areas of stratified squamous epithelium. The pathogenesis of this patient's microscopic findings is most similar to which of the following conditions? a)Barrett esophagus b)Cervical cancer c)Hypertrophic cardiomyopathy d)Interstitial cystitis e)Minimal change disease

Answer: Choice A (Barrett esophagus) -Normal bronchi are lined mainly by pseudostratified ciliated columnar cells that propel inhaled particles towards the oropharynx for removal by swallowing or expectoration. Interspersed goblet cells produce mucus that traps inhaled debris. The ciliated and goblet cells of respiratory epithelium are together responsible for mucociliary clearance -In response to chronic irritation, such as smoking, the respiratory epithelium may be replaced with squamous epithelium. This change of cell types is called squamous metaplasia and is seen histologically as a transition to stratified polygonal epithelial cells. This transition is initially adaptive, as squamous cells are more resistant to irritation than columnar cells. However, the lack of cilia and goblet cells prevents effective mucociliary clearance and increases the risk of respiratory infections. Squamous bronchial metaplasia is reversible and may resolve on discontinuation of smoking, but persistent irritant exposure can cause progression to dysplasia or squamous call carcinoma (biopsy pic) -Other examples of metaplasia include Barrett esophagus, in which esophageal squamous epithelium is replaced by intestinal columnar epithelium in response to chronic acid exposure. The metaplasia seen in Barrett esophagus is also associated with an increased risk of malignancy Choice B= Squamous cell carcinoma of the cervix is caused by oncogenic strains of human papillomavirus (HPV) that integrate into the host cell genome, leading to expression of viral genes that inactivate host tumor-suppressor proteins. Although HPV infects the metaplastic epithelium at the cervical transformation zone, squamous metaplasia of the cervix is a normal response to vagina acidity and is not associated with malignancy Choice C= Hypertrophic cardiomyopathy is an autosomal dominant defect of the beta-myosin heavy chains. This abnormality leads to disordered myocardial fibers, left ventricular outflow obstruction, arrhythmia and increased risk of sudden death Choice D= Interstitial cystitis is characterized by urinary frequency, urgency, and pelvic pain. Gross findings include erythema and ulceration of the bladder mucosa. Histopathology shows inflammatory infiltrates and fibrosis Choice E= Minimal change disease is associated with effacement of the processes of glomerular podocytes and is visible only on electron microscopy. The underlying cell type is not altered

-Age: 56 years construction worker -Comes to the office after receiving a letter from his employer advising him of significant exposure to asbestos during a building project 25 years ago. He is currently asymptomatic and feels well. Medical conditions include hypertension and osteoarthritis, for which he takes amlodipine daily and ibuprofen as needed. -The patient quit smoking 10 years ago and does not use alcohol or illicit drugs. Physical examination is within normal limits. Chest imaging is normal. He is concerned about his risk of developing cancer. Due to his occupational exposure, this patient is at greatest risk of malignancy arising from which of the following? a)Bronchial epithelium b)Hematopoietic stem cells c)Liver parenchyma d)Nasopharyngeal epithelium e)Pancreatic ductal cells f)Pleural mesothelium g)Urinary bladder epithelium

Answer: Choice A (Bronchial epithelium) Asbestos is a fiber composed of hydrated magnesium silicates commonly used in the shipbuilding, construction, and textile industries. Inhalation of fine asbestos fibers causes epithelial cell injury, activation of macrophages, and chronic interstitial inflammation and fibrosis The major clinical manifestations of asbestos exposure include the following: -Pleural disease includes pleural effusions and pleural plaques. Pleural plaques are a hallmark of asbestos exposure that typically affect the parietal pleura along the lower lungs and diaphragm. The plaques are composed of discrete circumscribed areas of dense collagen that frequently become calcified -Asbestosis is characterized by progressive pulmonary fibrosis that is most predominant in the lower lobes and by the presence of asbestosis bodies (golden-brown beaded rods with translucent centers) -Bronchogenic carcinoma is the most common malignancy associated with asbestos exposure. Smoking and asbestos exposure have a synergistic effect on the development of lung carcinoma, increasing the risk from 6-fold in nonsmoking patients with asbestos exposure to 60-fold in asbestos-exposed patients who smoke regularly -Malignant mesothelioma is a rare malignancy of the pleura for which asbestos is the only known environmental risk factor. It is less common than bronchogenic carcinoma in asbestos-exposed patients (Choice F). However, mesothelioma is more specific for heavy asbestos exposure Choice B= Direct-acting alkylating agents and benzene exposure are associated with acute leukemias, which occur due to malignant proliferation of hematopoietic stem cells Choice C= Aflatoxin exposure, which can occur after exposure to contaminated crops (e.g., peanuts, corn), is a chemical risk factor associated with hepatocellular cancer; although viral hepatitis, cirrhosis, and alcohol abuse are common risk factors Choice D= Epstein-Barr viral infection and heavy alcohol use, particularly when combined with smoking, increase the risk of nasopharyngeal cancer Choice E= Pancreatic cancer is strongly associated with tobacco use and obesity. However, this patient's asbestos exposure places him at higher risk of developing bronchogenic carcinoma Choice G= Beta-naphthylamine is used in aniline dyes and the rubber industry; it is associated with increased incidence of bladder carcinoma

-Age: 29 years African American woman -Being evaluated for exertional dyspnea and dry cough. She has no significant medical history and takes no medications. The patient works part-time as a bird keeper at a zoo -Chest x-ray reveals bilateral hilar adenopathy and reticular pulmonary infiltrates. Serum ACE levels and calcium levels are elevated. Purified protein derivative testing is negative. A bronchoscopy is scheduled to help determine the diagnosis. Which of the following is most likely to be the predominant cell type in this patient's bronchoalveolar lavage fluid? a)CD4+ lymphocytes b)CD8+ lymphocytes c)CD22+ lymphocytes d)Eosinophils e)Mast cells f)Neutrophils

Answer: Choice A (CD4+ lymphocytes) -This patient's hilar adenopathy, hypercalcemia, and elevated ACE levels are suggestive of sarcoidosis, a condition characterized histologically by noncaseating granulomas composed of activated macrophages and T lymphocytes. Activated macrophages can ectopically produce ACE and 1-alpha-hydroxylase (increases production of 1,25-dihydroxycholecalciferol, the active form of vitamin D). This results in high ACE levels and hypercalcemia commonly seen in patients with sarcoidosis. Any organ can be affected; however, the lungs (interstitial lung disease), lymph nodes (hilar adenopathy), skin (nodular rash), and eyes (anterior uveitis) are most commonly involved -Sarcoidosis may resemble other interstitial lung diseases (e.g., hypersensitivity pneumonitis, cryptogenic organizing pneumonia). In such cases, quantification of the CD4+/CD8+ ratio in bronchoalveolar lavage (BAL) fluid may help in determining the diagnosis. Sarcoidosis is a CD4+ T-cell-mediated disease in which large amounts of CD4+ lymphocytes release interferon-gamma and TNF-alpha to drive macrophage activation and granuloma formation. Therefore, the BAL fluid in patients with pulmonary sarcoidosis demonstrates a lymphocytic predominance with a high CD4+/CD8+ ratio (>2:1) Choice B= CD8+ cells predominate in the BAL fluid of patients with hypersensitivity pneumonitis. Although this patient has an exposure risk for hypersensitivity pneumonitis (bird handling), her hilar adenopathy as well as elevated serum calcium and ACE levels make sarcoidosis more likely Choice C= CD19, 20, and 22 are markers for the B-cell lineage. Precursor B-cell lymphoblastic leukemia (CD22) may sometimes involve the pleura or present as diffuse, patchy pulmonary infiltrates. In such cases, the BAL fluid may contain CD22+ lymphocytes, however, elevated ACE and calcium levels suggest sarcoidosis Choice D= BAL fluid eosinophilia is found in eosinophilic pneumonias and tropical pulmonary eosinophilia. Although patients with hypersensitivity pneumonitis may have elevations in circulating eosinophils, they do not have elevated eosinophil counts in their BAL fluid Choice E= Even though the quantity of mast cells in BAL fluid may be elevated in various pulmonary diseases, there are a few, if any, conditions reported to have a predominance of such cells Choice F= Neutrophils would predominate in patients with bacterial pneumonias

-Age: 36 years man -Comes to the office due to chronic, mild dyspnea and fatigue. The patient also says that his mind seems "foggy" all the time and he is not able to think clearly. -He has no significant past medical history and does not take any medication. He lives a sedentary lifestyle and has smoked half-pack of cigarettes a day for 5 years. BP (140/85), pulse (84), RR (18), BMI (32). Lung auscultation is unremarkable. Chest x-ray is normal -Arterial blood gas analysis shows partial pressure of oxygen (PaO2) is 66mmHg and partial pressure of carbon dioxide (PaCO2) is 58mmHg. His estimated alveolar to arterial oxygen gradient is 10mmHg. Which of the following best explains this patient's laboratory findings? a)Chronic hypoventilation b)Decreased lung diffusing capacity c)Destruction of lung parenchyma d)Increased lung compliance e)Increased tissue oxygen consumption f)Right-to-left shunt

Answer: Choice A (Chronic hypoventilation) -This patient with chronic fatigue, dyspnea, difficulty concentrating, hypoxemia (PaO2 <75mmHg) and hypercapnia (PaCO2 >45mmHg) in the setting of obesity (BMI >30) likely has obesity hypoventilation syndrome (OHS). OHS results from physical restriction of lung expansion by excessive thoracic tissue mass. As such, the hypoventilation mostly results from reduced tidal volume and patients may have normal or increased respiratory rate. Most patients with OHS also have concomitant obstructive sleep apnea (OSA); unlike isolated OSA that involves hypoventilation only with sleep, OHS involves hypoventilation throughout the waking hours -In healthy individuals, a normal alveolar to arterial (A-a) gradient ranges from 4-15mmHg, with older individuals having a gradient towards the higher end of normal. A normal A-a gradient indicates that the efficiency of gas exchange between the alveoli and the blood is intact; therefore, hypoxemia must result from low partial pressure of alveolar oxygen (PaO2). Low PaO2 can occur due to alveolar hypoventilation (e.g., OHS, neuromuscular disease, central depression of respiratory drive) or inspiration of low partial pressure of oxygen at high altitude Choices B and C= Destruction of lung parenchyma occurs with both emphysema and interstitial lung disease. This leads to disruption of the alveolar capillary membrane and impaired diffusion capacity with an increased A-a gradient. In OHS, the lung parenchyma is undamaged and diffusion capacity is intact Choice D= Although the lung parenchyma is unaffected in OHS; restricted expansion of the chest wall secondarily restricts lung expansion, effectively decreasing lung compliance Choice E= Tissue oxygen consumption does increase with increased body mass, but it is a relatively minor factor and would not cause hypoxemia or hypercapnia in the absence of hypoventilation Choice F= Right-to-left shunting (e.g., Eisenmenger syndrome) results in hypoxemia with an elevated A-a gradient because a large percentage of cardiac output bypasses the alveolar capillaries and does not undergo gas exchange

-Age: 42 years man -Comes to the ED due to several hors of dyspnea. He is a long-haul truck driver who became acutely short of breath during a 36-hour trip but previously had been healthy. The patient smokes a pack of cigarettes daily and drinks a 6-pack of beer on weekends. -Pulmonary examination reveals vesicular breath sounds bilaterally with the overlying areas resonant to percussion. CT pulmonary angiogram shows a right lower lobe pulmonary artery occlusion consistent with pulmonary embolism but no evidence of pulmonary infarction -Which of the following is the most likely reason for the lack of lung tissue necrosis in this patient? a)Collateral circulation b)Decreased surfactant c)Endothelial tissue plasminogen activator d)Increased pulmonary venous pressure e)Intrapulmonary shunting

Answer: Choice A (Collateral circulation) -The lung is supplied by dual circulation from both the pulmonary arteries and bronchial arteries. This collateral circulation can help protect against lung infarction as a complication of pulmonary embolism (PE). -The pulmonary arteries are the major suppliers of blood to the lungs, providing deoxygenated blood for gas exchange; the bronchial arteries supply nutrients, remove waste from the bronchi, and provide collateral blood flow to the remainder of the lung parenchyma. When a clot occludes the pulmonary system, the bronchial system continues to supply nutrients to pulmonary tissue and can even backfill the pulmonary capillaries to partially maintain gas exchange in the area of the thromboembolism -Distal PEs in small arteries (<3mm) are more likely to cause infarction as they may occlude areas distal to the pulmonary-bronchial anastomoses. When a pulmonary infarction does occur, it is typically hemorrhagic (red) rather than ischemic (white) due to the relatively low density of lung tissue and the dual blood supply Choice B= Alveolar inflammation triggered by PE can eventually lead to decreased surfactant and some degree of atelectasis. This process can take up to 2 days to develop, and the resulting intrapulmonary shunting makes a small contribution to the hypoxemia that occurs with PE Choice C= Recombinant tissue plasminogen activator (TPA) may be used to treat PE in patients who are hemodynamically unstable. However, endothelial-derived TPA is limited primarily to the bronchial circulation, and spontaneous recanalization of the pulmonary artery is a slow process Choice D= PE leads to increased pulmonary arterial pressure; however, the pulmonary venous pressure is unchanged or decreased as blood flow through the proximal pulmonary circulation is obstructed. Patients with an underlying elevation in pulmonary venous pressure (e.g., decompensated heart failure) may be more likely to experience lung infarction with PE due to the high pulmonary venous pressure impairing collateral blood flow from the bronchial circulation Choice E= Intrapulmonary shunting occurs when an area of the lung is adequately perfused but poorly ventilated. PE causes intrapulmonary shunting due to redistribution of blood away from segments directly affected by the clot; the remaining accessible alveoli are unable to fully oxygenate all the blood passing through the pulmonary circulation, resulting in hypoxemia. In contrast, areas distal to the clot receive adequate ventilation but poor perfusion (i.e., dead space ventilation)

-Age: 34 years man -Comes to the office due to shortness of breath. He reports an episodic cough and chest tightness for the past 4 months. The patient was recently on a 2-week vacation in Arizona and reports that he had no symptoms during the trip but that they restarted when he returned home. He has no prior medical conditions and takes no medications -The patient smoked marijuana in his early 20s for several years but does not use tobacco or alcohol. He started working at a chemical manufacturing factory 7 months ago and has had no sick contacts. Temp. (36.7C), BP (120/80), pulse (76), RR (18) -On physical examination, expiration is mildly prolonged and there are scattered wheezes. Office spirometry shows an FEV1/FVC of 82%, and chest radiography reveals no abnormalities. Which of the following features is most likely involved in the pathogenesis of this patient's condition? a)Exposure to inhaled allergens b)Inhalation of fungal spores c)Inherited protease inhibitor deficiency d)Mutation in BMPR2 gene e)Necrotizing vasculitis of small vessels

Answer: Choice A (Exposure to inhaled allergens) This patient with dyspnea, cough, and intermittent chest tightness likely has asthma. The correlation of symptom onset after starting a new job and relief while traveling is suggestive of occupational asthma (OA), which accounts for up to 25% of adult-onset asthma OA is characterized by airway inflammation, bronchial hyperreactivity, and a variable airflow obstruction triggered by a workplace exposure. Like other forms of asthma, patients may have normal chest imaging and pulmonary function tests between exacerbations Inciting factors in OA can be categorized as immunologic or nonimmunologic: -Immunologic: similar to atopic asthma, exposure to workplace aero-allergens induces a Th2-mediated reaction, which stimulates IgE formation and eosinophilic activation. There is often a latent period before symptoms develop during which the patient is sensitized to an allergen. Common causes include cereals, latex, and chemicals (e.g., formaldehyde, amines, dyes) -Nonimmunologic: exposure to aero-irritants induces denudation of the airway mucosa, resulting in persistent airway inflammation, loss of epithelial relaxation factors, and mast cell degranulation. This typically occurs after large accidental exposures (e.g., chemical spill), and symptom onset is sudden and severe. Common causes include chlorine and ammonia Choice B= Coccidiomycosis is endemic in the southwestern US. In addition to cough and chest pain, patients commonly have fever, arthralgia, and skin manifestations, and imaging often demonstrates hilar adenopathy. This patient developed symptoms prior to his travel to Arizona Choice C= Alpha-1 antitrypsin deficiency is an inherited disorder that cause panacinar emphysema. Patients with dyspnea, cough, and wheezing; however, imaging typically reveals basilar bulbous changes and spirometry shows an obstructive pattern Choice D= BMPR2 gene mutations cause idiopathic pulmonary hypertension; symptoms include progressive dyspnea and fatigue. However, cough, wheezing and the resolution of symptoms while traveling is unexpected. In addition, imaging often reveals enlarged pulmonary arteries Choice E= Eosinophilic granulomatosis with polyangiitis (Churg-Strauss) is a vasculitis of small- or medium-sized arteries characterized by asthma and rhinitis. However, imaging typically demonstrates patchy infiltrates, and symptoms would not be expected to resolve while traveling

-Age: 21 years man -Brought to the ED after being found unresponsive in a public restroom. He has a history of injection drug use. Temp. (36.1C), BP (110/70), pulse (62), RR (8). On physical examination, the patient is unconscious with pinpoint pupils. Endotracheal intubation is performed, and mechanical ventilation is begun. The ventilator settings are adjusted to provide a minute ventilation of 6.0L/min. Calculated alveolar ventilation is 4.2L/min. Which of the following most likely accounts for the difference between these 2 parameters? a)Dead space volume b)Functional residual capacity c)Residual volume d)Respiratory rate e)Tidal volume

Answer: Choice A (Dead space volume) Minute ventilation is the total volume of new air that enters the respiratory pathways per minute. It can be calculated by multiplying the average tidal volume by the number of breaths per minute (minute ventilation (L/min)= tidal volume (L) x breaths/min) Alveolar ventilation refers to only the volume of new air reaching the gas exchange areas (i.e., alveoli and respiratory bronchioles) per minute. It can be calculated by subtracting dead space volume (which does not participate in gas exchange) from the tidal volume -Alveolar ventilation (L/min)= (tidal volume - dead space volume) x breaths/min The total volume of dead space in the lungs is known as physiologic dead space. It consists of both the anatomic dead space of the conducting airways (i.e., nose, trachea, bronchi, bronchioles; normally 150ml) and alveolar dead space due to well-ventilated by poorly perfused alveoli. Because it is difficult to directly measure physiologic dead space, it is often estimated in mechanically ventilated patients by comparing arterial (a) and expiratory (E) pCO2 levels: -Physiologic dead space= tidal volume x ((PaCO2 - PECO2)/PaCO2) Choices B and C= Functional residual capacity is the volume of air remaining in the lungs after a normal exhalation. It is the sum of the residual volume (the volume of air remaining in the lungs after maximal exhalation) and expiratory reserve volume (the volume difference between normal end-expiration and maximal end-expiration) Choice D= Respiratory rate is the number of breaths taken per minute and is a factor in calculating both minute ventilation and alveolar ventilation Choice E= Tidal volume is the volume of new air that enters the lungs with each normal, unforced breath. It is a factor in calculating both minute ventilation and alveolar ventilation

-Age: 32 years woman -Brought to the ED after being stuck in a malfunctioning elevator for 45 minutes. The patient says she felt extremely anxious in the closed space of the elevator and experienced dizziness, shortness of breath, generalized weakness, and blurred vision -She has no other medical conditions and takes no medications. She does not use tobacco, alcohol, or illicit drugs. Which of the following is the most likely cause of this patient's symptoms? a)Decreased arterial partial pressure of CO2 b)Decreased arterial partial pressure of O2 c)Decreased arterial pH d)Decreased arterial total O2 content e)Increased alveolar-arterial O2 gradient f)Increased arterial lactic acid content

Answer: Choice A (Decreased arterial partial pressure of CO2) -This patient most likely had a panic attack while trapped in the elevator. Panic attacks are typically accompanied by hyperventilation with a resulting decrease in arterial partial pressure of CO2 (PaCO2). This hypocapnia can cause decreased cerebral perfusion with consequent neurologic sequelae, including generalized weakness, blurred vision, presyncope (i.e., dizziness, lightheadedness), and syncope -Cerebral blood flow (CBF) remains relatively constant over a wide range of perfusion pressures (cerebrovascular autoregulation) and is mainly influenced by arterial blood gas levels, particularly changes in PaCO2. Hypercapnia triggers an increase in CBF (to aid in removal of toxins), and hypocapnia triggers a decrease in CBF. In fact, because hypocapnia decreases CBF, mechanically ventilated patients with cerebral edema are often hyperventilated to decrease intracranial pressure and help prevent brain herniation -CBF is less sensitive to the arterial partial pressure of oxygen (PaO2). PaO2 has little influence on CBF until the level drops below 50mmHg, at which point a rapid increase in CBF is triggered Choices B and D= The total blood O2 content is predominantly made up of hemoglobin-bound O2 (determined by the blood hemoglobin level and PaO2) plus a small contribution of dissolved O2. Hyperventilation increases the alveolar partial pressure of O2 and should somewhat increase (rather than decrease) both PaO2 and total blood O2 content Choice C= Hyperventilation produces respiratory alkalosis, marked by an increase in arterial pH Choice E= Disturbances that increase the alveolar-arterial O2 gradient include V/Q mismatch (e.g., pulmonary embolism), diffusion limitation, and right-to-left shunting (e.g., Eisenmenger syndrome). Changes in ventilation rate (i.e., hypo- or hyperventilation) have no effect on the alveolar-arterial O2 gradient Choice F= Increased arterial lactic acid content is a sign of tissue hypoxia (e.g., due to sepsis or ischemia), which is unlikely in the setting of hyperventilation from a panic attacks

-Age: 72 years woman -Brought to the ED due to exertional chest pain. The patient is on vacation with her family and her symptoms began after arriving in the Colorado mountains. She has a history of coronary artery disease but has had no ischemic symptoms over the past several years. -The patient is a lifetime nonsmoker and has no history of lung disease. Physical examination shows mild hypoxemia, clear lungs, an S4, and no extremity edema. ECG shows T-wave inversion in the lateral leads. Serum troponin and D-dimer levels are within normal limits -It is determined that decreased oxygen levels at high altitude in combination with age-related pulmonary changes lead to an exacerbation of this patient's preexisting heart condition. Which of the following respiratory changes most likely contributed to this patient's increased susceptibility? a)Decreased chest wall mobility b)Decreased dead space ventilation c)Increased chemoreceptor responsiveness d)Increased forced vital capacity e)Increased lung elastic recoil

Answer: Choice A (Decreased chest wall mobility) The reduced partial pressure of inspired oxygen at high altitude leads to hypoxemia (i.e., reduced partial pressure of arterial oxygen (PaO2)). Hyperventilation is the most immediate and important physiologic adjustment to high altitude; peripheral chemoreceptors stimulate increased minute ventilation (via both increased tidal volume and respiratory rate) to help increase PaO2 and minimize the symptoms of hypoxemia (e.g., headache, dyspnea) In older individuals (e.g., age >65), the body is less able to compensate for the hypoxemia at high altitude due to a number of age-related changes to the respiratory system. These changes include the following: -Decreased chest wall compliance: Calcification of the intercostal joints and tendon insertion sites, as well as degeneration of the spine (e.g., kyphosis), reduces chest wall expansion and limits the potential increase in tidal volume -Decreased alveolar elastic recoil: Degeneration of elastin causes increased alveolar compliance with alveolar enlargement and increased air trapping. This increases residual volume and decreases forced vital capacity, further limiting the achievable increase in tidal volume (Choices D and E) -Increased alveolar-arterial O2 gradient: the alveolar enlargement increases ventilation-perfusion mismatch and decreases the efficiency of O2 exchange in 2 ways. First, it decreases the percentage of alveolar surface area in contact with alveolar capillaries, effectively increasing dead space (Choice B). Second, the increased air trapping decreases ventilation of highly perfused alveoli at the base of the lungs In this elderly patient, the body's ability to increase PaO2 at high altitude has likely been reduced, and, given her underlying coronary artery disease, she is likely experiencing angina (e.g., exertional chest pain) due to inadequate O2 supply to the myocardium Choice C= Aging appears to have minimal effect on chemoreceptor responsiveness; therefore, respiratory drive is appropriately increased at high altitude. Age-related changes that reduce the mechanical efficiency of the respiratory system (e.g., ability to respond to increased respiratory drive) are primarily responsible for the reduced compensation for hypoxemia at high altitude

-Age: 56 years man -Evaluated for dyspnea and chest discomfort. Medical history includes hypertension, type 2 diabetes, and chronic kidney disease. Temp. (37.8C), BP (160/90), pulse (98), RR (24), BMI (31). -The patient undergoes a 2-part nuclear medicine test. During the first part, he breathes in a radioactive tracer and a lung scan is performed. In the 2nd part of the test, he is given an IV injection of a different radioactive material while breathing normal air, and the scan is repeated. When the images are compared, a large area of the lower right lung is not visualized in the 2nd scan. Based on the observed findings, which of the following underlying disease processes is most likely present in this patient? a)Deep venous thrombosis b)Endobronchial neoplasm c)Left ventricular infarction d)Spontaneous pneumothorax e)Viral pleuritis with effusion

Answer: Choice A (Deep venous thrombosis) -This patient, with dyspnea, chest discomfort, and a perfusion defect on ventilation-perfusion (V/Q) scan likely has a pulmonary embolism (PE). Pes are most commonly caused by deep vein thrombosis in the lower extremities that subsequently embolizes to the pulmonary vasculature. Signs and symptoms include acute-onset pleuritic chest pain, shortness of breath, tachycardia and hypotension. CT pulmonary angiography is typically the imaging modality of choice; however, V/Q scans are helpful in evaluating patients in whom angiography is contraindicated (e.g., contrast allergy, renal failure) -V/Q scans are 2-part studies that compare regional ventilation and perfusion. The initial phase uses a radiolabeled aerosol that is inhaled and delivered throughout the tracheobronchial tree. The 2nd phase uses an IV tracer that is distributed throughout the pulmonary vasculature. The images are then overlaid for comparison. Normal results show even distribution of both radionucleotide tracers throughout the lung (i.e., V/Q match). V/Q mismatch occurs when alveoli are ventilated but not perfused (i.e., a defect in the perfusion phase). This pattern can be seen in diseases that increase physiologic dead space, including pulmonary embolism (as in this patient) and malignancies that obstruct arterial blood flow Choices B, C and E= Endobronchial neoplasms can cause obstruction of the airway. LV infarction can cause pulmonary edema. Exudative pleuritis causes a pleural effusion, often with compression and atelectasis of the adjacent lung. These diseases cause ventilation defects, typically with matched perfusion defects as the resultant alveolar hypoxia induces pulmonary vasoconstriction. However, this patient has isolated perfusion defects Choice D= Spontaneous pneumothorax would cause compression of the ipsilateral lung; V/Q scan would show decreased lung volume on ventilation and perfusion images compared to the normal lung

-Age: 56 years woman -History: hypertension; brought to the ED due to 3 days of dysuria and back pain. Temp. (39.2C), BP (70/40), pulse (130), RR (28). The patient is confused and has suprapubic and costovertebral angle tenderness. -The skin is diffusely warm but there are no rashes or edema. Complete blood count demonstrates leukocytosis with increased neutrophils but is otherwise normal. Coagulation studies are normal. Urinalysis is positive for leukocyte esterase and nitrites, and microscopy shows numerous bacteria. Several hours later, the patient becomes increasingly hypoxic and requires mechanical ventilation. This patient's respiratory symptoms are most likely due to which of the following pathologic conditions? a)Fluid accumulation in the alveolar spaces b)Necrotizing inflammation with pulmonary hemorrhage c)Scattered noncaseating granulomas d)Thick mucus plugs in the bronchi and bronchioles e)Wedge-shaped areas of hemorrhagic necrosis

Answer: Choice A (Fluid accumulation in the alveolar spaces) This patient has a urinary tract infection complicated by fever and hemodynamic instability consistent with sepsis. Her rapid-onset respiratory failure is suggestive of acute respiratory distress syndrome (ARDS). ARDS is characterized by bilateral pulmonary infiltrates and hypoxemia in the absence of heart failure. It can occur due to direct pulmonary trauma (e.g., pulmonary contusions, inhaled irritants) or indirect nonpulmonary insults (e.g., sepsis, burns, pancreatitis) that result in pulmonary epithelial and/or endothelial injury The 3 phases of ARDS follow the disease's progression -Exudative phase: inflammatory cytokines (e.g., TNF, IL-1, IL-6) activate the pulmonary endothelium and recruit neutrophils to the lung tissue Resultant endothelial damage leads to increased capillary permeability and leakage of protein-rich fluid into the alveolar space. Organization of the edema and cellular debris leads to the formation of hyaline membranes -Proliferative phase: One to two weeks later, endothelial cells, pneumocytes, and fibroblasts proliferate in attempts to repair the damaged lung; collagen is deposited, and scarring may occur. Edema is reabsorbed -Fibrotic phase: in a minority of patients, excessive collagen deposition leads to irreversible pulmonary fibrosis and pulmonary hypertension Choice B= Necrotizing inflammation and pulmonary hemorrhage are commonly associated with granulomatosis with polyangiitis, which involves both the lungs and the kidneys. However, this condition normally presents subacutely with cough , hemoptysis, and nephritic syndrome, this patient's acute respiratory decline in the setting of sepsis is more consistent with ARDS Choice C= Noncaseating granulomas are found in sarcoidosis, which typically presents more chronically with cough, skin findings, or uveitis. Hemodynamic instability and acute respiratory failure would be unexpected Choice D= Mucus plugging typically occurs in patients with underlying lung disease (e.g., cystic fibrosis, COPD). Mucus plugs can cause hypoxemia but would not be expected to cause hypotension and fevers, and acute respiratory deterioration in the setting of sepsis is more likely due to ARDS Choice E= Wedge-shaped areas of hemorrhagic necrosis can be seen with pulmonary embolism, which can cause acute respiratory failure, hypotension, and tachycardia. However, patients often have chest pain, and in general, decompensation occurs suddenly (not progressively over a course of hours). In addition, urinary symptoms, confusion and high fevers are unexpected in this condition

-Age: 52 years woman -With history of cirrhosis due to chronic hepatitis C experiences worsening dyspnea. She has no other medical conditions. Vital signs are within normal limits. Physical examination shows normal jugular venous pressure, right-sided dullness to percussion and decreased breath sounds, normal heart sounds, and moderate-sized ascites -Chest x-ray reveals a large right-sided pleural effusion with no parenchymal lesions. Thoracocentesis yields transudative fluid with normal cell counts. Which of the following is the most likely underlying mechanism of this patient's pulmonary findings? a)Fluid movement across the diaphragm b)Inflammatory disruption of pleural mesothelium c)Malignant obstruction of pleural lymphatic drainage d)Mechanical disruption of thoracic lymphatic flow e)Rapid decrease in intrapleural pressure

Answer: Choice A (Fluid movement across the diaphragm) -This patient with cirrhosis complicated by ascites who presents with a right-sided, transudative pleural effusion most likely has a hepatic hydrothorax. As ascites fluid accumulates in the abdomen, increased intraabdominal pressure can force fluid into the chest cavity through small fenestrations in the diaphragm; fluid movement across the diaphragm occurs most often on the right side -Because the development of ascites in patients with cirrhosis is driven by changes in hydrostatic and oncotic pressure (i.e., increased hydrostatic pressure due to portal venous hypertension and decreased oncotic pressure due to hypoalbuminemia), the pleural fluid in hepatic hydrothorax is transudative by Light criteria Choice B= Inflammatory disruption of the visceral pleural mesothelium can occur in pneumonia, allowing bacteria in the alveoli to translocate into the pleural space and cause a complicated parapneumonic effusion. Parapneumonic effusions and other pleural effusions resulting from an inflammatory disruption of membrane permeability (e.g., rheumatologic effusions, most malignant effusions) are exudative by Light criteria and usually have elevated leukocyte count Choice C= Malignant cells that have metastasized to the pleural space can cause pleural effusion by obstructing pleural fluid drainage via parietal pleura lymphatics. Liver cirrhosis predisposes to the development of hepatocellular carcinoma, which could metastasize to the pleural space; however, malignant effusions typically have high lymphocyte count and cytology frequently reveals malignant cells Choice D= Mechanical disruption of thoracic lymphatic flow (e.g., injury to the thoracic duct) can cause a chylothorax, which is a milky white pleural effusion with very high triglyceride content. Chylothorax is usually exudative by Light criteria Choice E= Atelectasis causes a rapid decrease in intrapleural pressure (as alveoli collapse and pull farther away from the chest wall), and the pressure gradient created between the pleural space and the lung may drive the movement of interstitial fluid into the pleural space, causing a transudative pleural effusion. Pleural effusions due to atelectasis are usually relatively small; hepatic hydrothorax is more likely in this patient with a large, right-sided pleural effusion and a history of cirrhosis

-Age: 46 years man -Previously healthy, develops sudden-onset, sharp, left-sided flank pain radiating to the groin. CT scan of the abdomen reveals a left ureteral calculus, which he passes spontaneously in the urine. Other incidental findings include a left lower lobe lung nodule and foci of calcification in the spleen. Chest imaging shows several small calcified nodules in both lungs and a calcified mediastinal lymph node. The patient has had no pulmonary symptoms and is a lifelong smoker. He is a farmer who lives in Ohio and has not traveled recently -Physical examination shows no abnormalities. Tuberculosis skin test shows a <5mm induration at 48 hours. Repeat chest imaging 3 months later reveals no changes. Which of the following is the most likely cause of the observed findings in this patient? a)Fungal infection b)Metastatic cancer c)Miliary tuberculosis d)Parathyroid adenoma e)Silica exposure

Answer: Choice A (Fungal infection) -The patient has calcified lung, mediastinal, and splenic lesions that are stable over time and asymptomatic, raising suspicion for a contained granulomatous infection. The most likely cause is the dimorphic fungus Histoplasma capsulatum, which exists as a mold in the soil of the Ohio and Mississippi River Valleys. The pathogen is inhaled into the lungs, converts to yeast forms, and is phagocytosed by alveolar macrophages. Macrophages cannot initially eliminate the organism due to microbial virulence factors that prevent phagolysosome formation and acidification. Therefore, H. capsulatum is able to replicate within the macrophage and spread through the draining lymphatic system and (often) into the reticuloendothelial system (e.g., spleen, liver) -After about 2 weeks, patients with intact immunity develop a cell-mediated immune response that contains the infection within granulomas. Over time, the granulomas fibrose and calcify and can be visualized on radiographic imaging at the initial sites of infection (e.g., lungs, hilar and mediastinal lymph nodes, spleen). In healthy patients, most infections with H. capsulatum are asymptomatic and are therefore often discovered incidentally Choice B= Metastatic cancer often causes multiple lung lesions. However, these lesions generally enlarge in size over time and calcification is rare. Choice C= Miliary TB is marked by multiple pulmonary lesions and clinical illness (e.g., fever, night sweats); calcified lesions would be atypical. Although primary TB is often associated with a calcified lung and an ipsilateral hilar lymph node lesion (Ranke complex), TB is unlikely in this case given the patient's negative tuberculin skin test Choice D= Parathyroid adenoma is a benign tumor that does not spread to the lungs. It is often associated with hypercalcemia Choice E= Silica exposure can cause innumerable, small, rounded opacities in the upper lobes of the lungs that may fibrose over time, resulting in pulmonary disease. Silica exposure is primarily occupational (e.g., miners, sandblasters)

-Age: 56 years man -Comes to the office due to chronic cough and fatigue. The patient has smoked 2 packs of cigarettes daily since age 18. Physical examination show cyanosis and expiratory wheezes scattered throughout the lungs. During evaluation for long-term oxygen therapy, his respiratory rate decreases shortly after he begins nasal cannular oxygen supplementation. This patient's reduced respiratory rate is most likely caused by a sudden decrease in stimulation of which of the following sensory receptors? a)Carotid bodies b)Central chemoreceptors c)Juxtaglomerular apparatus d)Pulmonary C fibers e)Pulmonary stretch receptors

Answer: Choice A (carotid bodies) -This patient with COPD developed a decrease in respiratory rate as a result of oxygen supplementation. Oxygen has a minimal effect on respiratory drive unless the arterial partial pressure of oxygen (PaO2) drops below 60-70mmHg; therefore, the arterial pressure of carbon dioxide (PaCO2) is the major stimulator of respiration in healthy individuals. However, patients with long-standing COPD have decreased sensitivity to PaCO2 (due to chronic CO2 retention) and may also have profound hypoxemia (PaO2< 60mmHg); therefore, PaO2 levels can become a significant contributor to respiratory drive. -The depth and rate of respirations are controlled by the medullary respiratory center based on input from central and peripheral chemoreceptors and airway mechanoreceptors. Peripheral chemoreceptors found in the carotid and aortic bodies are the primary sites for sensing PaO2 and are stimulated by hypoxemia. When supplemental oxygen is administered, the rapid increase in PaO2 can reduce peripheral chemoreceptor stimulation and decrease the respiratory rate -Reduced respiratory rate is a minor contributor to the oxygen-induced hypercapnia that can occur in patients with COPD; increased ventilation-perfusion mismatch triggered by alleviation of pulmonary vasoconstriction in poorly ventilated areas is the major mechanism Choice B= Central chemoreceptors, located in the medulla, are more involved in the respiratory response to hypercapnia than to hypoxemia. CO2 readily diffuses through the blood-brain barrier and forms hydrogen ions in the cerebrospinal fluid; the resulting decrease in pH is detected by medullary neurons, triggering an increase in respiration. Because the blood-brain barrier is relatively impermeable to hydrogen ions, blood pH itself has little effect on central chemoreceptors Choice C= The juxtaglomerular apparatus is located in the kidneys and consists of juxtaglomerular cells and the macula densa. It helps regulate renal blood flow and the glomerular filtration rate via renin secretion Choices D and E= Pulmonary stretch receptors include myelinated and unmyelinated C fibers in the lungs and airways. These receptors regulate the duration of inspiration depending on the degree of lung distention (Hering-Breuer reflex)

-Age: 6 years boy -Playing in a dusty field in windy weather inhales many small particles that become lodged in his terminal bronchioles. Which of the following respiratory components is most important in clearing these particles? a)Ciliated cells b)Goblet cells c)Macrophages d)Submucosal glands e)Type I pneumocytes f)Type II pneumocytes

Answer: Choice A (ciliated cells) -Mucociliary clearance is responsible for removing the vast majority of inhaled particles that lodge within the bronchial tree. Ciliated mucosal epithelium lines the pulmonary airways from the trachea to the proximal portions of the respiratory bronchioles. Mucus and fluid secreted onto this epithelial surface act to trap particles suspended in the inspired air. The trapped particles are constantly swept upward from the bronchioles toward the pharynx by cilia that collectively beat in the direction of the pharynx. The mucus and debris are then swallowed or expectorated upon reaching the pharynx -The terminal bronchioles are covered by ciliated cuboidal epithelium and club cells, which help with mucociliary clearance in this region Choice B= Goblet cells are mucus-secreting cells that are interspersed throughout the respiratory mucosa from the trachea down to the larger bronchioles. However, they are not found within the terminal bronchioles. This region is lined by a surfactant-containing fluid secreted by club cells that is continuous with the more proximal mucous layer Choice C= Clearance of particles that lodge distal to the terminal bronchioles is dependent on phagocytosis by alveolar macrophages. These cells can destroy some inhaled particles (e.g., bacteria) by lysosomal degradation. Nondigestible material must be transported by macrophages to the pulmonary lymphatics or to the terminal bronchioles (for clearance by the mucociliary system) Choice D= Submucosal mucous and mucoserous glands help form the mucous layer in the larger airways. They are found within the trachea and bronchi but not in the bronchioles Choices E and F= Type I and type II pneumocytes line the alveolar ducts and alveoli. Type I pneumocytes are thin cells stretched across the alveolar wall that mediate gas exchange. Type II pneumocytes produce surfactant and divide and differentiate into type I pneumocytes to replace damaged cells

-Age: 18 years man -Evaluated for recurrent episodes of shortness of breath, wheezing, runny nose, and watery eyes. The patient reports that his symptoms have recently worsened since he started training outdoors for an upcoming marathon. He does not use tobacco, alcohol or illicit drugs. -Skin testing is performed to determine symptom triggers. A pricking device is used to apply allergen extract underneath the skin. After 15 mins, the patient develops raised, erythematous plaques with surrounding erythema at the application site. Which of the following mediators is the first to be released during the pathogenesis of this patient's skin findings? a)Histamine b)Leukotriene D4 c)Major basic proteins d)Platelet-activating factor e)Prostaglandin D2

Answer: Choice A (histamine) -This patient's shortness of breath and wheezing are suggestive of asthma, and his rhinorrhea and watery eyes are suggestive of allergic rhinitis. Both of these conditions are examples of type I hypersensitivity, which involves the triggering of an allergic response via the binding of previously recognized antigen to IgE antibodies on mast cells. Many type I hypersensitivity reactions are composed of both an early and late phase Histamine is housed in preformed granules of unactivated mast cells and plays an important role in the early phase of type I hypersensitivity. Upon activation, mast cells rapidly release histamineviadegranulation, making histamine the first chemical mediator to take effect. Once released, histamine triggers smooth muscle contraction leading to bronchoconstriction, increases vascular permeability leading to edema, and increases Choice B= Leukotriene D4, along with the other cysteinyl leukotrienes (C4 and E4) are not preformed but require synthesis via the 5-lipoxygenase pathway of arachidonic acid metabolism. Therefore, these mediators are released by mast cells later than histamine. Once released, the cysteinyl leukotrienes are potent mediators of vasoconstriction, bronchoconstriction, and increased vascular permeability Choice C= Major basic protein is released from eosinophils in the late stage of a type I hypersensitivity reaction and causes localized tissue damage Choice D= Platelet-activating factor is a secondary inflammatory mediator that must be synthesized from phospholipid prior to being released from activated mast cells and basophils. Once released, it stimulates bronchospasm and increased vascular permeability Choice E= Prostaglandin D2 is synthesized via the cyclooxygenase pathway of arachidonic acid metabolism prior to being released from mast cells. Once released, it causes bronchospasm and vasodilation

-Researchers are studying the structure and function of hemoglobin. Arterial and venous blood samples are obtained from enrolled volunteers with no health problems. Red blood cells are isolated from whole blood by differential centrifugation, and the contents of the intact red blood cells are analyzed. It is found that the concentration of chloride is much lower in the red blood cells in the arterial sample compared to the venous sample. The activity of which of the following enzymes is most likely responsible for the observed finding? a)Bisphophoglycerate mutase b)Carbonic anhydrase c)Glucose-6-phosphate dehydrogenase d)Na+-K+ ATPase e)Pyruvate kinase

Answer: Choice B (Carbonic anhydrase) Hemoglobin is only found within red blood cells (RBCs), and it is responsible not only for O2 delivery to the tissues, but also for carrying CO2 from the tissues to the lungs. Hemoglobin carries CO2 in the form of carbaminohemoglobin, which is created as follows: -CO2 + Hb-NH3= 2H+ + Hb-NH-CO2 However, only a small percentage of total blood CO2 is carried by hemoglobin. The majority of total blood CO2 is carried in the plasma as bicarbonate ion (HCO3-) via the following process: -The CO2 produced by tissue metabolism enters RBCs and is hydrated by the enzyme carbonic anhydrase to form carbonic acid (H2CO3) -H2CO3 then undergoes spontaneous conversion to HCO3- and H+ -Then excess HCO3- is then transferred out of RBCs into the plasma via band 3 protein in exchange for chloride ions (Cl-) to maintain electrical neutrality. This exchange is known as "chloride shift" and is the principal cause of high RBC chloride content in venous blood Choice A= Bisphophoglycerate (BPG) mutase converts 1,3-BPG to 2,3-BPG. In RBCs, 2,3-BPG combines with hemoglobin to decrease its affinity for O2 and facilitate O2 unloading in the tissues Choices C and E= Glucose-6-phosphate dehydrogenase is the first enzyme in the pentose phosphate pathway (produces NADPH) and pyruvate kinase catalyzes the final step of glycolysis (produces ATP). Deficiency of either glucose-6-phosphate dehydrogenase or pyruvate kinase typically leads to shortened RBC lifespan and episodic hemolysis triggered by oxidative stressors Choice D= Na+/K+ ATPase is responsible for the maintenance of ionic concentration gradients across the plasma membrane in many cells in the body. This energy-requiring pump is electrogenic as it extrudes 3 sodium ions for every 2 potassium ions that enter the cell, which creates negative intracellular potential

-Age: 63 years man -Comes to the ED due to a 1-week history of progressively worsening shortness of breath and cough. The shortness of breath is exacerbated when the patient lies flat in bed, he has to prop himself up with 3 pillows to sleep well at night -He was treated for a myocardial infarction 2 weeks ago and had 2 stents placed in the left anterior descending artery. The patient's other medical conditions include long-standing hypertension and type 2 diabetes. BP (120/80), pulse (92), RR (22). Pulse oximetry is 89% on room air. -Physical examination reveals bilateral crackles at the lung bases and an S3. Which of the following is most likely present in this patient? a)Decreased alveolar surface tension b)Decreased lung compliance c)Increased dead space ventilation d)Increased functional residual capacity e)Left-to-right cardiac shunting

Answer: Choice B (Decreased lung compliance) -This patient's dyspnea, orthopnea, bibasilar crackles, and S3 suggest left-sided heart failure due to recent MI. Atherosclerotic plaque rupture in the left anterior descending or left circumflex artery can infarct large areas of the LV myocardium, leading to decreased contractility and reduced cardiac output. This can result in elevation of end-diastolic pressure that impairs diastolic return from the pulmonary veins, leading to increased hydrostatic pressure in the pulmonary capillaries with transudation of fluid into the lung parenchyma (pulmonary edema) -Pulmonary edema in the alveoli impairs ventilation and causes intrapulmonary shunting. In addition, edema in the lung interstitium makes the lungs heavy and stiff, restricting pulmonary expansion during inspiration. The decreased lung compliance can mimic that seen with other causes of restrictive lung physiology (e.g., interstitial lung disease) Choice A= Surfactant decreases the surface tension of the fluid layer that lines the alveolar cells, helping prevent alveolar collapse. Pulmonary edema dilutes pulmonary surfactant, increasing both alveolar surface tension and the tendency for alveolar collapse Choice C= Ventilation-perfusion (V/Q) mismatch exists on a spectrum with intrapulmonary shunting (perfusion without ventilation) and dead space ventilation (ventilation without perfusion) on the other end. Pulmonary edema causes V/Q mismatch via increased intrapulmonary shunting with a corresponding decrease in dead space ventilation. V/Q mismatch via increased dead space ventilation is seen with pulmonary embolism Choice D= Functional residual capacity is the volume of air in the lungs at the end of normal expiration. It is increased in conditions with increased lung compliance (e.g., COPD) and decreased in conditions such as pulmonary edema that reduce lung compliance Choice E= Interventricular septal rupture is a mechanical complication of MI that causes left-to-right cardiac shunting. However, this complication usually occurs within 3-5 days following MI and precipitates a dramatic and acute, rather than gradual, presentation of heart failure

-Age: 7 years girl -Brought to the ED due to 2 days of fever, productive cough, and shortness of breath. Medical history is significant for recurrent lung infections and cystic fibrosis. Weight is at the 5th percentile. Temp. (38.1C), pulse (104), RR (30). Pulse oximetry is 86% on room air. -Examination shows a thin girl with digital clubbing and a hyperexpanded chest. Diffuse wheezes and crackles are present bilaterally. The rest of the examination is unremarkable. -Chest x-ray reveals hyperinflation, chronic interstitial changes, and an infiltrate in the right lower lobe. Gram stain of sputum shows gram-negative rods. Which of the following is the most likely cause of this patient's recurrent infections? a)Deficient natural killer cell formation b)Development of bacterial macrocolonies c)Dysfunctional free radical production by neutrophils d)Inability to form the membrane attack complex e)Reduced endobronchial proteases activity

Answer: Choice B (Development of bacterial macrocolonies) -Cystic fibrosis (CF) is a multisystem disorder in which a defective chloride channel (CFTR) prevents the normal hydration of mucus. Accumulation of thick, viscous secretions in the lungs leads to mucus plugging and progressive bronchiectasis; which manifest as obstructive lung disease (e.g., hyperinflated lungs, barrel chest) and chronic hypoxia (e.g., digital clubbing). GI involvement can also lead to malabsorption with decreased body weight -The abnormal airway secretions in patients with CF impair mucociliary clearance and allow colonization of bacteria such as H. influenzae, S. aureus, and over time, gram-negative rods such as Pseudomonas aeruginosa and Burkholderia cepacia complex -The microenvironment is particularly suitable to mucoid P. aeruginosa because localized hypoxia within the airway mucus causes the bacterium to lose motility and produce alginate, a polysaccharide involved in biofilm formation. The biofilm acts as a protective matrix for the development of Pseudomonas macrocolonies, which are difficult to eradicate and cause recurrent and persistent infection Choice A= Natural killer cell deficiency is associated with severe, disseminated viral infections (e.g., herpes simplex virus, cytomegalovirus). This patient's infiltrate in the right lower lobe and gram stain findings are consistent with bacterial pneumonia, and she has no history of severe or recurrent viral infection Choice C= Neutrophil free radical formation is impaired in chronic granulomatous disease, which is characterized by recurrent multiorgan (e.g., lungs, liver, lymph nodes, skin) infections by catalase-positive organisms (e.g., Aspergillus, S. aureus). In contrast this patient has isolated, chronic pulmonary findings. Moreover, CF results in excessive neutrophil free radical formation due to persistent pulmonary infection, which contributes to the progressive lung damage seen in these patients Choice D= Inability form the membrane attack complex occurs with terminal complement deficiency. These patients have recurrent Neisseria meningitis, not lung infections Choice E= Bacterial colonization in the lungs of patients with CF induces a massive infiltration of neutrophils, which results in the release of elastase, a protease that promotes bronchiectasis. Therefore, chronic lung disease in CF is characterized by excessive, not reduced, endobronchial protease activity

-Age: 32 years hospitalized man -Evaluated for new-onset shortness of breath and confusion. The patient was admitted 3 days ago following a motor vehicle collision during which he sustained bilateral femoral fractures, a pelvic fracture, and urethral injury. -He underwent surgical repair with no operative complications. Temp. (37.9C), pulse (110), RR (24), oxygen saturation is 84% on room air. On physical examination, the patient appears agitated and disoriented. The lungs are clear to auscultation and there is no lower extremity edema. A petechial rash is present on the chest. ECG shows sinus tachycardia with no ischemic changes. Which of the following histologic changes has most likely taken place in this patient's lung tissue? a)Alveolar fluid rich in neutrophils b)Fat microglobules in pulmonary arterioles c)Prominent interstitial infiltration by mononuclear cells d)Red thrombus lodged in the pulmonary artery e)Scattered necrosis with alveolar hemorrhage

Answer: Choice B (Fat microglobules in pulmonary arterioles) -The clinical triad of acute-onset neurologic abnormalities, hypoxemia, and petechiae in a patient with traumatic bone fracture is strongly suggestive of fat embolism syndrome (FES). This condition most commonly occurs 24-72 hours following long-bone and/or pelvic fracture. Pathophysiologically, the traumatic event dislodges fat globules from the bone marrow and allows them to enter disrupted marrow venules, where they can then traverse the systemic veins and deposit in pulmonary microvessels. The pulmonary capillary occlusion impairs gas exchange and induces hypoxemia, release of free fatty acids from the fat globules also causes local toxic injury to the endothelium, potentially leading to acute respiratory distress syndrome -Some fat globules escape the lungs via precapillary arteriovenous shunts that open due to increased pulmonary artery pressure. This phenomenon appears to be responsible for the fat emboli-associated microvascular occlusion that can occur within the CNS, manifesting with confusion and neurologic impairment, and in the dermal capillaries, resulting in erythrocyte extravasation and a petechial rash Choices A, C and D= A neutrophil-rich alveolar exudate is not an early histologic manifestation of FES but rather of acute bacterial or aspiration pneumonia. Mononuclear interstitial pulmonary infiltrates are found early in the course of a variety of interstitial lung diseases. A red thrombus lodged in the pulmonary artery is consistent with a thromboembolism from a deep venous source. None of these conditions would typically be associated with neurologic impairment or petechial rash seen with FES Choice E= Focal necrosis of alveolar walls with associated intra-alveolar hemorrhage is often seen with pulmonary hemorrhage syndromes (e.g., anti-glomerular basement membrane antibody disease, granulomatosis with polyangiitis)

-Age: 65 years man -Evaluated for hypotension and increasing shortness of breath shortly after placement of a right-sided subclavian central venous catheter. The patient was recently diagnosed with colon cancer without metastasis and is scheduled for surgical resection -BP (80/50), pulse (110), RR (24). Examination shows jugular venous distention. Breath sounds are decreased on the right and the trachea is deviated to the left. -Which of the following is the most likely cause of this patient's hypotension? a)Decreased intravascular volume b)Decreased systemic vascular resistance c)Decreased venous return d)Impaired baroreceptor reflex e)Increased intrapericardial pressure

Answer: Choice C (Decreased venous return) -This patient with shortness of breath, hypotension, unilaterally decreased breath sounds, and tracheal deviation shortly after subclavian catheter placement likely has a tension pneumothorax. Because of the close proximity of the subclavian vein to the apex of the lung, there is a risk of lung puncture during the procedure -Lung puncture allows air to pass into the pleural space. If the puncture is small, the patient may remain asymptomatic. With a large puncture, air rushes into the pleural space, leading to pressure equalization and loss of intrapleural negative pressure, which causes shortness of breath due to inability to expand the affected lung. Intrapleural and lung pressures usually remain nearly equivalent (i.e., simple pneumothorax), and symptoms are limited to chest pain and respiratory difficulty -Sometimes lung puncture results in the formation of a one-way tissue valve that opens during inspiration to allow air into the pleural space and closes during expiration to trap that air, causing intrapleural pressure to progressively increase with each breath (i.e., tension pathophysiology). The increasing pressure leads to contralateral mediastinal shifting (e.g., tracheal deviation) and vena cava collapse, resulting in decreased venous return to the heart. Cardiac output is reduced, leading to hypotension and tachycardia (i.e., obstructive shock). Treatment requires emergency decompression of the pleural space with needle insertion or chest-tube placement to prevent cardiac arrest Choices A and B= Decreased intravascular volume is the cause of hypovolemic shock (e.g., due to massive hemorrhage) and decreased systemic vascular resistance is the primary disturbance in distributive shock (e.g., due to sepsis). The jugular veins are flat in these conditions, and unilaterally decreased breath sounds, and tracheal deviation are not expected Choice D= An impaired baroreceptor reflex can cause or worsen hypotension due to failure of reflex-mediated vasoconstriction and increased heart rate. This patient's tachycardia suggests that the baroreceptor reflex is intact Choice E= Increased intrapericardial pressure occurs in cardiac tamponade. The increased pressure impaired diastolic filling of the RV, leading to obstructive shock. Although jugular venous distention is expected, cardiac tamponade does not explain this patient's unilaterally decreased breath sounds and tracheal deviation

-Age: 66 years man -With poorly controlled type 2 diabetes is admitted to the hospital due to a 2-day history of fever and confusion. BP (110/50). Pulse oximetry shows an oxygen saturation of 97% on room air. Examination reveals warm extremities with full peripheral pulses, an infected neuropathic foot ulcer with surrounding cellulitis, and normal lung sounds -A central venous catheter terminating in the superior vena cava is placed. Blood aspirated from the catheter appears red. Blood gas analysis of this venous sample reveals an oxygen saturation of 87% (normal: 65%-70%) and lactate is moderately elevated. Which of the following is the most likely cause of this patient's abnormal blood gas findings? a)Decreased cardiac output to the vital organs b)Impaired mitochondrial oxidative respiration in the vital organs c)Large-vessel thrombosis throughout the vital organs d)Poor uptake of oxygen in the lungs e)Stronger oxygen-hemoglobin binding in peripheral tissues

Answer: Choice B (Impaired mitochondrial oxidative respiration in the vital organs) This patient has sepsis due to infected diabetic foot ulcer with surrounding cellulitis. Sepsis is a florid host inflammatory response to infection that can lead to multiple organ system dysfunction Septic organ dysfunction is driven mainly by poor tissue oxygen use. This cellular dysoxia is caused by 3 major mechanisms: -Bacterial components (e.g., endotoxin) and acute phase cytokines (e.g., IL-1-beta) trigger production of free radicals that damage mitochondria and interfere with the electron transport chain. Immediate postmortem analysis of patients with sepsis reveals surprisingly minimal tissue necrosis but, often, extensive mitochondrial damage. This mitochondrial dysfunction leads to decreased oxidative phosphorylation with loss of ATP production. A compensatory bioenergetic shift toward glycolysis often results in lactic acidosis -Widespread microcirculatory failure with vasodilation causes blood to shunt rapidly through organs, decreasing the opportunity for oxygen extraction -Increased capillary permeability causes tissue edema (third spacing), which increases the diffusion distance for oxygen to reach mitochondria of target cells Because oxygen use decreases, the transorgan arteriovenous oxygen gradient is small; therefore; oxygen saturation of central venous blood (SvcO2) (SVC)- returning from the periphery- increases. Central venous catheters are often placed therapeutically to deliver medications; they can be used diagnostically to differentiate septic (Increased SvcO2) from cardiogenic (decreased SvcO2) shock Choice A= Cardiac output (oxygen delivery) is typically increased in sepsis due to systemic vasodilation (warm extremities, wide pulse pressure). Decreased cardiac output is the hallmark of severe heart failure (cold extremities, narrow pulse pressure), and SvcO2 decreases with heart failure because peripheral oxygen consumption outstrips its delivery Choice C= Sepsis is associated with microvascular (rather than macrovascular) thrombosis due to endothelial injury and focal or disseminated intravascular coagulation, further impairing oxygenation. Widespread large-vessel thrombosis and infarction are seen in conditions such as catastrophic antiphospholipid-antibody syndrome Choice D= Sepsis can induce capillary permeability in the lung, leading to pulmonary edema with resultant arterial hypoxemia (ARDS). However, this patient's pulmonary function and oxygenation are intact, as evidenced by normal lung examination (no crackles) and arterial oxygen saturation Choice E= Hemoglobin oxygen unloading to peripheral tissues is facilitated by sepsis-induced (lactic acidosis) and fever, which lead to rightward shifting of the oxyhemoglobin dissociation curve (i.e., weaker oxygen-hemoglobin binding)

-Age: 62 years woman -Brought to the ED due to progressive weakness and dyspnea. The patient is visiting her niece, who says the patient forgot to bring her regular medications and has not been taking them for the past week. The patient began feeling weak and fatigued 3 days ago and has had shortness of breath since yesterday -Temp. (37C), BP (122/88), pulse (90), RR (24, shallow). On physical examination, the patient appears dyspneic with drooping at the eyelids and corners of the mouth. Her speech has a nasal quality. Arterial blood gas obtained on room air shows pH 7.32, PaCO2 is 52mmHg, PaO2 is 72mmHg, and HCO3 is 26mEq/L. -Bedside spirometry demonstrates decreased forced vital capacity. A pathologic process involving which of the following structures is the most likely cause of this patient's shortness of breath? a)Brain stem respiratory center b)Muscles of respiration c)Pulmonary arteries d)Terminal airways and alveoli e)Upper larger airways

Answer: Choice B (Muscles of respiration) -This patient has hypercapnic and hypoxic respiratory failure (low pH, high CO2 and low O2) indicating global hypoventilation. In association with ptosis, bulbar weakness, and low forced vital capacity, this presentation suggests myasthenic crisis (severe weakness and respiratory depression due to an exacerbation of myasthenia gravis (MG)) -MG is characterized by autoantibodies against nicotinic acetylcholine receptors on the postsynaptic membrane of the neuromuscular junction, resulting in receptor degradation. This reduces the sensitivity of the postsynaptic membrane to acetylcholine stimulation, leading to reduced muscular response despite normal acetylcholine release. Muscle weakness worsens with repetition as acetylcholine stores within the presynaptic nerve terminal become progressively depleted -Patients with MG typically have extraocular (e.g., ptosis, diplopia), bulbar (e.g., dysphonia, difficulty chewing), and facial (e.g., myasthenic snarl) weakness. In addition, neck and proximal muscle weakness may occur, and in severe cases the respiratory muscles may be affected, leading to respiratory failure (as in this patient). Acetylcholinesterase inhibitors (e.g., pyridostigmine, neostigmine) are used for symptomatic treatment, and withdrawal can trigger a myasthenic crisis Choice A= Impairment of the respiratory control centers in the brain stem causes hypoventilation during sleep (Ondine curse); however, voluntary breathing is unaffected Choice C= Pulmonary embolism can result in dyspnea and gas exchange abnormalities (e.g., hypoxia). However, significant muscular weakness (e.g., ptosis, nasal speech, decreased vital capacity) would be unexpected Choices D and E= Patients with diseases involving the terminal airways and alveoli (e.g., pulmonary fibrosis, emphysema) and large airways (e.g., asthma, chronic bronchitis) can present with respiratory failure subsequent to muscle fatigue. However, these diseases are not associated with non-respiratory muscle weakness (e.g., dysphonia, bulbar weakness)

-Age: 68 years man -Comes to the ED due to cough, breathlessness, and chest pain. The patient has a medical history of hypertension and type 2 diabetes. He is a former smoker with a 20-pack-year history -Temp. (38C), BP (130/80), RR (22), oxygen saturation is 95% while breathing ambient air. The trachea is central in position. There is dullness to percussion over the lower right lung area compared to the resonance on the left. -Auscultation of the right lower lobe reveals breath sounds that are loud, hollow, and high-pitched. When the patient recites the word "ninety-nine" in a normal voice, the sound has higher clarity and intensity over the right base compared to other lung areas. Which of the following is the most likely cause of these findings? a)Accumulation of interstitial fluid within the pleural space b)Embolic occlusion of a pulmonary segmental artery c)Exudation of neutrophil-rich fluid in the lung parenchyma d)Leakage of air from spontaneous rupture of visceral pleura e)Lung collapse due to a mass causing bronchial obstruction f)Transudation of interstitial fluid within the alveolar spaces

Answer: Choice C (Exudation of neutrophil-rich fluid in the lung parenchyma) -This patient's presentation is consistent with bacterial pneumonia. The infection causes the alveoli to fill with inflammatory, neutrophil-rich exudate (pus); alveolar filling with any kind of fluid (e.g., pus, edema, blood) creates alveolar consolidation -Alveolar consolidation leads to a number of characteristic physical examination findings. Sound travels faster and more efficiently through liquids than gases; therefore, fluid-filled alveoli transmit higher intensity sound than those filled with air. As a result, breath sounds and tactile fremitus (vibration) are more prominent over areas of alveolar consolidation. Bronchophony, a phenomenon in which a patient's spoken words (e.g., "ninety-nine", "toy boat") are heard more loudly and clearly when auscultated over an area of consolidation compared to normal lung, is also present. The higher density of fluid-filled alveoli also creates relative dullness to percussion compared to the resonant sound created on percussion of air-filled alveoli Choice A= Accumulation of interstitial fluid within the pleural space (pleural effusion) insulates breath sounds and vibrations that originate in the airways and lungs. Consequently, tactile fremitus and breath sound intensity are decreased. The fluid in the chest cavity creates dullness to percussion over the affected area Choice B= Pulmonary embolism typically has minimal effect on physical examination findings for the lungs. Choice D= As in pleural effusion, accumulation of air in the pleural space (pneumothorax) insulates sounds and vibrations that originate in the lungs, and breath sound intensity and tactile fremitus are decreased. In contrast to pleural effusion, the relatively low density of air in the pleural space creates hyperresonance to percussion Choice E= A mass in a proximal bronchial airway (e.g., malignancy, mucus plugs) can cause airway obstruction and consequent collapse (atelectasis) of distal alveoli. Breath sounds and tactile are decreased, and dullness to percussion is created due to absence of air Choice F= Transudation of interstitial fluid into the alveoli occurs in heart failure and can create alveolar consolidation with physical examination findings similar to bacterial pneumonia. However, in heart failure the findings are typically similar bilaterally. Pneumonia, with accumulation of inflammatory exudate, is more likely in this patient with low-grade fever and evidence of unilateral alveolar consolidation

-Age: 22 years Caucasian male -Has recurrent pulmonary infections and finger clubbing, the pancreas shows extensive exocrine gland atrophy and fibrosis. The pancreatic ducts are lined with squamous epithelium with areas of keratinization. The finding of squamous metaplasia in the pancreatic ducts is most likely related to a deficiency of a)Riboflavin b)Vitamin A c)Vitamin D d)Vitamin E e)Vitamin K f)Vitamin C

Answer: Choice B (Vitamin A) -Recurrent sinopulmonary infections and exocrine gland fibrotic atrophy in a young Caucasian are suggestive of cystic fibrosis (CF). In the pancreas, severe CF may cause total obstruction followed by complete fibrotic atrophy of the exocrine glands. The resulting pancreatic insufficiency can cause a deficiency of fat-soluble vitamins. -Avitaminosis A in particular may contribute to squamous metaplasia of the epithelial lining of pancreatic exocrine ducts, which are already injured and predisposed to squamous metaplasia by inspissated mucus. Normal levels of vitamin A and its metabolite, retinoic acid, are required to maintain orderly differentiation of specialized epithelia, including mucus-secreting columnar epithelium. When a deficiency state exists, the epithelium undergoes squamous metaplasia to a keratinizing epithelium Choices A and F= Riboflavin (vitamin B2) and vitamin C are water-soluble vitamins whose intestinal absorption is not affected by fat malabsorption Choice C= Vitamin D is a fat-soluble vitamin. However, vitamin D deficiency would not cause epithelial squamous metaplasia. Instead, it could produce rickets in children and osteomalacia in adults Choice D= Vitamin E is a fat-soluble vitamin. Deficiency in this vitamin could cause infertility and decreases in some serum phospholipids Choice E= Vitamin K is a fat-soluble vitamin. A vitamin K deficiency could cause a coagulopathy (bleeding diathesis) secondary to inadequate vitamin K dependent clotting factors

-Age: 55 years man -Comes to the ED due to sudden-onset dyspnea. Medical history is significant for hypertension, hyperlipidemia, type 2 diabetes, and chronic kidney disease. The patient takes multiple medications and has no drug allergies. -He works for an international bank and returned from a business trip in Australia a day ago. BP (110/70), pulse (110). Physical examination shows a moderately overweight man with tachypnea. The lungs are clear on auscultation. ECG shows sinus tachycardia. Ventilation/perfusion scanning is ordered. Which of the following findings would help confirm the suspected diagnosis in this patient? a)Absence of ventilation and perfusion abnormalities b)An area showing both ventilation and perfusion defects c)A perfusion defect without an associated ventilation defect d)A ventilation defect without an associated perfusion defect e)Several small areas of matched perfusion and ventilation defects

Answer: Choice C (A perfusion defect without an associated ventilation defect) -Pulmonary embolism (PE) should be suspected in this patient with recent extended travel (prolonged immobilization) who now has acute-onset dyspnea and tachypnea, normal lung examination, and sinus tachycardia on ECG. In most patients, CT angiography, which requires IV contrast administration, is the diagnostic test of choice. However, the contrast used in CT studies should be avoided in patients with chronic kidney disease due to increased risk of contrast-induced nephropathy; in such patients, a ventilation/perfusion (V/Q) scan is the preferred diagnostic study -A V/Q scan is a nuclear medicine study that uses a radioactive tracer placed in both inhaled gas and the bloodstream to visualize areas of ventilation and perfusion in the lung. In patients with PE, ventilation is preserved but the embolus prevents adequate blood perfusion to the affected areas of the lung. Therefore, the most definitive diagnostic testing for PE on V/Q scan is a large area showing a perfusion defect without ventilation defect (mismatched perfusion defect) Choice A= The absence of perfusion abnormalities= likely rules out significant PE Choices B and E= Matched ventilation and perfusion defects may result from chronic lung abnormalities (e.g., airway inflammation or obstruction, atelectasis) that lead to localized areas of poor ventilation with corresponding hypoxic vasoconstriction, or they may represent a PE that happens tot be in an area of poor ventilation. These results are often inconclusive and necessitate additional testing to confirm or rule out PE Choice D= An area of ventilation defect without perfusion defect is suggestive of conditions that lead to acute alveolar filling (e.g., pneumonia, pulmonary edema)

-A research study is conducted to evaluate the effects of various serum proteins in health and disease states. Healthy volunteers and patients with various medical conditions are recruited -In an experiment, purified insoluble elastin is exposed to stimulated neutrophils, which results in the degradation of the elastin molecules into smaller peptides. This process can be inhibited by adding serum from healthy volunteers but not from a specific patient population -Which of the following is the strongest recommendation for patients in this specific population? a)Avoidance of competitive sports participation b)Avoidance of immunosuppressive agents c)Avoidance of second-hand smoke d)Periodic aortic diameter monitoring e)Regular bone density measurement f)Regular ophthalmologic examinations

Answer: Choice C (Avoidance of second-hand smoke) -Neutrophil elastase is released by both neutrophils and macrophages and is the primary protease responsible for extracellular elastin degradation. The major serum inhibitor of elastase is alpha-1 antitrypsin (AAT). In this study, the patients whose serum is unable to inhibit elastin degradation likely have AAT deficiency, a condition characterized by early-onset panacinar emphysema due to the unopposed action of neutrophil elastase on alveolar cells -Tobacco exposure dramatically accelerates the development of emphysema in patients with AAT deficiency by inducing inflammation (increasing neutrophil and macrophage activation) and permanently inactivating the already low quantities of AAT through oxidation of a crucial methionine residue. Therefore, patients with AAT deficiency should be counseled to avoid smoking and exposure to second-hand smoke Choice A= Patients with hypertrophic cardiomyopathy are at risk of sudden cardiac arrest with exertion and are typically counseled to avoid participation in most competitive sports Choice B= Patients with chronic granulomatous disease (impaired phagocytic intracellular killing due to defect in nicotinamide adenine dinucleotide phosphate (NADPH) oxidase) should avoid certain immunosuppressive agents (e.g., tumor necrosis factor-alpha blockers) due to risk of overwhelming infection Choice D= Periodic aortic diameter monitoring is indicated in patients with Marfan syndrome (impaired fibrillin synthesis), who are at risk for both aortic dissection and chronic aortic regurgitation due to aortic root dilation Choice E= Patients with osteogenesis imperfecta (decreased production of type 1 collagen) are at high risk of developing osteoporosis and should undergo regular bone density monitoring Choice F= Patients with Ehlers-Danlos syndrome (impaired collagen synthesis) are at risk for scleral fragility, ocular globe rupture, and retinal detachment and should undergo routine ophthalmologic exams

-Age: 8 years girl -Brought to the ED due to worsening shortness of breath and chest tightness for the past 2 hours. On physical examination, the patient speaks in short sentences. Lung auscultation reveals diffuse wheezing. During the evaluation, the patient is instructed to take a deep breath and blow as hard as possible into the mouthpiece of a hand-held device. -The measured airflow rate is significantly lower than expected. Treatment with an inhaled medication is begun. A repeat evaluation several minutes later shows an increase in the airflow rate. The administered medication most likely affected which of the following to produce the observed finding in this patient? a)Airway inflammatory response b)Alveolar surfactant production c)Bronchiolar smooth muscle tone d)Lower-airway bacterial load e)Pulmonary vascular resistance

Answer: Choice C (Bronchiolar smooth muscle tone) -This patient's acute-onset shortness of breath and chest tightness are consistent with the airway spasm and bronchoconstriction that occurs in asthma. She received a bronchodilator to reduce airway resistance and increase airway flow. In asthma, bronchodilation is typically accomplished with a beta-2 receptor agonist (e.g., albuterol); these drugs work by stimulating beta-2 adrenergic receptors on bronchial smooth muscle cells, leading to an increase in cyclic AMP and consequent smooth muscle relaxation (decrease in smooth muscle tone) Choices A and B= Corticosteroids, both inhaled (e.g., budesonide, fluticasone) and systemic (e.g., prednisone), modify gene transcription to reduce the airway inflammatory response in patients with asthma. Corticosteroids are also used to stimulate alveolar surfactant production and reduce the risk of neonatal respiratory distress syndrome in premature infants Choice D= Inhaled antibiotics (e.g., tobramycin) are used to reduce lower-airway bacterial load in patients with cystic fibrosis Choice E= Several medication, including endothelin receptor antagonists (e.g., bosentan), phosphodiesterase inhibitors (e.g., sildenafil), and prostacyclin analogs, are used to lower pulmonary vascular resistance in patients with pulmonary arterial hypertension

-Age: 42 years woman -Comes to the office with a nonproductive cough and worsening shortness of breath with exertion. The patient has a history of primary pulmonary hypertension and underwent lung transplantation 8 months ago -She states that she has not missed any doses of her transplant medication. A chest x-ray reveals surgical evidence of her transplant but clear lung fields. Pulmonary function testing demonstrates a forced expiratory volume in 1 second (FEV1), 67% of her best posttransplant FEV1. Her forced vital capacity remains largely unchanged. A lung biopsy shows areas of total fibrotic obstruction the terminal bronchioles. This patient's condition is most likely caused by which of the following? a)Acute transplant rejection b)Chronic obstructive pulmonary disease c)Chronic transplant rejection d)Ischemia-reperfusion injury e)Opportunistic infection f)Recurrence of pulmonary hypertension

Answer: Choice C (Chronic transplant rejection) -The patient with cough, dyspnea, obstructive findings on spirometry, and fibrotic destruction of the small airways likely has chronic transplant rejection. Lung transplantation may be complicated by both acute and chronic rejection. Chronic rejection is a major cause of morbidity and mortality and occurs in almost half of all patients within 5 years of transplantation. These patients commonly present with dyspnea and a dry cough. Spirometry shows an airflow limitation with a drop in both FEV1 and FEV1/FVC ratio -Chronic rejection affects the small bronchioli producing the obstructive lung disease known as bronchiolitis obliterans. Initially, histopathology shows lymphocytic inflammation and destruction of the epithelium of the small airways. Subsequently, fibrinopurulent exudate and granulation tissue are found in the lumen of bronchioli, which ultimately results in fibrosis, scarring, and the progressive obliteration of small airways Choice A= Acute rejection usually occurs within the first 6 months. It is frequently asymptomatic and commonly discovered on surveillance biopsies. Histopathology demonstrates perivascular and interstitial mononuclear cell infiltrates Choices B and F= Underlying lung disease, such as pulmonary hypertension or COPD, may recur in the lung allograft. However, the obstructive pattern on spirometry and the histopathology findings are better explained by chronic rejection than by pulmonary hypertension or COPD, the latter of which would also show increased goblet cells and mucus gland hyperplasia Choice D= Ischemia-reperfusion injury is a form of noncardiogenic pulmonary edema that occurs in the allograft due to surgical trauma or organ ischemia. It is seen during the first few days following transplantation Choice E= Infection is a leading cause of death in lung transplant recipients. Cytomegalovirus causes the most significant opportunistic infection in transplant recipients. Infected cells have a characteristic intranuclear inclusion surrounded by a clear halo (owl's eye inclusion)

-Age: 35 years man -Comes to the clinic for evaluation of exertional dyspnea and dry cough for 8 months. Oxygen saturation on room air while lying down is 96% and drops to 88% after 6 mins of walking. -Examination show fine crackles in both lower lungs. Chest x-ray reveals reticular densities in both lung fields. Lung biopsy shows an infiltration of inflammatory cells, predominantly lymphocytes, as well as poorly formed noncaseating granulomas and moderate alveolar septal fibrosis. Which of the following is the most likely diagnosis? a)Bronchial asthma b)Chronic bronchitis c)Hypersensitivity pneumonitis d)Idiopathic pulmonary fibrosis e)Pulmonary tuberculosis

Answer: Choice C (Hypersensitivity pneumonitis) This patient with chronic dyspnea and cough accompanied by characteristic lung biopsy findings most likely has hypersensitivity pneumonitis (HP). HP is an exaggerated immunologic response to an inhaled antigen; common offending agents include mold, bacteria, animal protein, and chemicals. The condition has variable presentation that is largely dependent on the dose and chronicity of antigen exposure -Acute HP involves abrupt-onset and often recurring episodes of fever, chills, cough, dyspnea, and fatigue that coincide with intermittent high-dose antigen exposure. Leukocytosis is often present. Chest x-ray may be normal or show scattered micronodular opacification -Chronic HP involves persistent and gradually progressive symptoms (as in this patient) resulting from long-term, moderate-dose antigen exposure. Patients usually have symptoms of cough, dyspnea, fatigue, and weight loss for several months or more. Lung auscultation usually reveals fine crackles, and chest x-ray shows interstitial reticular opacities consistent with the development of pulmonary fibrosis The diagnosis of HP can be aided by pulmonary function testing that reveals a restrictive pattern; the restriction is worse with chronic HP and is accompanied by more severe impairment in gas exchange (e.g., hypoxemia, low DLCO). Biopsy in both acute and chronic disease reveals lymphocytic infiltrate with poorly formed noncaseating granulomas created by the walling off of inhaled antigens. Chronic HP is associated with progressive alveolar septal fibrosis. Removal of antigen exposure usually resolves acute disease; however, the fibrosis occurring in chronic disease is often irreversible Choice A= Asthma presents with intermittent cough and dyspnea. Bronchiolar biopsy can reveal noncaseating granulomas, but fibrosis is not typical and eosinophilic infiltrate is expected Choice B= Chronic bronchitis presents with dyspnea and cough and is expected to cause wheezing on lung auscultation. Lymphocytic infiltrate may be present on biopsy, but granulomas are not typical Choice D= Idiopathic pulmonary fibrosis can present quite similarly to chronic HP, but affected patients are usually older (e.g., age >50) and granulomas are not expected on lung biopsy Choice E= Pulmonary TB typically presents with fever, cough, dyspnea, night sweats, and weight loss. Lung biopsy demonstrates lymphocytic infiltrate, but caseating (rather noncaseating) granulomas

-Age: 24 years man -Comes to the office due to paroxysmal episodes of breathlessness and wheezing for the past 6 months. He does not recognize any triggers for these episodes and reports they are not associated with exercise or stress. The patient has no history of recent illness -He had eczema as a child. The patient has no other medical condition and takes no medications. Vital signs are within normal limits. Lung examination shows good air movement and no wheezing. Sputum microscopy reveals many granule-containing cells and crystalloid masses. The sputum findings in this patient are a direct result of which of the following cytokines? a)IL-1 b)IL-5 c)IL-12 d)Interferon-gamma e)TGF-beta

Answer: Choice C (IL-5) -This young patient with paroxysmal dyspnea and wheezing likely has asthma, a disease characterized by airway inflammation, bronchial hyperreactivity, and a variable airflow obstruction. Symptoms that cannot be related to pulmonary infection, inhalation of irritants, stress, exercise, or aspirin ingestion should raise suspicion for atopic (Extrinsic) asthma. Atopic asthma occurs in genetically predisposed individuals; patients often have a family history of asthma, allergies, or eczema. Sputum microscopy classically demonstrates elevated levels of eosinophils (granule-containing cells) with Charcot-Leyden crystals (bipyramidal-shaped accumulations of eosinophil membrane protein) -Atopic asthma occurs due to an excessive TH2-mediated reactions to environmental aero-antigens (e.g., pollen, pet dander). In response to allergic stimuli, these cells secrete IL-5, a critical cytokine for eosinophilic activation, recruitment, and prolonged survival in the bronchial mucosa. TH2 cells also synthesize IL-4, which stimulates IgE formation by plasma cells. As with other type I hypersensitivity reactions, these antigen-specific antibodies bind to receptors on mast cells, and repeated exposure induces mast cell degranulation. This leads to bronchoconstriction, increased vascular permeability, and increased mucus production associated with acute asthma exacerbations. Eosinophils also release inflammatory mediators (i.e., major basic protein, eosinophilic cationic protein) that damage the bronchial epithelium later in the immune response Choice A= Although IL-1 release from macrophages is involved in asthma pathogenesis, it is not a specific cytokine. IL-1 release is a component of almost all inflammatory processes and does not necessarily cause eosinophil infiltration Choice C= IL-12 functions to promote differentiation of TH1 cells; it is not directly involved in eosinophil recruitment Choice D= Interferon-gamma is secreted by helper T cells and functions to activate macrophages, thereby promoting adaptive immunity against intracellular pathogens. It is not directly chemotactic for eosinophils Choice E= Transforming growth factor beta (TGF-beta) is a growth factor involved in tissue regeneration and repair; high levels are associated with subepithelial fibrosis and airway remodeling seen in chronic asthma. However, it is not chemotactic for eosinophils

-Age: 63 years man -Comes to the office due to 3 months of increasing cough with occasional hemoptysis, night sweats, and unintentional weight loss. The patient recently emigrated from South Africa, where he had worked for many years in the gold mines. -He has a history of silicosis that was diagnosed 10 years ago. Vital signs show a low-grade fever. Physical examination reveals diffuse, fine crackles with right upper lobe predominance -Chest x-ray shows diffuse, small nodules, hilar adenopathy with prominent calcifications, and a cavitary lung lesion in the right upper lobe. A sputum sample is sent for appropriate staining and culture. This patient's increased susceptibility to the current infection is best explained by which of the following factors? a)Extensive pulmonary fibrosis b)Impaired immunoglobulin G response c)Impaired macrophage function d)Impaired type 2 helper T cell function e)Ineffective mucociliary clearance

Answer: Choice C (Impaired macrophage function) -This patient with a history of silicosis has developed hemoptysis, night sweats, weight loss, and a cavitary lung lesion in the upper lobe; this is highly suggestive of tuberculosis infection. Silicosis is an occupational lung disease that occurs after inhalation of crystalline silica and is characterized by multiple rounded nodules located in the upper lobes. Patients may also develop calcification of the rim of hilar lymph nodes (eggshell calcification) -Silicosis is associated with increased risk of mycobacterial infections, particularly Mycobacterium tuberculosis, due to impaired macrophage function. It is thought that macrophage phagolysosomes are disrupted by internalized silica particles, leading to impaired phagocytosis and increased apoptosis Choice A= Although interstitial fibrosis can be caused by silicosis, fibrosis alone would not greatly predispose to mycobacterial infection. In fact, scarred areas of lung would be less accessible and may limit spread of microbial pathogens Choices B and D= Th2 cells are important for mediating humoral immunity and combating parasitic infections. However, humoral immunity does not play a major orle in eliminating mycobacterial infections due to the intracellular survivability of these pathogens Choice E= Impaired mucociliary clearance, which occurs in patients with cystic fibrosis, can result in recurrent pulmonary infections due to impaired pathogen clearance. However, mucociliary clearance is not directly impaired by silicosis

-Age: 56 years man -Comes to the office due to worsening cough for the past several months. He initially had a morning cough productive of minimal sputum, but now the cough occurs throughout the day with large amounts of yellowish sputum. The patient also has breathlessness on moderate exertion. He has had 3 hospitalizations over the past year for respiratory infections -The patient has smoked a pack of cigarettes daily for the past 30 years and has failed multiple attempts to quit. Oxygen saturation is 90% at rest and decreases to 84% with moderate exertion. Which of the following additional findings is most likely to be seen in this patient? a)Decreased functional residual capacity b)Decreased right ventricular afterload c)Increased erythropoietin production d)Increased expiratory flow rates e)Increased left ventricular compliance

Answer: Choice C (Increased erythropoietin production) This patient with progressive dyspnea and productive cough in the setting of significant smoking history likely has chronic obstructive pulmonary disease (COPD). His recent hospitalizations for respiratory infections likely represent COPD exacerbations Hypoxemia (O2 saturation <92%) is common in advanced COPD and occurs via 2 major mechanisms: -Emphysematous destruction of the alveolar capillary membrane results in impaired diffusion of O2 -Airway obstruction (due to bronchial inflammation and mucus secretion) and air-trapping (due to both bronchial inflammation and emphysematous airway collapse) cause ventilation-perfusion mismatching Hypoxemia leads to hypoxia throughout the tissues of the body, which is sensed by specialized interstitial cells in the renal cortex and medulla. In response, these cells release erythropoietin, which stimulates the bone marrow to increase production of RBCs and bolster the oxygen-carrying capacity of blood. The resulting increase in hematocrit, known as secondary polycythemia, is an expected finding in disease or conditions that cause significant hypoxemia (e.g., COPD, interstitial lung disease, obesity hypoventilation syndrome, residing at high altitude) Choice A= Air-trapping leads to an increase in the functional residual capacity in patients with COPD Choice B= Hypoxic vasoconstriction in advanced COPD increases pulmonary vascular resistance and leads to increased RV afterload Choice D= Airway obstruction in COPD leads to decreased expiratory flow rates. This is apparent as a reduction in forced expiratory volume in 1 second (FEV1) Choice E= LV compliance is increased in conditions that lead to eccentric LV hypertrophy (e.g., dilated cardiomyopathy, ischemic heart disease, severe aortic or mitral valve regurgitation). It is not significantly affected in COPD

-Age: 46 years hospitalized man -Experiences worsening dyspnea and hypoxemia. The patient was admitted 2 days ago due to alcohol-induced acute pancreatitis. He has no history of cardiopulmonary disorders. Chest x-ray reveals new bilateral infiltrates. -His respiratory condition progressively deteriorates, and endotracheal intubation is performed. Mechanical ventilation with a lung-protective strategy is begun and a positive end-expiratory pressure of 10cm H2O is applied. Which of the following is the most likely effect of this latter intervention? a)Decreased alveolar pressure b)Decreased intrapleural pressure c)Increased functional residual capacity d)Increased intrapulmonary shunting e)Increased minute ventilation

Answer: Choice C (Increased functional residual capacity) The hospitalized patient with worsening hypoxemia and bilateral lung infiltrates most likely has acute respiratory distress syndrome (ARDS), a potential complication of acute pancreatitis. ARDS involves an inflammatory response in the lungs that leads to alveolar capillary leakage and diffuse pulmonary edema; the edema prevents ventilation of affected alveoli and facilitates alveolar collapse, leading to intrapulmonary shunting (perfusion without ventilation). In addition, functional residual capacity (FRC) (i.e., the volume of air in the lungs at end-tidal expiration) is reduced Mechanical ventilation is typically needed for the management of ARDS because it allows for the application of positive end-expiratory pressure (PEEP). PEEP helps treat ARDS by opening collapsed alveoli to reduce intrapulmonary shunting (Choice D) and increased FRC back to near-normal levels. The increase in FRC has the following beneficial effects: -There is a critical lung capacity (i.e., closing capacity) at which some of the lung's small airways collapse during expiration (due to decreased radial traction at lower lung volumes). When FRC drops below this critical capacity, alveoli supplied by the collapsed airways are without ventilation for part of the respiratory cycle, which contributes to intrapulmonary shunting and increases ventilation-perfusion mismatching. Increasing FRC minimizes the time spent below closing capacity and in doing so decreases ventilation-perfusion mismatching -Because the FRC air volume remains in the lungs throughout the respiratory cycle, it acts as a store of oxygen that the body can pull from during brief periods of increased need. Therefore, increasing FRC increases the oxygen reserves in the lungs Choice A= PEEP increases, rather than decreases, alveolar pressure, which has the negative effect of increasing the risk of pulmonary barotrauma Choice B= Intrapleural pressure is normally negative throughout the respiratory cycle due to the opposing elasticity of the lungs (tends to collapse) and chest wall (tends to expand). Increased end-expiratory airway pressure during PEEP reduces the collapsing force of the lungs, causing the intrapleural pressure to increase Choice E= Minute-ventilation is determined by the product of tidal volume and RR. PEEP causes breathing to occur at a higher baseline lung volume (i.e., higher FRC), but tidal volume and RR are not directly affected, and minute ventilation is unchanged

-Age: 66 years woman -Evaluated in the clinic for worsening shortness of breath over the last year. She has no previous medical problems and is a lifelong nonsmoker. BP (126/78), pulse (82), RR (18), BMI (23). Physical examination shows a normal oropharynx, flat neck veins, and a midline trachea -Diaphragmatic excursion is decreased bilaterally. Lung auscultation reveals bilateral fine inspiratory crackles. Arterial blood gas analysis is performed while the patient is at rest and after a vigorous walk for several mins. -Arterial partial pressure of oxygen is normal at rest but falls significantly with exercise. Which of the following exercise-related changes is most likely contributing to the latter observation in this patient? a)Impaired ventilation-perfusion mismatch b)Increased alveolar ventilation c)Increased pulmonary blood flow d)Increased pulmonary capillary recruitment

Answer: Choice C (Increased pulmonary blood flow) -Oxygen transfer across the alveolar membrane depends on (1) gas diffusion rate and (2) capillary blood perfusion rate. In healthy lungs, diffusion of oxygen occurs very rapidly; red blood cells become fully saturated with oxygen at only one-third of the total alveolar capillary length. For this reason, oxygen saturation does not fall even with large increases in cardiac output during exercise. In other words, normal oxygen transfer is perfusion limited (i.e., diffusion is so fast that oxygen transfer depends on the perfusion rate (cardiac output) only) -Diffusion limitation is a mechanism of hypoxia that occurs in diseases the disrupt the alveolar-capillary membrane (e.g., emphysema, pulmonary fibrosis). This patient's progressive dyspnea and fine inspiratory crackles are consistent with interstitial lung disease. Fibrotic thickening of the interstitial space (between the air and blood) increases the distance that oxygen must cross, limiting the degree of oxygen diffusion. During exercise, the increase in pulmonary blood flow accelerates the transit through the pulmonary capillaries, reducing the time for oxygen extraction. In patients with diffusion limitation, the increased blood flow during exercise can result in exertional hypoxemia, even if oxygenation is normal at rest Choices A, B and D= In healthy individuals, exercise results in increased minute ventilation (i.e., product of increased respiratory rate x larger tidal volumes). At the same time, recruitment of apical pulmonary capillaries (increased perfusion) promotes continued ventilation/perfusion (V/Q) matching as ventilation increases. The remarkably stable V/Q matching during exercise enhances oxygen exchange, preventing hypoxemia

-Age: 56 years man -Comes to the ED due to progressively worsening dyspnea. The patient can walk only a few blocks before becoming short of breath. He also finds it difficult to sleep lying flat and requires 3 pillows to prop himself upright when sleeping -Other medical problems include long-standing hypertension, for which he occasionally takes his prescribed antihypertensive medication. The patient does not use tobacco, alcohol or illicit drugs. BP (170/100), pulse (80, regular). Physical examination reveals bilateral basilar lung crackles, jugular venous distention, and bilateral lower extremity edema -Chest x-ray reveals cardiomegaly and hilar prominence. ECG shows left ventricular hypertrophy. Echocardiogram shows elevated pressures in the pulmonary artery. Which of the following is the most likely underlying cause of the observed echocardiographic finding in this patient? a)Hypoxia-induced pulmonary vasoconstriction b)Increased pulmonary arterial blood flow c)Increased pulmonary venous pressure d)Inflammatory pulmonary vascular disease e)Obliteration of the pulmonary vascular bed f)Thrombotic obstruction of the pulmonary arterial tree

Answer: Choice C (Increased pulmonary venous pressure) -This patient with dyspnea, orthopnea, and pulmonary crackles has left-sided heart failure (LHF) most likely due to long-standing, poorly controlled hypertension. Hypertensive heart disease typically manifests as heart failure due to concentric left ventricular (LV) hypertrophy and consequent LV diastolic dysfunction. LHF of any cause (e.g., LV systolic dysfunction, valvular dysfunction) will result in higher diastolic filling pressures. This increase in pressure is transmitted backward to the left atrium and pulmonary veins, resulting in pulmonary venous congestion and consequent elevations in pulmonary capillary and pulmonary arterial pressure. The resulting pulmonary hypertension (PH) can lead to right-sided heart failure with jugular venous distention and peripheral edema -Over time, remodeling of the pulmonary vasculature occurs with increased smooth muscle cell proliferation (medial hypertrophy) and collagen deposition (intimal thickening and fibrosis). The remodeling is less extensive than in (primary) pulmonary arterial hypertension; therefore, the PH is at least partially reversible with treatment of the LHF Choices A and E= Hypoxia-induced vasoconstriction and emphysematous obliteration of the vasculature underlie PH that occurs secondary to COPD. Although there is a small component of hypoxia-induced pulmonary vasoconstriction in LHF, pulmonary venous congestion is the major cause of PH Choice B= Congenital heart disease that causes left-to-right shunting (e.g., ventricular septal defect, atrial septal defect) can lead to PH via an increase in pulmonary arterial blood flow. In LHF, PH occurs due to backwards transmission of pressure from increased pulmonary venous blood volume; pulmonary arterial blood flow remains the same or decreases as LHF worsens Choice D= Inflammatory large-vessel vasculitis (i.e., Takayasu arteritis, giant cell arteritis) can sometimes involve the pulmonary arteries and cause PH. However, pulmonary vascular inflammation does not play a role in PH occurring secondary to LHF Choice F= Thrombotic obstruction of the pulmonary arterial tree occurs in acute pulmonary embolism, and some patients can develop chronic thromboembolic PH despite receiving appropriate anticoagulation therapy. However, this patient has no history of thromboembolic disease, and his orthopnea and crackles are more suggestive of PH due to LHF

-Age: 62 years man -Comes to the physician because of recent weight loss, cough, and occasional hemoptysis. His past medical history is significant for poorly controlled diabetes mellitus and chronic obstructive pulmonary disease treated with bronchodilators and oral corticosteroids -Chest x-ray shows pulmonary infiltrates and an area of cavitation in the right upper lobe. Sputum microscopy shows acid-fast bacilli. Which of the following is the most accurate statement concerning this patient's pulmonary infection? a)First exposure to the bacilli occurred recently b)Healing of the lung lesion would result in Ghon complex formation c)It originated from reactivation of an infection d)It was facilitated by low levels of protective antibodies e)Negative tuberculin skin test would signify strong cell-mediated immunity

Answer: Choice C (It originated from reactivation of an old infection) -This patient's symptoms (cough, hemoptysis, weight loss), acid-fast bacilli on sputum culture, and upper lobe cavitary lesion are suggestive of secondary (reactivation) TB. His advanced age, multiple comorbidities, and partial immune suppression secondary to chronic oral corticosteroid use also place him at risk for reactivation disease. -Primary TB infection occurs following inhalation of aerosolized Mycobacterium TB. The organisms are deposited in the lower lungs and phagocytosed by alveolar macrophages, where they proliferate until the macrophages are activated by Th1 lymphocytes. The infection can be subsequently eliminated if the area of involvement is small enough. However, larger regions of caseating necrosis become walled off, allowing M. TB to survive in a dormant state without causing disease or symptoms. Later in life (usually following immunosuppression by drugs or HIV) the bacteria can reactivate and establish infection in the upper lungs (particularly the apex). The predilection for upper lung regions may be related to decreased lymphatic flow or increased oxygen tension. The organisms multiply in the apices, causing caseous and liquefactive necrosis and extensive cavitary disease. Erosion into the pulmonary vessels can result in severe hemoptysis. Hematogenous dissemination may also occur, causing miliary or extrapulmonary (e.g., Potts disease, tuberculous meningitis) tuberculosis Choice A= Primary TB infection often begins as focal lesion in the mid-to-lower lungs (Ghon focus). M. TB then spreads lymphatically to the hilar lymph nodes, forming a Ghon complex. The organisms can remain dormant in a walled-off Ghon complex for many years before reactivating. Alternatively, the lesion may heal, forming a benign, calcified Ranke complex that is not associated with reactivation TB. Choice B= A Ghon complex forms during primary TB infection and consists of a Ghon focus and hilar lymphadenopathy. The patient's upper lung involvement and cavitary lesion are more characteristic of secondary (reactivation) TB. Healing of this lesion would result in a persistent cavity that may become secondarily infected with Aspergillus flavus and form fungus balls Choice D= M. TB is a facultative intracellular bacterium that can survive and multiply within macrophages; as a result, circulating antibodies cannot bind it to promote phagocytosis or complement-mediated killing. Therefore, humoral immunity plays no role in the control of M. TB Choice E= A negative skin tuberculin test after M. TB exposure suggests anergy against TB antigens and a weak cell-mediated immune response. This can occur in the setting of HIV, sarcoidosis, and other illnesses

-Age: 54 years man -Comes to the ED due to fevers, shaking chills, and cough productive of copious sputum. He began having fever, cough, and sharp chest pains 10 days ago and was prescribed oral antibiotics by his primary care physician after a chest x-ray revealed a right lower lobe infiltrate. The patient has not been taking the medication as advised, and his symptoms have progressively worsened -Temp. (39C), BP (114/62), pulse (116). Physical examination reveals crackles in the right lower lung. Repeat chest x-ray reveals a round density with an air-fluid level in the lower lobe of the right lung. Which of the following is the most important contributor to the observed lung lesion in this patient? a)Interferon-gamma production by CD4 lymphocytes b)Interleukin-12 secretion by dendritic cells c)Lysosomal content release by neutrophils d)Major basic protein release by eosinophils e)Transforming growth factor-beta secretion by macrophages

Answer: Choice C (Lysosomal content release by neutrophils) -Lung abscess is a necrotic infection of the pulmonary parenchyma that usually presents with several days of fever, cough productive of copious sputum (often foul-smelling), and chest x-ray evidence of cavitation with air-fluid level. Most cases are due to aspiration of anaerobic bacteria from the oropharynx, but lung abscess can also develop in the setting of untreated pneumonia -Neutrophils are the key player in the formation of lung abscess. They are recruited from the system circulation by chemokines and are subsequently activated by microbial molecules (e.g., lipopolysaccharide, peptidoglycan, bacterial DNA) and opsonizing factors (e.g., IgG, complement) to phagocytize and destroy the pathogenic bacteria. Activated neutrophils also release cytotoxic granules (lysosomes) containing myeloperoxidase and other digestive enzymes that destroy extracellular bacteria and recruit additional immune components to the area. However, these enzymes also cause significant damage to the pulmonary parenchyma and can result in liquefying necrosis of lung tissue and (potentially) lung abscess Choices A and B= Dendritic and other antigen-presenting cells release IL-12, which stimulates differentiation of Th1 helper cells and production of IFN-gamma by T-cells. Interferon-gamma activates macrophages, leading to the development of mature phagolysosomes capable of destroying phagocytosed bacteria. Macrophages are particularly important for the elimination of certain intracellular infections such as Mycobacterium tuberculosis. Although tuberculosis can be associated with pulmonary cavitation, the cavitations usually form in the upper lobes and generally take months to develop Choice D= Major basic protein is the predominant component of eosinophilic granules and plays a crucial role in the elimination of parasites. Lung abscess formation is primarily mediated by neutrophils, not eosinophils Choice E= TGF-beta is secreted by inflammatory cells and results in the recruitment of fibroblasts and the deposition of connective tissue. This cytokine contributes to scar formation after injury and plays a role in the fibrosis seen with chronic inflammation

-Age: 64 years man -Comes to the office due to 4 weeks of progressive dyspnea. For the past several months, he has a nonproductive cough and fatigue. The patient's medical conditions include degenerative joint disease and peptic ulcer disease. He has smoked 2 packs of cigarettes daily for 38 years but quit 4 years ago. On examination, there are decreased breath sounds and percussive dullness at the base of the right lung. Chest CT scan reveals a right-sided pleural effusion and diffuse nodular thickening of the pleura. On thoracocentesis, bloody fluid is obtained. Pleural biopsy shows proliferation of epithelioid-type cells that are jointed by desmosomes, contain abundant tonofilaments, and are studded with very long microvilli. Diagnosis? a)Adenocarcinoma b)Carcinoid lung tumor c)Mesothelioma d)Small cell carcinoma e)Squamous cell carcinoma

Answer: Choice C (Mesothelioma) -This patient with progressive dyspnea and cough has nodular pleural thickening and a pleural effusion. This presentation, in conjunction with characteristic histopathology, suggests mesothelioma. Mesothelioma is a rare, malignant neoplasm arising from mesothelial cells, which line body cavities (e.g., pleural, peritoneal, pericardial). Histopathology shows tumor cells with numerous long, slender microvilli and abundant tonofilaments. Immunohistochemical markers (e.g., pancytokeratin) are useful in diagnosis -Mesothelioma typically presents with slowly progressive dyspnea, cough, and chest pain. Pleural effusions are common and are often hemorrhagic. Radiography may demonstrate nodular or smooth, unilateral pleural thickening (yellow arrow) and plaque formation; the lung parenchyma is typically uninvolved. Asbestos exposure is the primary risk factor; individuals involved in asbestos mining and industrial applications (e.g., insulation, shipbuilding) are at risk for mesothelioma Choice A= Adenocarcinoma cells (which can display glandular or papillary characteristics) have short, plump microvilli that distinguish them from mesothelioma cells. Adenocarcinoma tends to involve peripheral lung parenchyma Choice B= Carcinoid tumors are neuroendocrine malignancies that most commonly affect the GI tract and lungs. They sometimes cause carcinoid syndrome (flushing, diarrhea, wheezing), which is seen more commonly with GI tumors. They tend to involve the large bronchi and can appear histologically as sheets of uniform cells with a "salt and pepper" pattern (chromatin with fine and coarse clumps) Choice D= Small cell carcinoma is associated with smoking but usually arises from the major bronchi. On chest imaging, it is seen as a hilar mass. Pathology shows flat, oval-shaped cells with scant cytoplasm and hyperchromatic nuclei. Neuroendocrine markers such as chromogranin and synaptophysin are usually positive Choice E= Squamous cell carcinoma is associated with smoking but arises from the major bronchi; a hilar mass would be expected on imaging. Although desmosomes (intercellular bridges) are seen in SCC, they are nonspecific and can be found in any tissue that experiences mechanical stress (e.g., mesothelium, epithelium, heart). In addition, keratinization (keratin pearls) is expected on histologic examination

-Age: 42 years man -Comes to his primary care physician due to daytime sleepiness. He often falls asleep during meetings and while watching television and has even fallen asleep while driving. The patient does not feel refreshed when waking and has occasional morning headaches -He has not had abnormal dreams or visual hallucinations when falling asleep or on walking. The patient has no significant past medical history and is a lifetime non-smoker. He drinks 2 or 3 beers on Friday nights. BP (148/100), pulse (78, regular), BMI (32). -Cardiopulmonary examination shows no abnormalities. Arterial blood gas analysis is normal. Diagnosis? a)Central sleep apnea b)Narcolepsy c)Obesity hypoventilation syndrome d)Obstructive sleep apnea e)Primary insomnia f)Restless leg syndrome

Answer: Choice D (Obstructive sleep apnea) -This patient has several features that suggest obstructive sleep apnea (OSA0, including daytime somnolence, nonrestorative sleep, and elevated BP in the setting of underlying obesity. OSA is caused by closure of the upper airway due to relaxation of pharyngeal muscle tone during sleep. Additional contributing factors include obesity, tonsillar hypertrophy, and hypothyroidism. When the airway is occluded, PO2 declines and PCO2 rises until chemoreceptors in the carotid body and brainstem trigger arousal and pharyngeal tone returns. Sleep is repeatedly disrupted throughout the night, even in the absence of cortical awareness -Typical symptoms include excessive daytime sleepiness, morning headaches, cognitive impairment, and depression. Most patients also experience loud snoring due to partial closure of the airway. Chronic OSA can lead to systemic and pulmonary hypertension with right heart failure and an increased risk for cardiac arrhythmias Choice A= Central sleep apnea is due to diminished respiratory drive from a neurologic disorder. Symptoms may superficially resemble OSA, but it is usually associated with significant underlying chronic illness (e.g., congestive heart failure, cerebrovascular disease, renal insufficiency) and is not more common in obesity Choice B= Narcolepsy is characterized by poorly regulated rapid eye movement (REM) sleep. It frequently causes excessive daytime sleepiness, but patients also suffer from cataplexy (sudden muscle weakness when person awakes), sleep attacks, hypnagogic/hypnopompic hallucinations (hallucinations that seem very real), and sleep paralysis Choice C= Obesity hypoventilation syndrome (Pickwickian syndrome) is caused by restricted expansion of the chest wall due to severe obesity. This leads to hypoventilation with a chronically elevated PCO2 and reduced PO2. This patient's normal blood gases and mild obesity are not consistent with obesity hypoventilation Choice E= Patients with simple insomnia may have daytime fatigue, but nonrestorative sleep, morning headaches, and elevated BP suggest OSA Choice F= Restless leg syndrome is characterized by vague discomfort in the limbs that is brought on when trying to sleep and relieved with movement. Symptoms recur through the night and lead to nonrestorative sleep, but patients are usually aware of the symptoms

-Age: 47 years man -Comes to the ED due to a 30-minute history of sudden-onset severe chest pain and shortness of breath. He is on an RV road trip with his family and has had pain in and swelling of the left leg for the past 2 days -The patient has no significant medical history and takes no medications. Temp. (37.2C), BP (90/56), pulse (124), RR (32). Oxygen saturation is 82% on room air. BMI is 36. ECG shows sinus tachycardia. Chest x-ray shows no abnormalities. -Which of the following is most likely increased in this patient? a)Functional residual capacity b)Lung diffusion capacity c)Physiologic dead space d)Pulmonary compliance e)Total airway resistance

Answer: Choice C (Physiologic dead space) -This patient has developed a deep venous thrombosis in his left leg following a lengthy car trip and now has clinical features highly consistent with acute pulmonary embolism (PE). Obstruction of the pulmonary circulation by an embolus causes increased dead space ventilation (i.e., alveoli are ventilated but not perfused), the term "dead space" refers to the volume of inspired air that does not participate in gas exchange. As dead space ventilation increases, blood that continues to flow through the pulmonary circulation cannot be fully oxygenated by the decreased number of accessible alveoli (ventilation/perfusion (V/Q) mismatch), leading to hypoxemia Choice A= Functional residual capacity refers to the volume of air remaining in the lungs after a normal expiration. It is increased in COPD due to air trapping; however, it is unchanged in PE Choice B= Diffusion capacity is reduced by condition that disrupt the alveolar-capacity interface (e.g., pulmonary fibrosis), but it is not altered in PE Choice D= Pulmonary compliance (ability to expand) is reduced in restrictive lung disease (e.g., pulmonary fibrosis) and is increased in COPD (due to emphysematous destruction of the distal airways). However, it is not significantly altered in the setting of PE Choice E= Total airway resistance is increased by bronchoconstriction, which occurs in patients with asthma. PE does not affect airway resistance

-Age: 32 years man -Hospitalized due to nausea, vomiting and severe abdominal pain. He has a history of heavy alcohol use and was admitted to the hospital for acute pancreatitis a year ago. He has continued to drink alcohol since his last hospitalization and had a party last weekend, during which he consumed an entire 750-mL bottle of liquor. -Temp. (38.3C), BP (110/80), pulse (104), RR (20). Abdominal examination is notable for marked tenderness in the epigastric region. Serum lipase is 2,392 U/L. The patient is admitted and started on IV fluids, analgesics, and antiemetics with subsequent improvement of his symptoms. His vital signs remain stable with adequate urine output. However, on the 2nd day of hospitalization, the patient develops progressive shortness of breath and hypoxemia -Chest x-ray reveals new bilateral opacities. Which of the following parameters is most likely to be normal in this patient? a)Capillary permeability b)Lung compliance c)Pulmonary capillary wedge pressure d)Ventilation/perfusion matching e)Work of breathing

Answer: Choice C (Pulmonary capillary wedge pressure) -This patient with pancreatitis (abdominal pain, elevated lipase, recent heavy alcohol intake) has developed hypoxic respiratory failure with bilateral pulmonary infiltrates. In a young patient without a history of cardiac disease, this is highly suggestive of acute respiratory distress syndrome (ARDS). Direct (e.g., irritant chemicals, trauma, gastric aspiration) or indirect (e.g., pancreatitis, sepsis) pulmonary insults cause excessive cytokine release, leading to endothelial activation, neutrophilic migration to the lung, and degranulation with release of toxic mediators. This process worsens the pulmonary alveolar and endothelial injury, leading to increased pulmonary capillary permeability, leakage of fluid into the alveoli and pulmonary edema (Choice A) -ARDS-associated interstitial edema and hyaline membrane formation along with fluid-filled alveoli collapse and atelectasis result in decreased lung compliance and increased work of breathing (Choices B and E). The fluid-filled, collapsed alveoli are unable to deliver oxygen despite continuing to receive adequate blood flow, resulting in ventilation/perfusion mismatch (i.e., physiologic shunting) and hypoxia (Choice D) -Noncardiogenic pulmonary edema (e.g., ARDS, high-altitude pulmonary edema) can be distinguished from cardiogenic pulmonary edema (e.g., decompensated LV failure, volume overload) by a normal pulmonary capillary wedge pressure (6-12mmHg). Other distinguishing features include the clinical history (pneumonia/sepsis or pancreatitis suggest ARDS) and physical exam (peripheral edema and jugular venous distention suggest a cardiogenic source)

-Age: 24 years man -Comes to the office due to nonproductive cough, subjective fevers, right-sided pleuritic chest pain, and increasing dyspnea for the past 4 weeks. He was empirically treated for pneumonia with levofloxacin at an urgent care center 2 weeks ago but has had no improvement. -The patient is an exchange student from the Philippines. Temp. (37.3C), BP (118/66), pulse (90), RR (20). Physical examination is remarkable for decreased right-sided breath sounds. Chest x-ray shows a large, right-sided pleural effusion -Pleural fluid is drained, and a subsequent chest CT scan reveals right upper lobe infiltrate and calcified hilar adenopathy. This patient's pleural fluid analysis is most likely to demonstrate which of the following findings? a)Cytology positive for atypical cells b)Elevated fluid triglyceride level c)Fluid protein concentration >4.0g/dl d)Normal lactate dehydrogenase level e)Predominant neutrophilic leukocytosis

Answer: Choice C (fluid protein concentration > 4.0g/dl) -This patient's clinical findings are consistent with reactivation tuberculosis, given his nonresponse to routine pneumonia treatment, prior residence in an endemic region (i.e., Southeast Asia), and CT scan showing calcified hilar adenopathy (representing granulomatous lymph node involvement). Reactivation TB typically affects the upper lobes of the lungs and is often associated with pleural effusion -Pulmonary infection causes pleural effusion primarily via an inflammatory increase in vascular and pleural membrane permeability, leading to relatively high pleural fluid protein and lactate dehydrogenase (LDH) concentration (Choice D). Such effusions are exudative by Light criteria, with pleural fluid/serum protein ratio >0.5, pleural fluid/serum LDH ratio >0.6, and/or pleural fluid LDH >2/3 the upper limit of normal serum LDH. Normal serum total protein ranges from 6.0g/dl to 7.8g/dl; therefore, pleural fluid protein >4 g/dl (ratio >0.5) is most likely in this patient Choice A= Pleural fluid cytology showing atypical cells can be seen with a malignant pleural effusion but is not expected with TB effusion. Malignant effusions are also typically exudative by Light criteria Choice B= An elevated pleural fluid triglyceride level is diagnostic of chylothorax, which results from disruption (e.g., trauma, malignant invasion) of the thoracic duct leading to leakage of chylomicron-rich lymphatic fluid into the pleural space Choice E= TB pleural effusions are typically lymphocyte predominant. Neutrophil predominance is expected with a typical parapneumonic effusion (e.g., due to community-acquired bacterial pneumonia)

-Age: 9 months girl -Brought to the ED due to fever and cough. Her symptoms began approximately 2 days ago and have been worsening. The patient is otherwise healthy except for an episode of acute otitis media 3 weeks ago that resolved following a course of oral amoxicillin -She lives with her mother, father, and grandmother who have been in good health. Her mother smokes cigarettes. Temp. (38.5C), RR (34). Pulse oximetry is 95% on room air. Pulmonary examination reveals mild subcostal retractions and focal crackles in the left lower lobe. Which of the following underlying factors is most likely contributing to this patient's current condition? a)Decreased presence of lung monocytes b)Exaggerated airway reactivity c)Impaired mucociliary function d)Inhibited B cell maturation e)Reduced alveolar surface area

Answer: Choice C (impaired mucociliary function) -This patient has fever, cough, and focal crackles, which are findings consistent with pneumonia. Transmission typically occurs via droplet spread of a bacterium (e.g., S. pneumoniae) with subsequent colonization of the nasopharynx and microaspiration into the lungs -Respiratory defense mechanisms often prevent infection of the lower respiratory tract, and the first line of defense is the ciliated epithelium and mucus-producing cells that line the respiratory tract. Pathogens become entrapped in mucus and are transported upward and out of the nasopharynx by the sweeping cilia. Other defense mechanisms include binding by immunoglobulin (e.g., secretory IgA, opsonizing IgG and IgM), phagocytosis by alveolar and interstitial macrophages, and cell-mediated cytotoxic activity -Secondhand smoke exposure disrupts natural host defenses and increases the risk of pneumonia, particularly in infants who have a parent that smokes. Cigarette smoke induces mucus overproduction and impairs ciliary function, leading to delayed clearance of pathogens and respiratory secretions. In addition to respiratory infections, children with secondhand smoke exposure are also at increased risk for asthma and wheezing and are more likely to have recurrent otitis media Choice A= A decreased number of monocytes may occur with bone marrow dysfunction, which would typically cause severe, disseminated infections (not seen in this patient). In addition, although the phagocytic function of alveolar macrophages (differentiated type of monocyte) is impaired in patients with cigarette smoke exposure, these cells are typically present in increased (not decreased) numbers Choice B= Although cigarette smoke exposure is associated with increased risk of asthma, airway hyperreactivity causes bronchoconstriction and wheezing, not pneumonia as seen in this patient Choice D= Inhibited B cell maturation, as seen with X-linked agammaglobulinemia, typically presents with recurrent sinopulmonary infections in young boys, not girls. In addition, cigarette smoke can increase (not decrease) IgE production due to allergic sensitization Choice E= Reduced alveolar surface area occurs with emphysema, which is a late lung finding in patients who smoke. Emphysema would not be present in infancy and does not cause pneumonia

-Age: 14 years girl -Brought to the office due to intermittent shortness of breath over the past 4 months. She is unable to link the episodes to specific events or exposures. The patient has no prior medical conditions and takes no medications. She does not smoke and has no second-hand tobacco exposure. Her family has a cat that remains indoors -There is no significant family history. BP (110/65), pulse (80), RR (16). On physical examination, heart sounds are normal, and the lungs are clear to auscultation. Lung function tests reveal a FEV1/FVC ratio of 83%. Which of the following would be most helpful in excluding a diagnosis of asthma in this patient? a)Absence of peripheral blood eosinophilia b)Negative allergy skin tests c)Negative methacholine challenge d)Normal chest radiography e)Normal total serum IgE level

Answer: Choice C (negative methacholine challenge) -Asthma is a disease of airway inflammation and bronchial hyperreactivity, with variable airflow obstruction that improves with bronchodilator medications. Classic symptoms include cough, shortness of breath, and wheeze, although these are nonspecific. Most patients with asthma have a chronic reduction in the FEV1/FVC ratio (<70%, normal 80%) due to increased expiratory airflow resistance. However, asymptomatic patients may have normal lung function tests, with alterations in lung function occurring only during exacerbations. This patient with intermittent shortness of breath has a normal FEV1/FVC ratio, therefore, further studies are required to exclude the diagnosis of asthma -Bronchial challenge testing is a highly sensitive test with high negative predictive value used to assess bronchial hyperreactivity and exclude asthma. A provocative stimulus (typically aerosolized methacholine, a cholinergic muscarinic agonist) is administered at increasing concentrations to induce bronchoconstriction. Patients with asthma are hyperresponsive to this stimulus and experience a decline in FEV1 at lower doses than nonasthmatics. However, the test is nonspecific and positive results may also be seen with COPD, cystic fibrosis, and allergic rhinitis Choice A= Although peripheral eosinophil counts may be elevated in asthmatics, the presence of eosinophilia is not a sensitive or specific indicator of asthma and may be seen in other allergic diseases (e.g., allergic rhinitis, drug reactions), parasitic infections (e.g., Strongyloides), and certain myeloid neoplasms Choice B= Most patients with asthma are sensitive to a variety of inhaled allergens, and in such patients, skin tests to various allergens will provoke a localized, immediate, type 1 hypersensitivity reaction. However, patients may have allergic disease without asthma and patients with nonallergic (intrinsic) asthma may not have allergies, therefore this test is not useful for diagnosing asthma Choice D= During an asthma attack, the lungs may appear hyperinflated on chest x-ray. However, chest x-rays are often normal between attacks; therefore, a chest x-ray cannot exclude asthma Choice E= Although serum IgE levels are generally elevated in patients with allergic asthma, patients with nonimmune-mediated asthma have normal levels of serum IgE. Intrinsic asthma can be precipitated by pulmonary infections (especially viral), aspirin ingestion, cold air, inhaled irritants (e.g., perfumes), stress and exercise

-Age: 36 years woman -Comes to the clinic for evaluation of a 4-month history of nonproductive cough and exertional dyspnea. Her only other medical problem is frequent heartburn, for which she takes over-the-counter antacids. Temp. (37.1C), BP (126/74), pulse (88), RR (20). Examination shows bilateral crackles and normal heart sounds. The remainder of the examination is unremarkable -Chest imaging shows bilateral interstitial opacities. Bronchioalveolar lavage reveals >50% of cells being lymphocytes. Which of the following is the most likely diagnosis? a)Asbestosis b)Asthma c)Chronic aspiration d)Hypersensitivity pneumonitis e)Pulmonary edema

Answer: Choice D (Hypersensitivity pneumonitis) -The patient with several months of nonproductive cough and exertional dyspnea as well as a lymphocyte-predominant bronchoalveolar lavage (BAL) most likely has hypersensitivity pneumonitis (HP). HP results from an exaggerated immunologic response that some individuals develop to an inhaled antigen (e.g., mold, animal protein). Those most commonly affected include farmers (i.e., farmer's lung due to exposure to moldy hay) and bird keepers (i.e., bird fancier's lung due to exposure to avian proteins) -The presentation of HP can be acute or chronic. Acute disease usually involves recurrent episodes of abrupt-onset cough, dyspnea, fever, chills and fatigue that correlate with intermittent antigen exposure. Chronic disease likely results from chronic, long-term antigen exposure, and presents with gradually progressive cough, dyspnea, fatigue and weight loss over a period of several months. Lung crackles are present with both acute and chronic disease. Chest x-ray in acute disease is frequently normal, whereas diffuse reticular interstitial opacities are present with chronic disease as a network of interstitial inflammation and fibrosis develops -Normally, the leukocytes in alveolar fluid consist of approximately 85% alveolar macrophages, 10% lymphocytes and a small percentage of neutrophils and eosinophils. With both acute and chronic HP, BAL usually shows high relative lymphocyte count (e.g., >20%, often >50%), which helps support the diagnosis. Other causes of high relative lymphocyte count in BAL include sarcoidosis, lymphoma, and chronic fungal or mycobacterial infection Choice A= Asbestosis can present with progressive dyspnea and cough, but BAL, typically shows increased neutrophils and characteristic asbestosis bodies Choice B= Asthma is expected to cause wheezing rather than crackles on lung auscultation. BAL may show increased eosinophils as well as bronchial epithelial cells Choice C= Patients with GERD can have chronic microaspiration, leading to respiratory symptoms (e.g., cough, dyspnea). However, BAL typically shows increased neutrophils and characteristic lipid-laden macrophages that result from the aspiration of lipid-containing food or drink Choice E= BAL in cardiogenic pulmonary edema often demonstrates hemosiderin-laden macrophages, resulting from elevated pulmonary capillary hydrostatic pressure leading to extravasation of RBCs into the alveoli. Leukocyte percentages are not significantly affected

-Age: 19 years man -Has history of chronic lung problems; dies due to respiratory distress after a prolonged episode of pneumonia. Autopsy shows dilation of the bronchial tree, which extends almost to the pleural surface -Many of the small airways are filled with mucin plugs containing abundant inflammatory cells and debris. Which of the following is the most likely cause of this patient's autopsy findings? a)Accumulation of proteinaceous material within the alveoli b)Antigen-mediated aggregation of macrophages c)Congenital deficiency of protease inhibitor d)Impaired clearance of airway secretions e)Increased pulmonary venous pressure

Answer: Choice D (Impaired clearance of airway secretions) -This patient with chronic lung problems has autopsy findings consistent with cystic fibrosis (CF), an autosomal recessive disorder caused by a genetic mutation (e.g., delta-F508) affecting the CF transmembrane conductance regulator. A defect in this chloride channel prevents normal hydration of mucus and results in the accumulation of thick, viscous secretions throughout the body (e.g., lungs, pancreas, vas deferens) -Respiratory disease, the most common CF manifestation, is the predominant cause of morbidity and mortality. Patients typically have chronic cough due to impaired clearance of inspissated secretions in the bronchioles. This buildup leads to mucus plugging (i.e., obstructive lung disease), bacterial colonization (i.e., recurrent pneumonia) and chronic infiltration of inflammatory cells. Over time, elastase produced by neutrophils causes bronchiectasis (weakened, dilated bronchial walls) and parenchymal destruction. Advanced disease is associated with irreversible damage, progressive respiratory failure, and shortened life-expectancy Choice A= Accumulation of proteinaceous material within alveoli occurs with pulmonary alveolar proteinosis, a condition in which clearance of surfactant by alveolar macrophages is impaired. Inflammatory cells are not present in this disorder Choice B= Antigen-mediated aggregation of macrophages describes the formation of granulomas, as occurs with Mycobacterium tuberculosis and histoplasma capsulatum infection. The inciting organism can usually be identified on microscopy and mucus plugs are not characteristically present Choice C= Deficiency of alpha-1 antitrypsin (protease inhibitor of elastase) causes liver dysfunction, emphysema, and eventually bronchiectasis. The onset of pulmonary symptoms typically occurs at age >20, and mucus plugs would not be expected Choice E= Increased pulmonary venous pressure is characteristic of left-sided heart failure, which results in pulmonary congestion and interstitial and alveolar edema. Bronchiectasis is not an associated finding

-Age: 2 days boy -Brought to the ICU form the nursery with tachypnea and hypoxia. The patient was born at 33 weeks, and his mother did not receive prenatal care. Physical examination showed cyanosis. He was intubated but died 2 days later -An autopsy is performed, and a sample of the patient's lung tissue is obtained. Analysis of several large, cuboidal alveolar cells under electron microscopy reveals decreased numbers of granules containing parallel stacks of membrane lamellae. Which of the following pathological processes is most likely to result from the abnormal autopsy findings? a)Alveolar wall destruction by protease b)Bronchial smooth muscle hyperresponsiveness c)Excessive airway mucus production d)Increased alveolar tendency to collapse e)Pulmonary arterial smooth muscle proliferation f)Uncontrolled proliferation of dysplastic bronchial cells

Answer: Choice D (Increased alveolar tendency to collapse) -Surfactant, which is produced in type II pneumocytes, works to decrease the surface tension in alveoli, facilitating lung expansion during respiration. When there is insufficient surfactant, as in neonatal respiratory distress syndrome, the result is the collapse of alveoli (atelectasis) due to increased surface tension. Surfactant is stored and transported to the cell surface by lamellar bodies (organelles containing parallel stacks of membrane lamellae). It is normally released by exocytosis into the alveolar spaces, where the lamellar contents unravel and spread along the alveolar lining. As it degrades, surfactant is recycled back into the same type II pneumocytes by endocytosis for reprocessing Choice A= Emphysema results from excessive activity of intra-alveolar proteases released locally by infiltrating neutrophils and activated alveolar macrophages Choice B= Bronchoconstriction can be caused by alveolar hypocapnia, bronchial inflammation, parasympathetic (cholinergic) efferent nerve activity, and inhaled irritants. A deficiency of pulmonary surfactant would not cause bronchoconstriction Choice C= Chronic bronchitis manifests as excessive mucus production in the airways and increased inflammatory cells visible in the submucosa Choice E= Vasoconstriction of pulmonary arterioles can result from hypoxia secondary to atelectasis. However, actual remodeling of pulmonary vasculature and pulmonary hypertension solely from atelectasis-induced hypoxia are rare Choice F= A deficiency of pulmonary surfactant would not be carcinogenic

-Age: 6 years Caucasian boy -Brought to the office by his mother due to recurrent episodes of sinusitis. He also has a chronic cough that never seem to go away. The mother says that his previous pediatrician performed a "sweat test" to help determine the cause of his frequent infections, but results were normal. DNA testing was inconclusive -Despite these results, the new physician suspects that the patient has an inherited disorder and orders nasal transepithelial potential difference measurements. In this test, an isotonic solution of sodium chloride is applied to the nasal mucosal surface, and the electrical potential overlying the mucosa is compared with that of the interstitial fluid. The results show a transepithelial potential difference that is more negative than normal. This patient's nasal mucosa is most likely to demonstrate which of the following physiologic findings? a)Decreased bicarbonate secretion b)Increased chloride secretion c)Increased mucus water content d)Increased sodium absorption e)Intracellular potassium depletion

Answer: Choice D (Increased sodium absorption) -Chronic cough and recurrent sinusitis in a young Caucasian patient should raise suspicion for cystic fibrosis (CF). The diagnosis of CF typically is based on elevated sweat chloride concentrations, characteristic clinical findings (recurrent sinopulmonary infections, pancreatic insufficiency), and/or a positive family history. However, patients with mild mutations of the CFTR gene may have normal sweat testing. In these cases, a useful diagnostic adjunct involves measuring the nasal transepithelial potential difference -In intestinal and respiratory epithelia, the CFTR channel secretes chloride ions into the lumen and also has a tonic inhibitory effect on the opening of the epithelial sodium channel (ENaC), which decreases sodium reabsorption into the cell. This high luminal salt content helps retain water in the lumen, forming well-hydrated mucus. During the nasal transepithelial potential difference test, a saline solution is applied to the nose. Because patients with CF have increased sodium absorption via the ENaC, sodium is absorbed intracellularly but chloride in the saline solution is retained in the lumen. The higher relative amounts of negatively charged chloride on the epithelial surface results in a more negative transepithelial voltage difference. -CFTR channel functioning is reversed in sweat ducts compared with that in respiratory and intestinal glands. CFTR reduces the salt content of sweat by reabsorbing luminal chloride and stimulating ENaC to increase sodium absorption from the lumen into the cells. CFTR mutations therefore result in the production of sweat with high chloride and sodium content Choice A= CFTR mutations lower the rate of bicarbonate secretion in the pancreatic duct, promoting mucin precipitation and the formation of intraductal concretions that cause exocrine pancreatic insufficiency. However, this effect is not related to the nasal transepithelial potential difference test Choice B= CFTR mutations impair the passive transport of chloride along its concentration gradient, decreasing chloride secretion by respiratory epithelial cells Choice C= The water content of mucus is decreased (not increased) in patients with CF, resulting in dehydrated mucus that predisposes to formation of mucus plugs and chronic sinopulmonary infections Choice E= CFTR mutations do not significantly alter transmembrane potassium transport

-Age: 58 years man -Comes to the clinic due to recent-onset headache and dyspnea. The patient says he has had a puffy face for 2 weeks along with a persistent dry cough. He does not have any neck or shoulder pain. He has no other medical problems. -Physical examination shows symmetrical facial swelling and conjunctival edema. The pupils are equal, round, and reactive to light. Dilated veins are noted over his neck and upper trunk. -Heart and lung sounds are normal. Peripheral strength and sensation are intact. Which of the following is the most likely cause of this patient's condition? a)Airway obstruction b)Autoimmune disease c)Hormone secretion d)Mediastinal mass e)Pericardial effusion f)Pleural effusion g)Superior sulcus tumor

Answer: Choice D (Mediastinal mass) This patient's symptoms are consistent with superior vena cava (SVC) syndrome, a condition that develops when SVC obstruction impairs venous return from the head, neck, upper trunk, and upper extremities The SVC is formed by the union of the right and left brachiocephalic veins behind the right first costal cartilage and descends inferiorly for 6-8cm before draining directly into the right atrium. During its course, the SVC is surrounded by multiple mediastinal structures (e.g., trachea, right bronchus, aorta, right pulmonary artery) and is close to the perihilar and paratracheal lymph nodes. Due to is thin walls, the SVC can be easily compressed by mediastinal masses (most commonly malignancy) that either extend into the mediastinum from the central lung (e.g., bronchogenic carcinoma) or nearby lymph nodes (e.g., lymphoma) or arise from other mediastinal structures Patients with SVC syndrome often experience the following symptoms due to increased venous pressure in the upper body: -Face and neck swelling and conjunctival edema -Distended veins due to increased collateral venous drainage -Headache, dizziness, and/or confusion due to cerebral edema and elevated intracranial pressure -Cough and/or dyspnea due to laryngeal edema and/or direct compression of adjacent structures (e.g., trachea) by the mediastinal mass Choices A and B= Airway obstruction and autoimmune disease can cause dyspnea, but facial swelling and dilated collateral veins would not be expected Choice C= Inappropriate secretion of ACTH by some small cell lung cancers can cause Cushing syndrome, which is often associated with a swollen or moon face due to increased fatty tissue deposits. However, Cushing syndrome is not associated with conjunctival edema or distended neck veins Choice E= Pericardial effusions can cause dyspnea and distended neck veins; however, facial swelling and headaches would not be expected. In addition, distant heart sounds or pulsus paradoxus (decrease in SBP >10mmHg during inspiration) is often present Choice F= Patients with pleural effusions often have dyspnea but do not typically have facial swelling or dilated veins. In addition, decreased breath sounds and dullness to percussion are often present Choice G= Superior pulmonary sulcus tumors (Pancoast tumors) arise at the lung apex and can also cause SVC syndrome. However, shoulder pain is typically present due to tumor invasion into the brachial plexus or nearby chest wall. Horner syndrome (ptosis, miosis and anhidrosis) also is common due to cervical sympathetic ganglia involvement. Neurologic deficits (e.g., hand weakness) also can occur. The patient's lack of shoulder pain, Horner syndrome (his pupils are normal), and neurologic deficits (peripheral strength is normal) makes a superior sulcus tumor unlikely

-Age: 50 years man -Comes to the clinic for evaluation prior to a left total knee arthroplasty. Medical history is significant for osteoarthritis. The patient does not smoke. Preoperative chest x-ray demonstrates a 1.5cm, round lesion in the periphery of the right upper lobe of the lung. -Further imaging studies are suggestive of a pulmonary hamartoma. If a transthoracic biopsy were performed, which of the following microscopic features would be most likely consistent with this diagnosis? a)Atypical cells lining alveoli without invasion b)Expression of neuroendocrine markers c)Invasive nest with keratin pearl formation d)Nodules of cartilaginous tissue e)Sheets of large pleomorphic cells

Answer: Choice D (Nodules of cartilaginous tissue) -Pulmonary hamartomas are common, benign lung lesions that usually present as asymptomatic, incidental findings on chest imaging in middle-aged adults. In radiographs, they are typically solitary, small (<3cm), round, well-circumscribed lesions (i.e., coin lesions) with a popcorn pattern of calcification; they often grow slowly at the periphery of the lung. Microscopy shows nodules of mature connective tissue (e.g., cartilage, fat) and clefts lined by entrapped respiratory epithelium -Hamartomas are lesions composed of tissue types that are native to the organ of involvement but show an abnormal, disorganized arrangement. Many hamartomas, including pulmonary hamartomas, are now thought to be benign neoplastic lesions (rather than malformations), as evidenced by the demonstration of clonal chromosomal abnormalities Choice A= Microscopic features of adenocarcinoma in situ include atypical cells lining the alveolar septa (i.e., lepidic growth) without invasion. On chest x-ray, adenocarcinoma in situ often appears as a solitary, peripheral mass or as a pneumonia-like consolidation Choice B= Neuroendocrine markers (e.g., synaptophysin, chromogranin) expression is characteristic of neuroendocrine tumors (e.g., small cell carcinoma, carcinoid tumors). Small cell lung carcinoma commonly presents as a rapidly growing, central mass in patients with a history of smoking. Carcinoid tumors often arise within the bronchi and may present with symptoms of bronchial obstruction Choice C= Invasive nests of tumor cells with keratin peal formation are a feature of squamous cell carcinoma, which typically a centrally located lung lesion in patients with a history of smoking Choice E= Large cell carcinoma is an aggressive neoplasm that demonstrates large, pleomorphic, epithelial tumor cells without squamous, glandular, or neuroendocrine differentiation (i.e., diagnosis of exclusion). Large cell carcinoma frequently presents as an enlarging, necrotic, peripheral lung mass

-Age: 35 years man -Comes to the office due to worsening shortness of breath. The patient is an avid hiker; he began to have shortness of breath 18 months ago on challenging hikes, but it now occurs even while he is walking. He has also had mild wheezing and sputum production recently. -The patient takes an antihistamine as needed without much improvement. He has no other medical problems. The patient has smoked half a pack of cigarettes daily for the past 5 years. -His father died from lung and liver disease at a young age. Spirometry shows decreased FEV1, decreased FVC, and decreased FEV1/FVC ratio. This patient's lower lung lobes are most likely to demonstrate which of the following findings? a)Centriacinar emphysema b)Colonization with pathogenic bacteria c)Compensatory hyperinflation d)Panacinar emphysema e)Subpleural blebs

Answer: Choice D (Panacinar emphysema) -This patient with chronic progressive dyspnea has spirometry findings (reduced FEV1/FVC ratio) that are consistent with obstructive lung disease. His young age and family history of lung and liver disease suggest emphysema due to alpha-1 antitrypsin deficiency. The diagnosis is further suggested by his minimal smoking history; COPD disease due to tobacco exposure along is most commonly seen in those with a >30-pack-year smoking history -Alpha-1 antitrypsin is the major inhibitor of neutrophil elastase, and deficiency leads to unchecked elastase-mediated tissue damage in the lungs. Because alpha-1 antitrypsin is deficient throughout the acinus, the entirety of the acinus is affected, resulting in panacinar emphysema. The lower lung lobes are predominantly affected, likely due to relatively greater perfusion compared to the upper lung lobes, allowing for an increased rate of neutrophil infiltration Choice A= Centriacinar emphysema is characteristic of tobacco-related emphysema, as only the portion of the acinus most exposed to smoke particles is affected. Tobacco-related centriacinar emphysema predominantly affects the upper lung lobes, possibly due to a higher ventilation-perfusion ratio in those regions Choice B= Colonization of the lung with pathologic bacteria most commonly occurs in patients with cystic fibrosis and is not a common feature of alpha-1 antitrypsin deficiency Choice C= Compensatory hyperinflation refers to the expansion of normal lung parenchyma that occurs when adjacent lung segments or lobes collapse or are surgically removed Choice E= Subpleural blebs can develop in severe emphysema but typically occur in the apices, rather than the bases, of the lung. Their rupture is a cause of spontaneous pneumothorax

-Age: 45 years man -Evaluated for persistent cough and progressive dyspnea on exertion. The patient has no medical issues but has been exposed to large quantities of coal dust while working as a miner for the past 15 years. Chest imaging reveals small, nodular opacities in the upper lobes. Lung biopsy shows fine carbon particles within the patient's respiratory bronchioles and alveolar ducts. Which of the following mechanisms is most directly responsible for clearing the foreign particles from this patient's respiratory tract? a)Coughing and sneezing b)Immunologic memory c)Mucociliary transport d)Phagocytosis e)Upper airway trapping

Answer: Choice D (Phagocytosis) -This patient has coal worker pneumoconiosis, a form of interstitial lung fibrosis secondary to long-term inhalation of coal particles -Inorganic dust is constantly being inhaled and must be cleared by the respiratory tract to prevent disease. The clearance mechanisms used by the lungs vary depending on the size of the particles. Larger particles become trapped by mucus secretions in the trachea, bronchi, and proximal bronchioles; these trapped particles are swept upward toward the pharynx by the collective beating of ciliated cells. The finest particles (<2um) can travel past the highly ciliated airways into the respiratory bronchioles and alveoli, where they are phagocytized by alveolar macrophages -Engulfment of inorganic dust causes macrophage activation and the release of a number of cytokines that induce pulmonary inflammation. Growth factors, including PDGF and insulin-like growth factor (IGF), are also released and stimulate fibroblasts to proliferate and produce collagen. This production results in progressive interstitial lung fibrosis that characterizes the pneumoconioses Choices A and E= The largest dust particles become trapped in the upper airways (e.g., nasal passages, pharynx) and are cleared through coughing and sneezing Choice B= Immunologic memory can help with the clearance of previously encountered microorganisms but would not contribute to the removal of inorganic dust particles Choice C= Mucociliary transport is the primary means of eliminating particles that lodge in the bronchi and bronchioles (typically, medium-sized particles)

-Age: 2 days boy -Develops abdominal distention and refuses to breastfeed. He was born via normal spontaneous vaginal delivery at 39 weeks gestation to a primigravid mother. Examination shows significant abdominal distention with palpable intestinal loops -The rectum has no stool and rectal tone is normal. During the examination, the infant has several episodes of dark green emesis. Plain films of the abdomen show air-fluid levels and small bowel dilation. Contrast enema fails to relieve the obstruction, and the patient is taken for emergency laparotomy, which shows an inspissated, green fecal mass obstructing the distal ileum. The infant recovers from the surgery uneventfully. Which of the following will be the most likely cause of mortality in this patient? a)Enterocolitis b)Liver cirrhosis c)Malabsorption d)Pneumonia e)Renal failure

Answer: Choice D (Pneumonia) -This patient's abdominal distention, bilious emesis, and x-ray findings (air-fluid levels, small bowel dilation) are consistent with bowel obstruction. The intraoperative finding of a green inspissated mass (dehydrated meconium) in the distal ileum points to the diagnosis of meconium ileus as the source of obstruction. Meconium ileus is a very specific finding for cystic fibrosis (CF). Although only 10-20% of infants with CF will have meconium ileus at birth, almost all full-term infants with meconium ileus are eventually diagnosed with CF. Abnormalities in chloride, sodium and water transport by the ductal epithelium of intestinal mucous glands cause isotonic dehydration of the lumen contents. This results in secretion of abnormally viscous mucus into the small bowel. In the US, pneumonia, bronchiectasis, and cor pulmonale account for most deaths due to CF Choice A= Patients with Hirschsprung disease (HD) typically present with signs of interstitial obstruction in the newborn period that are similar to meconium ileus. However, HD typically presents with increased rectal tone, "squirt-sign" (forceful expulsion of stool after rectal examination), and obstruction at the rectosigmoid region with a transition zone between the aganglionic rectum and proximal dilated colon. Enterocolitis (diarrhea, abdominal pain/distention, and fever) is the most common cause of mortality in patients with HD Choice B= Focal biliary or multilobular cirrhosis due to plugging of bile canaliculi by dehydrated mucinous material develops in approximately 5% of CF patients. Death from cirrhotic liver failure is much less likely than terminal pneumonia Choice C= Most patients with CF have exocrine pancreatic insufficiency, which causes malabsorption and steatorrhea. Infants often have large foul-smelling stools, abdominal distention, and poor weight gain. Consequences include deficiencies of fat-soluble vitamins, hypoproteinemia with generalized edema, and persistent diarrhea with rectal prolapse. However, pancreatic involvement is rarely fatal with pancreatic enzyme supplementation Choice E= Although the CFTR protein is expressed in nephrons, patients with CF usually do not manifest serious renal dysfunction or failure. Patients with CF, however, are at elevated risk for nephrolithiasis and renal damage from nephrotoxic medications (e.g., aminoglycosides)

-Age: 48 years man -Dies in a motor vehicle crash. He had a history of IV drug use and had recently been released from prison. The patient's other medical conditions: hepatitis C and hypertension -On autopsy, he is found to have a small, fibrotic focus in the upper portion of the lower lobe of the right lung and a calcified right hilar lymph node. These autopsy findings are most consistent with which of the following? a)Hematogenous dissemination of Mycobacteria tuberculosis b)Mycobacterial elimination from the body c)Poor immunity against mycobacterial infection d)Primary M. tuberculosis infection e)Secondary M. tuberculosis disease

Answer: Choice D (Primary M. tuberculosis infection) -Primary infection with M. tuberculosis (TB) begins with the inhalation of contaminated respiratory droplets into the lower lobe of the lung. Initially, the organism replicates in an unchecked fashion within the alveoli due to virulence factors (e.g., cord factor) that prevent bacterial destruction by alveolar macrophages. The pathogen is also carried by infected macrophages through the draining lymphatic system to ipsilateral hilar lymph nodes. These initial areas of infection (lower lobe, ipsilateral hilar lymph node) are collectively referred to as a Ghon complex -After a few weeks, antigen presenting cells display mycobacterial antigens leading to the stimulation of CD4 cells and the subsequent cytokine-mediated activation of macrophages. Activated macrophages control the infection (in >95% of patients) by forming mature phagolysosomes to kill intracellular mycobacteria and by differentiating into cells (e.g., epithelioid cells, Langerhans giant cells) that wall off extracellular mycobacteria within granulomas. Over time, granulomas become fibrosed and calcified, which reinforced their structure. It also allows areas of primary infection to be visualized on x-ray or gross pathology. A fibrosed and calcified Ghon complex is referred to as a Ranke complex Choices A and C= Most patients with primary TB contain the infection, but those with ineffective immunity (e.g., extremes of age, immunosuppression) can develop progressive illness with hematogenous spread to distant areas (e.g., Potts disease, meningitis). Widespread, massive hematogenous dissemination can also occur, resulting in innumerable, small, millet seed-like lesions in multiple organs (miliary TB) Choices B and E= Primary TB infections are usually contained within granulomas but not fully eliminated. If the walls of the granuloma weaken, M. TB can escape and cause reactivation (secondary) disease. This is most common in the setting of acquired immunosuppression (e.g., HIV, tumor necrosis factor-alpha inhibitors) and usually presents with symptoms (e.g., weight loss, cough and fever) and apical cavitary lesions

-Age: 32 years woman -Comes to the office due to exertional dyspnea that has progressed over the last year. She can hardly walk a block without stopping to rest. The patient's mother died of "heart failure" at age 40. During auscultation, the pulmonary component of S2 is louder than the aortic component in the right and left second intercostal space. -An accentuated impulse can also be palpated along the left upper sternal border. Chest x-ray shows clear lungs. Which of the following is the most likely cause of this patient's findings? a)Hypertensive heart disease b)Hypertrophic cardiomyopathy c)Left bundle branch block d)Pulmonary hypertension e)Pulmonic valve stenosis

Answer: Choice D (Pulmonary hypertension) -This patient's clinical presentation is consistent with pulmonary hypertension. The disease most commonly presents with progressive dyspnea, and patients may also experience exertional angina or syncope. Physical examination reveals a loud pulmonic component (P2) of S2, caused by forceful pulmonic valve closure in the setting of high pulmonary arterial pressure. In addition, the right ventricle becomes enlarged due to increased pressure load (i.e., concentric RV hypertrophy), which can create an accentuated impulse palpated at the left sternal border (left parasternal lift due to RV heave) -Pulmonary arterial hypertension describes pulmonary hypertension directly caused by vascular remodeling of the small pulmonary arteries/arterioles; relatively young women are most commonly affected. Fatigue and exertional dyspnea are common and result from decreased cardiac output due to the inability of the RV to pump blood through the lungs. Right-sided heart failure eventually develops, but because LV function remains intact, there is an absence of pulmonary edema in pulmonary arterial hypertension Choice A= Hypertensive heart disease occurs as the result of prolonged systemic hypertension. It involves concentric LV hypertrophy, leading to impaired ventricular compliance and diastolic heart failure. Pulmonary edema is expected with diastolic heart failure that is severe enough to cause dyspnea Choice B= Hypertrophic cardiomyopathy typically demonstrates a crescendo-decrescendo systolic murmur best heart at the left sternal border. The murmur intensifies with maneuvers that decrease LV blood volume (e.g., abrupt standing, Valsalva strain phase) Choice C= LBBB can delay closure of the aortic valve, leading to later occurrence of aortic component (A2) of S2 and narrowed or paradoxical splitting. However, LBBB is unlikely to significantly affect the relative audible intensity of A2 and P2 Choice E= Pulmonic valve stenosis can delay the occurrence of P2 (later valve closure) but typically leads to a softer intensity of P2

-Age: 54 years man -Comes to the office due to shortness of breath that has slowly worsened over the last 6 months. He also has a persistent, nonproductive cough. The patient does not use alcohol, tobacco, or illicit drugs. Temp. (36.8C), BP (132/78), pulse (74), RR (16). -Physical examination reveals fine crackles bilaterally on pulmonary auscultation and drumstick-shaped fingers. Chest x-ray reveals diffuse reticular opacities. Pulmonary function testing shows decreased FVC, increased FEV1/FVC ratio, and expiratory flow rates that are higher than normal when corrected for lung volume -This patient's supernormal expiratory flow rates are best explained by an increase in which of the following parameters? a)Lung compliance b)Outward recoil of the chest wall c)Physiological dead space d)Radial traction on airway walls e)Wall thickness of bronchioles

Answer: Choice D (Radial traction on airway walls) -This patient's clinical presentation (progressive dyspnea, fine crackles, clubbing, diffuse reticular opacities) is consistent with interstitial lung disease (ILD). Most ILDs cause progressive pulmonary fibrosis with thickening and stiffening of the pulmonary interstitium. This causes increased lung elastic recoil, as well as airway widening due to increased outward pulling (radial traction) by the surrounding fibrotic tissue. The resulting decrease in airflow resistance leads to supernormal expiratory flow rates (higher than normal when corrected for lung volume) -Additional pulmonary function test findings in restrictive lung diseases include reduced total lung capacity, vital capacity, inspiratory capacity, functional residual capacity, and residual volume. The FVC and FEV1 are decreased as well. However, the FEV1/FVC ratio is typically normal or increased as FEV1 decreases less than FVC (due to airway widening relative to the low lung volumes). In addition, fibrosis causes a reduction in the diffuse capacity for carbon monoxide Choice A= Lung compliance is defined as the change in lung volume for a given change in pressure. Increased compliance means that the lung tissue stretches and expands more in response to increased pressure. Restrictive lung diseases cause decreased lung compliance and lower lung volumes due to interstitial fibrosis Choice B= The outward recoil of the chest wall limits the volume of air that can be expired during maximal exhalation. An increase in outward chest wall recoil would decrease expiratory flow rates, as the respiratory muscles must work harder during expiration to oppose the increased outward force Choice C= Physiological dead space refers to the volume of inspired air that does not participate in gas exchange. Changes in the amount of physiological dead space can affect alveolar ventilation but do not affect expiratory flow rates Choice E= Thickening of the bronchioles, which occurs in some obstructive lung diseases (e.g., asthma), leads to narrowing of the airways. Although reduced airway radius leads to local increase in airflow velocity (Venturi effect), overall, the increased resistance causes a decrease in expiratory flow rates. In fibrotic lung disease, the interstitum, not the bronchiolar walls, is thickened

-Age: 48 years woman -Previously healthy, seen in the clinic for community-onset bacterial pneumonia. Chest x-ray reveals dense consolidation of the right middle lobe. Her pulse oximetry is 90% on room air. -The patient is treated with appropriate antibiotics. Two days later, the patient feels well overall, and her pulse oximetry is improved to 98% on room air. Examination reveals unchanged crackles and egophony from her previous visit. Which of the following processes best explains this patient's improved oxygenation? a)Decreased alveolar consolidation b)Decreased hemoglobin oxygen-binding affinity c)Increased erythrocyte production d)Restored hypoxic pulmonary vasoconstriction

Answer: Choice D (Restored hypoxic pulmonary vasoconstriction) -This patient has community-acquired bacterial pneumonia with lobar consolidation (i.e., alveoli filled with pus) and hypoxemia. Continued blood flow (perfusion) through these gasless (nonventilated) alveoli results in severe ventilation-perfusion (V/Q) mismatch, the mechanism of hypoxemia -The normal response to local alveolar hypoxia is hypoxic pulmonary vasoconstriction (HPV). Hypoxia is detected by mitochondria in pulmonary vascular cells, stimulating smooth muscle contraction. HPV limits blood flow to nonventilated alveoli, shunting it toward better-ventilated ones, preserving V/Q matching to optimize oxygenation -However, HPV is impaired (released) by inflammatory states such as acute pneumonia or sepsis. Proinflammatory cytokines cause regional vasodilation, resulting in hyperemia (increased perfusion) in affected lung areas, further lowering the V/Q ratio and worsening the hypoxemia -Once treatment is initiated, vasoactive inflammatory mediators are downregulated over the ensuing hours to days and HPV is restored. This occurs prior to resorption of alveolar debris and fibrinous edema, a slower process carried out by alveolar macrophages. Therefore, radiographic clearance of pneumonic infiltrates often lags weeks behind clinical improvement in oxygenation. A repeat chest x-ray obtained at this time would demonstrate unchanged consolidation, consistent with her crackles and egophony (Choice A) Choices B and C= Chronic hypoxemia (e.g., obstructive sleep apnea, living at high altitudes) causes increased erythropoietin levels to stimulate red cell production, augmenting the oxygen carrying capacity of blood. At the same time, the oxygen affinity of hemoglobin decreases, partially due to allosteric effects of (anaerobic) glycolytic intermediates, promoting greater oxygen release to the tissues. These chronic adaptations merely compensate for hypoxemia but do not actually correct it, and they are unlikely to occur acutely (e.g., initial days following bacterial pneumonia)

-Age: 55 years woman -Comes to the office due to a cough producing thick, yellowish sputum for the past 2 weeks. She has also had night sweats and increased fatigue over this time. Temp. (38.3C), BP (110/70), pulse (88), RR (20). There are crackles on lung auscultation. -Chest imaging reveals a nonspecific pulmonary lesion. A transthoracic needle aspiration is performed. The specimen is cultured and grows several bacterial species, including Peptostreptococcus and Fusobacterium. Which of the following predisposing factors is most likely responsible for this patient's condition? a)Mitral valve prolapse b)Occult malignancy c)Penetrating chest trauma d)Seizure disorder e)Tobacco smoking f)Urinary infection

Answer: Choice D (Seizure disorder) Peptostreptococcus and Fusobacterium are anaerobic bacteria that are part of normal mouth flora. The presence of these organisms in this patient's lung lesion is highly suggestive of developing lung abscess. As an abscess evolves, it typically forms a cavitary lesion with an identifiable air-fluid level on imaging. Symptoms are often indolent and include fever, night sweats, weight loss, and a cough producing foul-smelling sputum (indicating anaerobes) Lung abscesses are usually caused by one of the following: -Aspiration of oral bacteria into the lower airways (most common)- these abscesses are usually composed of a combination of anaerobic oral flora (e.g., Peptostreptococcus, Prevotella, Bacteroides, Fusobacterium) and aerobic organisms (e.g., streptococcus). Risk is greatest in those who have conditions associated with loss of consciousness or impaired swallowing, such as alcoholism, drug abuse, neurologic disease (e.g., seizures, stroke), or anatomic abnormalities (e.g., esophageal strictures or diverticula) -Bacterial pneumonia- Lung abscess can occur in the setting of certain bacterial pneumonias such as those due to S. aureus, E. coli, Klebsiella pneumoniae, or P. aeruginosa. Most cases arise in the hospital setting and occur in patients with immunosuppression, older age, or underlying lung disease -Bacteremia and/or infectious endocarditis- hematogenous spread of an infection to the lung usually causes multiple, monomicrobial lung abscesses. The most common causative agents are Staphylococcus and Streptococcus spp. Choice A= Mitral valve prolapse is a risk factor for development of subacute bacterial endocarditis (with Streptococcus species). Left-sided endocarditis could cause septic embolization to downstream organs (e.g., spleen, brain) but would not send septic emboli to the lungs Choice B= Bronchogenic carcinoma can cause bronchial obstruction and postobstructive pneumonia, which may lead to secondary abscess formation. However, these lung abscesses would be less likely to contain oral anaerobic flora Choice C= Lung abscess may be associated with a prior penetrating trauma. Staphylococcus and Streptococcus spp. (introduced from the skin) are usually responsible Choice E= Tobacco smoking compromises pulmonary defenses and predisposes patients to developing community-acquired pneumonia due to common pathogens such as Streptococcus pneumoniae and Mycoplasma pneumoniae. These organisms are less likely to cause lung abscess Choice F= Urinary infections can be associated with bacteremia, particularly in those with diabetes mellitus, advanced age, or structural urinary abnormalities. However, most urinary infections are caused by coliform bacteria (e.g., E. coli) from the lower GI tract. Lung abscess is not a common complication of urinary infection, and Peptostreptococcus and Fusobacterium would be unusual urinary organisms

-Age: 65 years man -Comes to the office due to several weeks of nonproductive cough. The patient also has anorexia and unintentional weight loss. His medical history: hypothyroidism due to Hashimoto thyroiditis and a 50-pack-year smoking history. -Examination shows an enlarged right supraclavicular lymph node. Imaging reveals a large mediastinal mass causing tracheal deviation. The enlarged node is biopsied, and microscopy demonstrates clusters of small, ovoid cells with scant cytoplasm and high mitotic count. -Immunohistochemical staining is positive for chromogranin. This patient most likely has which of the following conditions? a)Adenocarcinoma b)Hodgkin lymphoma c)Papillary thyroid carcinoma d)Small cell cancer e)Squamous cell cancer

Answer: Choice D (Small cell cancer) -This patient with a heavy smoking history, weight loss, cough, and a mediastinal mass with evidence of metastases (i.e., supraclavicular node enlargement) has small cell lung cancer (SCLC), also known as oat cell carcinoma. SCLC makes up 10-20% of all primary lung malignancies and is strongly associated with smoking. It is usually centrally located and arises from the primitive cells of the basal layer of the bronchial epithelium. On light microscopy, small, round or oval cells with scant cytoplasm, hyperchromatic (blue) nuclei, and granular chromatin are visualized. The cells, which may resemble lymphocytes but are typically larger, form clusters or sheets. Abundant mitoses are usually seen -Small cell carcinomas can display varying degrees of neuroendocrine differentiation. Immunohistochemical stains are frequently positive for neuroendocrine markers, such as neuron-specific enolase, chromogranin, neural cell adhesion molecule (CD56), and synaptophysin. On electron microscopy, some of the cells have secretory granules in the cytoplasm Choice A= Adenocarcinoma is the most common form of cancer in both nonsmokers and the total population. It is histologically characterized by glandular differentiation (e.g., gland formation, mucin production), and the tumor cells often show abundant cytoplasm and eccentrically placed nuclei. The cells often stain positive for mucin. It typically arises in the periphery of the lung Choice B= Hodgkin lymphoma typically presents with cervical lymphadenopathy and can cause a mediastinal mass. However, microscopy demonstrates Reed-Sternberg cells (large cell with multilobulated nucleus or multiple nuclei, prominent nucleoli, abundant cytoplasm, and an "owl's eye" appearance) in a background of inflammatory cells. In addition, other B symptoms (e.g., fever, night sweats) are common Choice C= Papillary thyroid carcinoma presents with a thyroid nodule and cervical lymphadenopathy but can also cause an anterior mediastinal mass. Histology reveals branching papillae, which are composed of a fibrovascular stalk covered by neoplastic cuboidal cells with clear ("ground-glass") nuclei. Psammoma bodies (laminated calcium deposits) can sometimes be seen Choice E= Squamous cell carcinoma typically arises centrally and is composed of polygonal cells with eosinophilic cytoplasm and distinct borders. Well-differentiated squamous cell carcinomas show keratin pearls and intercellular bridges on light microscopy

-Age: 4 months girl -Brought to the office for a well-child evaluation. The girl has been growing and developing normally and her parents have no concern. When questioned about the home environment, the parents say that they both smoke cigarettes but "try to smoke outside the house". -The patient's mother smoked a pack of cigarettes a day until her second trimester, when she found out she was pregnant. However, she resumed smoking shortly after delivery and says that the stress of sleep deprivation and difficulty with postpartum weight loss have made it difficult for her to quit. The parents are counseled on the risks of secondhand smoke exposure -Which of the following conditions is the infant at highest risk for developing from exposure to cigarette smoke? a)Eczema b)Food allergies c)Obesity d)Sudden infant death syndrome e)Urinary tract infection

Answer: Choice D (Sudden infant death syndrome) -Secondhand smoke (SHS) has numerous adverse effects on nearly every organ system. The level of tobacco in the fetus of a mother who smokes is the same as that of an active smoker. Maternal tobacco use impairs fetal oxygenation, alters fetal development and response, and exposes the fetus to multiple toxins (e.g., nicotine, carbon monoxide, ammonia). As a result, detrimental outcomes include abnormal placentation (e.g., previa, abruption), prematurity, perinatal mortality, and significantly reduced birth weight -One of the most dangerous effects of SHS exposure (pre- and postnatal) is the increased risk of sudden infant death syndrome (SIDS). SIDS refers to the unexpected death of a seemingly healthy infant during sleep. Up to half of all SIDS cases are due to SHS exposure, likely due to impaired arousal and abnormal cardiovascular responses to stimuli -SHS also increases the risk of recurrent otitis media, asthma and other respiratory tract illnesses (e.g., pneumonia) in children. Parents who smoke outside the home should be counseled on cessation as chemicals from cigarette smoke are adsorbed and retained by clothing, skin, and hair and therefore pose a risk to children Choices A and B= Although the risk of asthma increases with secondhand smoke exposure, there appears to be no increased risk of other atopic diseases, such as eczema or food allergies. The major risk factor for atopic disease is a family history of atopy Choice C= Although prenatal smoke exposure is associated with low birth weight and poor fetal growth, it doesn't increase the risk of obesity. The primary risk factors for childhood obesity are parental obesity and excessive TV viewing Choice E= SHS exposure has been shown to reduce renal function (decreased GFR) in adolescents but does not increase the risk of UTIs. Risk factors for recurrent UTIs in children include anatomic anomalies (e.g., posterior urethral valve, vesicoureteral reflux)

-Age: 42 years woman -Comes to the office due to several months of episodic shortness of breath and cough. The patient has a history of migraines and occasionally takes acetaminophen. -She is a school teacher and does not use tobacco, alcohol, or illicit drugs. Temp. (36.1C), BP (110/80), pulse (84). Pulse oximetry shows 96% on room air. Lung examination and chest x-ray are normal. Spirometry shows that FEV1 is 71% of the predicted value. Sputum analysis shows predominant eosinophils. Exposure to which of the following is most closely associated with this patient's symptoms? a)Acetaminophen b)Gliadin-containing grains c)Histoplasma microconidia d)House dust mites e)Shellfish and other seafood f)Thermophilic actinomycetes

Answer: Choice D (house dust mites) -This patient's intermittent respiratory symptoms (e.g., dyspnea, cough) in the setting of a normal chest x-ray, sputum eosinophils, and reduced FEV1 (<80% predicted) are highly suggestive of asthma -Asthma is a reversible obstructive pulmonary disease characterized by bronchial hyperresponsiveness to various triggers, such as exercise, cold air, respiratory infection, and exposure to inhaled allergens. Common aeroallergens include dust mites, cockroaches, pet dander, mold, and pollen. In sensitive individuals, allergens promote bronchospasm and airway inflammation by interacting with IgE bound to pulmonary mast cells; subsequent mast cell activation leads to the release of inflammatory mediators (e.g., histamine, prostaglandins, leukotrienes) and recruitment of inflammatory cells, particularly eosinophils. Therefore, allergen avoidance is an important preventive measure for patients with asthma Choice A= Aspirin and other NSAIDs that inhibit cyclooxygenase can precipitate asthma attacks due to leukotriene overproduction in the airways. Acetaminophen is only a weak inhibitor of cyclooxygenase, and although some evidence suggests an increased incidence of asthma in patients with frequent acetaminophen use, a clear association has not been established Choice B= Celiac disease (gluten-sensitive enteropathy) is caused by antibodies to gliadin, a gluten protein found in wheat. Patients classically have signs/symptoms of malabsorption (e.g., diarrhea, steatorrhea, weight loss, nutrient/vitamin deficiencies) Choice C= Although aspergillus molds are ubiquitous in the environment and are a common trigger of asthma symptoms. Histoplasma mold is a relatively uncommon trigger. Aspergillus molds can also cause allergic bronchopulmonary aspergillosis, a chronic allergic lung condition that typically affects patients with asthma or cystic fibrosis Choice E= Hypersensitivity to shellfish or other seafood usually presents with urticaria, angioedema, and/or anaphylaxis immediately after food consumption. Food is rarely an inciting factor for asthma. Most commonly, food-related asthma is the result of added preservatives, such as sulfides Choice F= Inhalational exposure to thermophilic actinomycetes species is classically associated with hypersensitivity pneumonitis in farmers (farmer's lung). Patients typically develop respiratory and systemic symptoms (e.g., fever, headache, malaise) after exposure to moldy hay or contaminated compost

-Age: 56 years man -Brought to the ED due to burning substernal pain that began 6 hours ago. The patient has a history of type 2 diabetes and hypertension. He smokes 2 packs of cigarettes daily and consumes alcohol occasionally. An ECG performed in the ED shows ST segment elevation in leads I, aVL and V3-V6. During the next several hours, the patient develops progressive shortness of breath that worsens when lying flat. -Which of the following histologic features is most likely to be present in this patient's lung tissue? a)Fat globules and bone marrow cells in the pulmonary arterioles b)Focal necrosis of the alveolar walls with intraalveolar hemorrhage c)Neutrophil-rich fluid filling the bronchi, bronchioles, and alveoli d)Numerous hemosiderin-laden macrophages in the alveoli e)Engorged capillaries and alveoli fille with acellular pink material f)Prominent mononuclear cell infiltration in the interstitium

Answer: Choice E (Engorged capillaries and alveoli filled with acellular pink material -The patient has chest pain and ECG evidence of acute MI involving the lateral wall of the left ventricle. His subsequent development of dyspnea and orthopnea is most likely due to MI-induced acute LV failure, which can result in rapid onset of pulmonary venous hypertension and acute pulmonary edema -Cardiogenic pulmonary edema represents increased filtration of fluid and electrolytes into the lung interstitium and alveoli. The fluid that accumulates is a transudate (an ultrafiltrate of plasma caused by changes in hydrostatic or oncotic pressure) rather than an exudate (an extravasation of not only fluid and electrolytes but also plasma proteins and circulating leukocytes due to inflammatory disruption of the vascular membrane). On light microscopy, engorged alveolar capillaries are evident and the intraalveolar transudate appears as acellular pink material Choice A= Fat globules and bone marrow cells in the pulmonary arterioles occur with fat embolism syndrome, which is not a complication of MI but is typically associated with long bone (e.g., femur) or pelvic fracture Choice B= Focal necrosis of alveolar walls with intraalveolar hemorrhage is typical of pulmonary hemorrhage syndromes such as Goodpasture syndrome and other vasculitides (e.g., hypersensitivity angiitis, granulomatosis with polyangiitis) Choice C= The presence of numerous neutrophils in the alveolar fluid is consistent with exudative alveolar filling, as occurs in bacteria pneumonia Choice D= Hemosiderin-laden macrophages ("heart failure cells") form as macrophages digest RBCs that leak from alveolar capillaries damaged by high intravascular pressure. They are a sign of chronic lung congestion and would not be present acutely Choice F= Mononuclear interstitial pulmonary infiltrates are found in various interstitial lung diseases (e.g., idiopathic pulmonary fibrosis)

-Age: 1 month boy -Brought to the office on a hot summer day due to vomiting and excessive sleepiness. He had been exclusively breastfeeding with normal feeding patterns until this afternoon, when he had 2 episodes of emesis and appeared much sleepier than usual -The infant was born at 41 weeks gestation after an unremarkable pregnancy. Newborn screening was positive for a homozygous delta-F508 mutation affecting an epithelial transmembrane protein; further confirmatory testing is pending. -Physical examination shows a somnolent boy with a sunken fontanelle and dry mucous membranes. Which of the following electrolyte disturbances is most likely responsible for this patient's current symptoms? a)Hyperchloremia b)Hyperkalemia c)Hypernatremia d)Hypomagnesemia e)Hyponatremia

Answer: Choice E (Hyponatremia) -The delta-F508 mutation in the cystic fibrosis transmembrane conductance regulator (CFTR) protein is the most common mutation in patients with cystic fibrosis (CF). In unaffected individuals, the CFTR protein serves as a chloride channel that regulates the flow of sodium, chloride, and water across the epithelial membranes of the airways, biliary tree, intestines, sweat ducts, and pancreatic ducts -Newborn screening for CF is performed in the US, but positive results should be confirmed by sweat testing. When eccrine sweat is first produced, it is isotonic with extracellular fluid. As sweat travels through the eccrine duct to the skin's surface, chloride is normally reabsorbed via CFTR; and sodium and water follow. CFTR facilitates production of hypotonic sweat in patients without CF. However, patients with CF are unable to reabsorb chloride and sodium in the eccrine ducts and therefore secrete sweat with high sodium and chloride levels -The most likely cause of this patient's symptoms (lethargy, vomiting) is hyponatremia due to excessive salt wasting from his sweat. Risk factors for hyponatremia in patients with CF include exclusive breast or formula feeding prior to the introduction of sodium-rich, solid foods; exposure to high-temperature environments, and exercise. Therefore, salt supplementation is recommended for patients with CF Choice A= Chloride ions are an important contributor to extracellular fluid volume, and hypochloremia is also very common in patients with CF due to sweat losses. In contrast, hyperchloremia is more likely to develop in normal patients with excessive sweating due to increased free water loss from hypotonic sweat Choice B= Hyperkalemia typically occurs with cell lysis (e.g., rhabdomyolysis, tumor lysis syndrome), metabolic acidosis (due to intracellular to extracellular K+ shift), or abnormalities with renal excretion Choice C= Hypernatremia classically occurs with excessive salt intake or free water loss (e.g., diarrhea, diabetes insipidus, improperly prepared infant formula). Patients with CF typically do not experience hypernatremia as they lose excess sodium through their sweat glands Choice D= Hypomagnesemia most commonly arises from urinary loss due to medications (e.g., loop diuretics), alcohol use, or uncontrolled diabetes mellitus with polyuria

-Age: 55 years woman -History of Crohn disease; is admitted to the hospital due to perforated appendicitis. The patient quickly develops respiratory difficulty, and acute respiratory distress syndrome is diagnosed. She is intubated and mechanically ventilated with positive pressure ventilation -Despite the use of high positive end-expiratory pressure, the patient's condition continues to deteriorate. She is intermittently placed in the prone position while mechanically ventilated. Which of the following is most likely to occur due to this position change? a)Alveolar hyperdistention b)Decreased cardiac output c)Decreased functional residual capacity d)Decreased right ventricular preload e)Improved ventilation-perfusion matching

Answer: Choice E (Improved ventilation-perfusion matching) -Prone positioning (lying face down) is sometimes used to improve arterial oxygenation in patients with severe acute respiratory distress syndrome. It likely does so by reducing atelectasis of posterior alveoli to reduce intrapulmonary shunting and improve ventilation-perfusion matching throughout the lungs. -Because the heart occupies anterior space in the chest cavity, the majority of lung mass is located posteriorly. When a patient is in the supine position (lying face up), the weight of the heart and anterior lung segments compresses the posterior lung segments and limits ventilation of posterior alveoli. In addition, abdominal organs and tissue mass weighs considerably on the posterior portion of the diaphragm, displacing it cranially and further contributing to compression of the posterior basal lung segments. The anterior alveoli, subjected to minimal compression and ample ventilation in the supine position, are often hyperinflated, increasing the risk of barotrauma in those regions -Prone positioning causes the heart and anterior lung mass to rest on the anterior chest wall, alleviating compression of the posterior lung segments. In addition, the freely hanging abdomen removes pressure on the diaphragm, allowing the diaphragm to move caudally and further open up collapsed posterior basal alveoli. With a higher percentage of total alveoli opened, ventilation is more evenly distributed, improving arterial oxygenation and reducing alveolar hyperinflation and hyperdistention (which may decrease the risk of barotrauma) (Choice A) Choices B and D= Prone positioning may increase venous return (i.e., right ventricular preload) and, consequently, cardiac output by positioning the heart slightly below the large reservoir of venous blood in the splanchnic circulation (i.e., additional stores of venous blood are mobilized to the right atrium) Choice C= By reducing posterior lung compression and decreasing atelectasis, prone positioning increases functional residual capacity (i.e., the volume of air in the lungs at the end of tidal expiration)

-Age: 4 years girl -Brought to the ED by her parents due to difficulty breathing. The patient was in her usual state of health until this morning when she developed a fever. Over the past 8 hours, her breathing has become more rapid and strained -Temp. (39.8C), BP (88/64), pulse (134), RR (34). On examination, the patient appears scared. She is sitting up on the bed leaning forward while supported by her arms with her head extended. Cardiac examination reveals tachycardia with a normal rhythm without rubs or murmurs. The abdomen is soft and nondistended. Which of the following additional signs is most likely to be found on examination? a)Asymmetrical breath sounds b)Diffuse rhonchi c)Expiratory stridor d)Expiratory wheeze e)Inspiratory stridor f)Inspiratory wheeze

Answer: Choice E (Inspiratory stridor) -This febrile, ill-appearing patient with respiratory distress who is in the tripod position likely has epiglottitis, a rare, potentially fatal infections most commonly caused by H. influenzae type b. Edema of the epiglottis can cause laryngeal obstruction that leads to inspiratory stridor -During inspiration, faster airflow causes decreased intraluminal pressure in the airways (e.g., drop in intralaryngeal pressure). In patients with airway obstruction at the level of the larynx, this decreased pressure leads to increased airway narrowing and increased turbulent airflow. Therefore, patients with airway obstruction at the level of the larynx (e.g., epiglottis, laryngomalacia, bilateral vocal cord paralysis) would be expected to have inspiratory stridor. Tripod positioning (extending neck and chin to the sniffing position) pulls the tongue forward to partially open the laryngeal airway Choice A= Asymmetric breath sounds can occur with unilateral obstruction below the level of the carina (e.g., foreign body aspiration) or with pleural disorders (e.g., effusion, pneumothorax). This patient's tripod positioning is more suggestive of laryngeal obstruction Choice B= Rhonchi are low-pitched rattling sounds produced when air flows through bronchi that have excessive secretions. Because they are caused by movement of air through wet secretions, rhonchi can be heard on both inspiration and expiration Choice C= Expiratory stridor occurs due to tracheal pathology (e.g., tracheomalacia) because inspiration decreases intrathoracic pressure (relative to the airways), which widens the intrathoracic trachea. Expiration increases intrathoracic pressure, which narrows the intrathoracic trachea, worsening the obstruction Choices D and F= Wheezing (i.e., high-pitched whistling noise) occurs with constriction of the smaller airways of the lungs (e.g., asthma). Because these small airways are intrathoracic, the constriction is more pronounced during expiration due to increased intrathoracic pressure

-Age: 68 years woman -Brought to the ED due to worsening fever, cough, and confusion. The patient has had a "cold and congestion" since last week, which initially improved, but she began feeling worse again 3 days ago. Her daughter reports that the patient also has been eating poorly. Other medical conditions include COPD and chronic kidney disease. -Temp. (39.4C), BP (74/46), pulse (128), RR (30). Oxygen saturation is 94% on 2L/min. On physical examination, the patient is lethargic with dry mucous membranes and flat neck veins. Lung examination reveals dullness to percussion and crackles at the right base. Chest x-ray shows a right lower lobe consolidation. IV access is established. IV administration of which of the following is the most appropriate next step in management of this patient? a)0.45% saline b)3% saline c)5% dextrose in 0.45% saline d)Albumin solution e)Lactate Ringer solution f)Sodium bicarbonate solution

Answer: Choice E (Lactated Ringer solution) -This patient with fever, cough, and radiologic evidence of consolidation in the right lower lung has pneumonia. Her tachycardia, hypotension, and confusion (evidence of end-organ hypoperfusion) are most likely due to septic shock, which is characterized by increased permeability of the vascular endothelium with leakage of intravascular fluid into the extravascular space -In the management of septic shock, rapid restoration of intravascular volume and adequate end-organ perfusion is critical. This is best accomplished with administration of boluses of isotonic crystalloid in the form of 0.9% (normal) saline or lactated Ringer solution through large-bore, peripheral intravenous catheters. These solutions are ideal for volume resuscitation because they have osmolarity very close to the normal osmolarity of the blood (285mOsm/kg H2O), while the sodium and chloride ions help retain fluid in the extracellular space -Prompt initiation of appropriate empiric antibiotic therapy is also critical in the management of septic shock Choices A and C= 0.45% (half-normal) saline is hypotonic; 5% dextrose in 0.45% saline is hypertonic initially but becomes hypotonic following infusion because the dextrose is rapidly metabolized. These hypotonic solutions are often used at low infusion rates for patients with a deficiency of free water (i.e., hypernatremia) or for maintenance hydration. However, they are not effective for rapid volume resuscitation because the low osmolarity causes much of the fluid volume to shift into the intracellular space following infusion Choice B= Infusion of 3% (hypertonic) saline can lead to rapid fluid-shifting from the intracellular to the extracellular space with potentially devastating consequences (e.g., osmotic demyelination syndrome); therefore, 3% saline is not appropriate for rapid volume resuscitation. It is appropriate for careful use in patients with severe symptomatic hyponatremia Choice D= Albumin solution is an isotonic colloid solution that can be used for rapid volume resuscitation. However, it is less preferred due to high cost and limited availability compared to isotonic crystalloid Choice F= Sodium bicarbonate solutions can have variable tonicity and are typically used at low infusion rates for patients with severe metabolic acidosis. These solutions are generally not used for rapid volume resuscitation

-Age: 44 years man -Comes to the ED with a 3-day history of fever, chills, malaise, dyspnea and a cough productive of "greenish" sputum. The patient has no prior medical problems and has never been hospitalized. He has a 25-pack-year smoking history and drinks 4-5 beers a week -Temp. (39.4C), BP (130/80), pulse (98), RR (20). On examination, dullness to percussion, crackles, and egophony are present at the right lung base. The remainder of the examination is normal. Chest x-ray shows a dense infiltrate occupying the entire right lower lobe. -Which of the following most likely accounts for the color of this patient's sputum? a)Epithelial necrosis b)Hemolysis c)High bacterial load d)Mucopolysaccharides e)Myeloperoxidase

Answer: Choice E (Myeloperoxidase) -This middle-aged man presenting with new-onset fevers, productive cough, and a dense lobar infiltrate likely has community-acquired pneumonia (CAP). CAP in otherwise healthy individuals is most commonly caused by S. pneumoniae, the most common bacterial etiology worldwide. Tobacco use further increases its risk. In nonelderly patients, pneumococcal pneumonia presents with abrupt-onset fevers, rigors, tachypnea, and productive cough with consolidation in one lobe of the lungs -Neutrophil myeloperoxidase is responsible for the green color of pus and sputum in bacterial infections. It is a blue-green heme-based pigmented molecule contained within the azurophilic granules of neutrophils and catalyzes the production of hypochlorous acid from chloride and hydrogen peroxide during the phagocytic respiratory burst Choices A and B= Both the "currant-jelly" sputum seen in pneumonia caused by Klebsiella pneumoniae and the "rusty" color occasionally seen in pneumococcal pneumonia are due to extravasation of RBCs and hemoglobin into the sputum caused by extensive inflammation and necrosis Choice C= Pseudomonas pneumonia can cause a blue-green pigment due to the production of pyocyanin, but Pseudomonas is not a common cause of CAP in otherwise healthy individuals Choice D= Respiratory epithelium sloughing and mucopolysaccharide production can contribute to the formation of sputum in patients with respiratory infection but does not contribute to sputum's green color

-Age: 54 years woman -Comes to the office due to worsening nonproductive cough and dyspnea for the past several months. She also reports anorexia and an unintentional weight loss of 7kg. The patient has no prior medical conditions, takes no medications, and is a lifelong nonsmoker -Physical examination shows decreased breath sounds on the left side. Chest x-ray reveals a large left-sided pleural effusion. Thoracocentesis yields fluid with a large number of red blood cells and numerous atypical cells staining positive for mucin. Which of the following is the most likely primary mechanism of this patient's pleural effusion? a)Decreased plasma oncotic pressure b)Disruption of the thoracic lymphatic duct c)Increased intercostal venous pressure d)Increased pulmonary capillary pressure e)Occlusion of pleural lymphatic stoma

Answer: Choice E (Occlusion of pleural lymphatic stoma) This patient's several-month history of nonproductive cough, shortness of breath, and weight loss combined with a large left-sided pleural effusion on chest x-ray raises suspicion for malignancy. Thoracentesis yielding a high erythrocyte concentration (a common finding in malignant effusions) and atypical mucin cells is consistent with adenocarcinoma; lung adenocarcinoma and breast adenocarcinoma are 2 of the most common causes of malignant pleural effusion Malignant effusions are exudative by Light criteria and can occur via several mechanism: -Localized lung inflammation can cause increased vascular permeability, resulting in increased inflow of fluid into the pleural space -Once malignant cells have metastasized to the pleural space, they can occlude the pleural lymphatic stoma located on the parietal surface and prevent pleural fluid reabsorption. This is likely the primary mechanism of effusion in this patient with evidence of pleural metastasis (i.e., atypical mucin cells in pleural fluid) -Disruption of the thoracic lymphatic duct is an occasional cause of malignant effusion that leads to chylothorax (milky white pleural fluid with high triglyceride content). This mechanism is most commonly seen with lymphoma; it can sometimes occur due to mass effect of lung cancer on the thoracic duct, but this is relatively uncommon (Choice B) Choice A= Decreased plasma oncotic pressure (i.e., hypoalbuminemia) is a common cause of transudative pleural effusion. Although hypoalbuminemia due to malnutrition may complicate malignancy and could contribute to pleural effusion, it is unlikely to be the primary mechanism in this patient with evidence of pleural metastatic disease Choices C and D= Increased hydrostatic pressure in the pulmonary capillaries and intercostal veins can cause transudative pleural effusion in patients with decompensated heart failure or other causes of intravascular volume overload (e.g., renal failure)

-Age: 53 years man -Comes to the ED due to progressive shortness of breath and nonproductive cough. Medical history is significant for long-standing hypertension and type 2 diabetes, for which he takes lisinopril and metformin. The patient has no drug allergies -BP (160/100), pulse (110), RR (20). On physical examination, heart sounds are regular. Lung examination reveals decreased tactile fremitus over the lower right lung along with dullness to percussion. Which of the following is the most likely diagnosis? a)Bronchospasm b)Emphysema c)Lobar consolidation d)Pericardial effusion e)Pleural effusion f)Pneumothorax g)Pulmonary edema

Answer: Choice E (Pleural effusion) This patient mostlikelyhasaright-sidedpleuraleffusion.Excess fluid within the pleural space acts to insulate vibrations and breath sounds that originate in the airways of the lungs. Consequently, tactile fremitus, the transmission of vibration from vocalized sound (e.g., saying "ninety-nine"), is decreased over a pleural effusion. Breath sounds are also decreased or absent. The high density of pleural fluid compared to normallung(alveolus-air composite) causes dullness to percussion over the effusion Choice A= Bronchospasm is likely to have minimal effect on tactile fremitus and dullness to percussion Choice B= The hyperinflated alveoli in emphysema should demonstrate hyperresonance to percussion and decreased tactile fremitus Choices C and G= Because sound vibrations travel faster and more efficiently through liquids than through gases, alveolar filling processes such as lobar consolidation (alveoli filled with pus) and pulmonary edema (alveoli filled with transudate) create increased breath sound intensity and increased tactile fremitus Choice D= A large pericardial effusion might cause dullness to percussion with decreased tactile fremitus over the precordium, but not over the lower lung Choice F= Like pleural effusion, pneumothorax (air in the pleural space) acts to insulate sound originating in the airways; therefore, tactile fremitus and breath sounds are decreased. However, the low density of air compared to normal lung creates hyperresonance to percussion

-Age: 54 years man -Comes to the office due to daytime sleepiness and lack of energy. The symptoms began 6 months ago and have progressively worsened so that he feels "completely drained" by the end of the day. The patient's wife mentions that he snores loudly. -His past medical history is unremarkable, although he has not seen a physician in over 10 years. The patient does not use tobacco or alcohol, and he works in the warehouse of an agricultural supply company. BMI is 34. Physical examination shows a narrow oropharynx and a large neck circumference. The patient is at increased risk of developing which of the following? a)Bronchiectasis b)Hypertrophic cardiomyopathy c)Laryngeal carcinoma d)Narcolepsy e)Pulmonary hypertension

Answer: Choice E (Pulmonary hypertension) -This patient, an obese man with loud snoring, daytime sleepiness, and suggestive examination findings (e.g., thick neck, narrow airway) most likely has obstructive sleep apnea (OSA). OSA is characterized by recurrent obstruction of the upper airway during sleep; each nocturnal episode of reduced ventilation causes transient hypercapnia and hypoxemia. These blood gas derangements result in reflexive systemic and pulmonary vasoconstriction, endothelial dysfunction, abnormal venous return and cardiac output, and sympathetic cardiac stimulation -Prolonged, untreated OSA can cause pulmonary hypertension and right heart failure. Most patients with OSA will develop systemic hypertension due to chronic sympathetic stimulation and elevated plasma norepinephrine levels. Patients also lose the normal diurnal variation in BP. Other cardiovascular complications of OSA include atrial fibrillation and other arrhythmias, coronary artery disease, and increased risk of sudden cardiac death Choice A= Acquired bronchiectasis may be seen in patients with recurrent infection, impaired drainage (e.g., cystic fibrosis), airway obstruction (e.g., foreign body aspiration), or inadequate host defense (e.g., hypogammaglobulinemia) Choice B= Systemic hypertension, as seen in OSA, can lead to mild-to-moderate LV hypertrophy and impaired systolic and diastolic function. In contrast, hypertrophic cardiomyopathy is an autosomal dominant disease of the cardiac sarcomere characterized by severe myocardial hypertrophy. It is not associated with hypertension or OSA Choice C= Laryngeal carcinoma is associated with cigarette smoking and heavy alcohol use Choice D= Like OSA, narcolepsy can also cause daytime drowsiness. However, narcolepsy is also associated with cataplexy (sudden loss of muscle tone), sleep attacks, sleep paralysis, and hypnagogic hallucinations. OSA is not a risk factor for narcolepsy

-Age: 65 years woman -With chronic obstructive pulmonary disease comes to the office for a follow-up appointment. The patient reports increasing shortness of breath that is not relieved by her inhalers. Her other medical conditions include osteoarthritis -She is a former smoker with a 45-pack-year history. BP (120/70), pulse (75), RR (22). Oxygen saturation on room air is 91% at rest. On physical examination, jugular venous pressure is elevated. There is increased intensity of the pulmonic component of S2. Breath sounds are decreased bilaterally with scattered wheezes but no crackles -There is 2+ lower extremity edema. Which of the following hemodynamic parameters is most likely to be increased in this patient? a)Left ventricular preload b)Left ventricular stroke volume c)Pulmonary arterial compliance d)Pulmonary capillary wedge pressure e)Right ventricular afterload

Answer: Choice E (Right ventricular afterload) -This patient with COPD has evidence of pulmonary hypertension complicated by right-sided heart failure. Hypoxic vasoconstriction is a physiologic mechanism unique to lung tissue that helps minimize ventilation-perfusion mismatch and increase overall gas exchange efficiency. However, chronic and diffuse pulmonary vasoconstriction, which occurs with advanced hypoxic lung disease (e.g., COPD, interstitial lung disease, obesity hypoventilation syndrome), and can lead to pulmonary hypertension (i.e., increased RV afterload) -Pulmonary hypertension typically presents with dyspnea and/or fatigue, and some patients may develop exertional angina or syncope due to reduced cardiac output. Physical examination can reveal left parasternal lift (due to RV hypertrophy) and a loud pulmonic component of S2 due to high pulmonary artery pressure. Over time, the RV may be unable to pump against the increased afterload and right-sided heart failure can develop, evidenced by jugular venous distention and prominent lower extremity edema Choices A, B and D= The hemodynamic parameters in pulmonary hypertension secondary to hypoxic lung disease are similar to those in pulmonary arterial hypertension. LV preload and stroke volume (cardiac output) are decreased due to impaired pumping of blood through the lungs to the left atrium. Pulmonary capillary wedge pressure, an estimate of left atrial pressure and a representation of left-sided preload, is also decreased Choice C= Pulmonary arterial compliance decreases with pulmonary hypertension due to vascular remodeling and stiffening of the pulmonary arterial wall

-Age: 66 years man -Hospitalized 3 days ago for left lower lobe pneumonia is evaluated for persistent fever. He has been receiving the appropriate IV antibiotics as determined by antibiotic susceptibility testing of his sputum cultures. The patient has a history of diabetes mellitus and hypertension -Temp. (38C), pulse (87). Examination shows dullness to percussion and decreased breath sounds over the left lower lung. Imaging studies reveal a loculated, left-sided pleural effusion. A tube is placed to drain the fluid via an incision at the 5th intercostal space in the midaxillary line. Which of the following muscles is most likely to be dissected during the procedure? a)External oblique b)Infraspinatus c)Latissimus dorsi d)Pectoralis major e)Serratus anterior

Answer: Choice E (Serratus anterior) -This patient most likely has empyema requiring drainage with a chest tube. The technique involves placing the chest tube through the skin and subcutaneous fat into the 4th or 5th intercostal space in the anterior axillary or midaxillary line. The tube traverses the serratus anterior muscle, intercostal (external, internal, innermost) muscles, and parietal pleura to reach the pleural cavity -The serratus anterior originates as multiple branches from the side of the chest along the 1st-8th ribs and inserts along the entire length of the medial scapular border. The muscle is divided into 3 parts (superior, intermediate, inferior) depending on the insertion site at the scapula. The inferior part of the muscle facilitates arm elevation by pulling the lower end of the scapular forward (scapular rotation). All 3 muscle parts can help with respiration by lifting the ribs when the shoulder girdle is fixed Choice A= The external oblique originates from the lateral ribs (5-12) and extends anteriorly across the abdomen to insert into the linea alba, pubis, and iliac crest of the hip bones. The muscle is usually inferior to the chest tube insertion site Choice B= The infraspinatus is a rotator cuff muscle that attaches medially to the infraspinatus fossa of the scapula and laterally to the greater tubercle of the humerus. The muscle stabilizes the shoulder joint in addition to externally rotating the humerus Choice C= The latissimus dorsi is a back muscle that helps with multiple shoulder movements (e.g,, extension, adduction). It originates from the spinous processes of T7-L5, the 9th-12th ribs, and the inferior angle of the scapula. The muscle inserts into the humerus and is located posterior to the chest tube insertion site Choice D= The pectoralis major arises from the anterior clavicle, sternum, costal cartilages, and aponeurosis of the external oblique muscle. It attaches on the lateral lip of the bicipital groove of the humerus and serves to adduct and internally rotate the humerus. The muscle is anterior to the chest tube insertion site

-Age: 52 years man -Comes to the office due to an 8-month history of progressive shortness of breath that peaks during strenuous activity. Several of his coworkers have experienced similar symptoms. His medical history includes hypertension and type 2 diabetes. -Pulmonary examination reveals diffuse fine crackles. Chest x-ray reveals nodular densities in both lungs that are most prominent in the apical regions. Calcification of the hilar lymph nodes is also seen. Bronchoscopy with transbronchial biopsy of a calcified node is performed, and polarized microscopy shows birefringent particles surrounded by dense collagen fibers. This patient most likely has a history of exposure to which of the following substances? a)Asbestos b)Beryllium c)Coal dust d)Organic dust e)Silica

Answer: Choice E (Silica) -This middle-aged patient with dyspnea on exertion, nodular densities on x-ray, calcified hilar lymph nodes, and birefringent particles on biopsy has silicosis. Silicosis is a form of pneumoconiosis (i.e., a type of interstitial lung disease caused by inhalation of mineral dust). Inhaled crystalline silica, typically due to industrial exposure (e.g., mining, sand blasting), is toxic to alveolar macrophages and promotes the formation of intrapulmonary free radicals, leading to progressive lung injury and interstitial collagen deposition. Histologically, silicosis is characterized by birefringent silicate particles within dense, whorled collagenous nodules surrounded by dust-laden macrophages -Silicosis is often initially asymptomatic but can present with dyspnea on exertion and productive cough, typically 10-20 years after initial exposure. Radiography varies based on disease progression; simple silicosis typically demonstrates numerous small, rounded nodules predominant in the upper lobes, that may rarely coalesce to form mass-like upper lobe fibrosis (progressive massive fibrosis). Calcification of the rim of hilar nodes (eggshell calcification) may also be seen Choice A= Asbestosis can present with dyspnea on exertion, but chest x-ray is more likely to reveal an interstitial pattern of involvement most prominent in the lower zones. Pleural plaques may also be noted. Histology shows ferruginous bodies featuring fusiform rods with a translucent asbestos center and golden-brown iron coating Choice B= Berylliosis may present with dyspnea and ill-defined nodular or irregular opacities on chest x-ray. Histology reveals noncaseating epithelioid granulomas without obvious associated particles Choice C= Coal worker's pneumoconiosis can present with exertional dyspnea and nodular interstitial opacities on chest x-ray. Histology of nodal and perilymphatic lung tissue shows accumulations of black, carbon-laden macrophages (coal macules) Choice D= Inhalation of organic dusts can cause hypersensitivity pneumonitis. Patients have exertional dyspnea and diffuse nodular interstitial infiltrates on chest x-ray. Histology of nodal tissue may reveal noncaseating granulomas

-Age: 45 years man -Previously healthy, brought to the ED after being hit by a car while cycling. The patient is hypotensive and tachycardic on arrival and has comminuted fractures of the pelvis and lower extremities. Volume resuscitation is performed with IV fluids and multiple units of packed RBCs and plasma. Several hours later, the patient develops worsening dyspnea and hypoxia -Chest imaging reveals new bilateral infiltrates. Endotracheal intubation is performed for respiratory support. Despite aggressive resuscitation, the patient dies 24 hours later. Autopsy examination of the lungs show protein-rich fluid within the alveolar airspaces. Breakdown of the barrier formed by which of the following cells most likely contributed to this patient's pulmonary pathology? a)Alveolar macrophages b)Ciliated epithelial cells c)Club cells (previously Clara cells) d)Goblet cells e)Type I pneumocytes f)Type II pneumocytes

Answer: Choice E (Type I pneumocytes) This patient who developed hypoxia, pulmonary edema (bilateral pulmonary infiltrates), and protein-rich fluid in the alveoli had acute respiratory distress syndrome (ARDS). ARDS is a severe inflammatory reaction that occurs in the lungs and results in respiratory failure with hypoxemia and noncardiogenic pulmonary edema. Risk factors include sepsis, severe trauma (e.g., long-bone fractures with fat embolism, extensive tissue injury), and transfusion of blood products, all of which can initiate pulmonary injury and trigger a cascade of inflammation: -In response to injury, alveolar macrophages release multiple proinflammatory cytokines that recruit neutrophils to the lungs (Choice A) -Upon arrival to pulmonary tissue, the neutrophils release inflammatory mediators (e.g., proteases, free radicals). This increases inflammation and leads to damage to the pulmonary endothelium and alveolar pneumocytes -The alveolar-capillary barrier, which is formed by the endothelial cells and type 1 pneumocytes, is then destroyed, allowing fluid and RBCs to escape the vascular and interstitial spaces and pour into the alveoli. There, they combined with material from necrotic cells and form a thick proteinaceous fluid and a hyaline membrane -Destruction of type 2 pneumocytes, which normally produce surfactant and proliferate in response to injury, leads to alveolar collapse (but does not cause destruction of the alveolar capillary barrier) (Choice F) The net effect is impaired alveolar gas exchange and respiratory failure, with affected patient having a high mortality rate Choices B and D= Ciliated epithelium and goblet cells line the larger airways. Goblet cells secrete mucus that traps particulate matter (e.g., dust, bacteria), while ciliated epithelial cells assist in its elimination by sweeping the mucus up the bronchi and trachea to the pharynx, where it is swallowed. These cells are not prominently involved in ARDS Choice C= Club cells (formerly called Clara cells) are nonciliated, secretory constituents of the terminal respiratory epithelium. They secrete protein and surfactant components and help detoxify inhaled substances (e.g., tobacco smoke). They are not prominently involved in ARDS

-Age: 34 years woman -Comes to the office due to progressive exertional dyspnea for the past 6 months. She has no chest pain, lightheadedness, or syncope. Medical history is unremarkable, and the patient takes no medications. She does not use tobacco, alcohol, or illicit drugs. -The patient's mother died of pulmonary arterial hypertension at age 32. BP (125/74), pulse (75), BMI (23). On physical examination, breath sounds are clear without wheezes or crackles. There is a loud S2 at the left upper sternal border. -Chest x-ray reveals clear lungs. ECG shows right axis deviation. If the patient's condition is inherited, which of the following is the most likely direct cause of her disease? a)Elevated left atrial pressure b)Hypoxic vasoconstriction c)Right ventricular hypertrophy d)Thrombotic occlusion of pulmonary arteries e)Vascular smooth muscle proliferation

Answer: Choice E (Vascular smooth muscle proliferation) -This patient's presentation and family history are suggestive of pulmonary arterial hypertension (PAH). Hereditary PAH is most often due to inactivating mutation in BMPR2, which is transmitted in an autosomal dominant manner with variable penetrance. Patients with this mutation have a predisposition for dysfunctional endothelial and smooth muscle cell proliferation. An insult (e.g., infection, drugs) is thought to then activate the disease process, which involves increased levels of vasoconstrictive, proliferative mediators (e.g., endothelin) and decreased levels of vasodilative, antiproliferative mediators (e.g., nitric oxide, prostacyclin). The end result is vasoconstriction with vascular smooth muscle proliferation, intimal thickening and fibrosis, increased pulmonary vascular resistance, and progressive pulmonary hypertension -Increased pulmonary arterial pressure can cause dyspnea and fatigue and can often be detected as a loud pulmonic component of S2. Significant pulmonary hypertension leads to compensatory RV hypertrophy (Choice C), which may cause right axis deviation on ECG. Right-sided heart failure can eventually occur, evidenced by elevated jugular venous pressure, hepatic congestion, and peripheral edema. The lungs remain clear in PAH as the lung parenchyma is unaffected Choice A= Elevated LA pressure is indicative of heart failure, most commonly due to LV dysfunction. The elevated pressure transmits back to the pulmonary venous system and can lead to pulmonary hypertension; however, evidence of pulmonary edema (e.g., crackles, pulmonary congestion on chest x-ray) would be expected Choice B= Excessive hypoxic vasoconstriction of the pulmonary arteries can lead to pulmonary hypertension in patients with severe hypoxic lung disease, including COPD, interstitial lung disease, and obesity hypoventilation syndrome. However, this patient's normal chest x-ray and BMI rule out these disorders Choice D= Chronic pulmonary arterial thromboembolism decreases the cross-sectional area of the pulmonary vasculature and can lead to pulmonary hypertension. However, PAH is more likely in this patient with a positive family history and no evidence of venous thromboembolism

-Age: 60 years woman -Comes to the ED due to shortness of breath and productive cough. The patient has a 30-pack-year smoking history. Oxygen saturation is 88% on room air. On physical examination, she appears uncomfortable and uses her accessory respiratory muscles to breathe -Expiratory wheezes are heard throughout the lungs. The patient is started on high-flow oxygen supplementation. Shortly afterward, she becomes increasingly lethargic and confused. This patient's clinical decline is most likely attributable to an increase in which of the following parameters? a)Pulmonary vascular resistance b)Reactive oxygen species production c)Renal bicarbonate reabsorption d)Respiratory minute ventilation e)Ventilation-perfusion mismatch

Answer: Choice E (Ventilation-perfusion mismatch) -This patient's clinical presentation (i.e., dyspnea, productive cough, hypoxemia, expiratory wheezing, accessory muscle use, smoking history) is consistent with COPD exacerbation. Supplemental oxygen may be warranted in patient with COPD who have significant hypoxemia; however, administration of excessively high oxygen concentrations can lead to increased CO2 retention (oxygen-induced hypercapnia), resulting in confusion and a depressed level of consciousness (e.g., lethargy) -Several mechanisms contribute to oxygen-induced hypercapnia, but the major cause is increased ventilation-perfusion mismatch. Hypoxia causes vasoconstriction of the pulmonary arterioles, which acts to shunt blood toward alveoli with the highest ventilation, thereby minimizing physiologic dead space. Providing high-concentration supplemental oxygen allows lung regions with relatively poor ventilation to have higher oxygen levels, reversing pulmonary vasoconstriction. The redistribution of blood flow away from well-ventilated alveoli leads to an increase in physiologic dead space (well-ventilated alveoli are less perfused) with a corresponding reduction in CO2 excretion Choice A= Oxygen administration alleviates hypoxic pulmonary vasoconstriction, causing an overall decrease in pulmonary vascular resistance Choice B= Hyperoxia can increase production of reactive oxygen species (e.g., superoxide, hydroxyl radical, hydrogen peroxide), leading to injury of the airways and lung parenchyma. Oxygen toxicity typically manifests with substernal heaviness, pleuritic chest pain, and cough/dyspnea within 24 hours after breathing pure oxygen Choice C= In patients with hypercapnia and respiratory acidosis, the kidney compensate by increasing bicarbonate reabsorption and normalizing the serum pH level. This response occurs slowly over several days and would tend to ameliorate the effects of hypercapnia Choice D= Peripheral chemoreceptors in the carotid and aortic bodies sense arterial PaO2 and are normally stimulated by hypoxemia. High-flow oxygen reduces chemoreceptor stimulation, resulting in a decreased (not increased) respiratory rate and minute ventilation

-Age: 45 years man -Hospitalized; being evaluated for shortness of breath and hypoxemia. The patient was admitted 3 days ago after a motor vehicle collision. At that time, evaluated revealed a femur fracture but no other injuries, and the patient underwent open reduction and internal fixation of the fracture. This morning, he suddenly began experiencing shortness of breath and sharp chest pain. -The patient has no significant medical history and has been receiving morphine for pain control. He has smoked a pack of cigarettes daily for 15 years and drinks alcohol occasionally -Temp. (36.1C), BP (120/70), pulse (98), BMI (42). Chest x-ray is unremarkable, and arterial blood gas analysis on room air shows a pH of 7.51, PaCO2 of 30mmHg, and PaO2 of 65mmHg. Which of the following is the primary cause of hypoxemia in this patient? a)Alveolar hypoventilation b)Decreased blood oxygen-carrying capacity c)Increased tissue oxygen extraction d)Primary diffusion impairment e)Ventilation/perfusion mismatch

Answer: Choice E (Ventilation/perfusion mismatch) -Sudden-onset shortness of breath and chest pain in a hospitalized patient should raise suspicion for pulmonary embolism (PE). This patient has 2 important risk factors for PE. Immobilization (causes venous stasis) or recent surgery (inflammation induces a hypercoagulable state). The risk of PE is high after large orthopedic procedures; prophylactic anticoagulation reduces but does not eliminate the risk. Thrombi most commonly originate in the deep veins of the pelvis and lower extremities before embolizing to the lungs. Although the risk of fat embolism is also increased following a long-bone fracture, this patient lacks the typical skin rash and neurologic findings (e.g., confusion) -Thrombotic occlusion of the pulmonary circulation leads to redistribution of blood flow in the lungs, resulting in a ventilation/perfusion (V/Q) mismatch. With many well-ventilated alveoli now inaccessible to blood flow (dead-space ventilation), the remaining accessible alveoli are unable to fully oxygenate the volume of blood that continues to flow through the pulmonary circulation, and hypoxemia results. As with any V/Q mismatch that affects only part of the lungs, supplemental oxygen can help correct the hypoxemia by increasing the alveolar partial pressure of oxygen, allowing accessible alveoli to transfer additional oxygen to the blood Choice A= Alveolar hypoventilation refers to a global decrease in the alveolar partial pressure of oxygen caused by a decrease in tidal volume or respiratory rate. Patients also develop respiratory acidosis due to decreased CO2 excretion. This patient's respiratory alkalosis is more typical of V/Q mismatch with subsequent hyperventilation, as V/Q mismatch does not hinder CO2 removal as much as blood oxygenation Choice B= Decreased blood oxygen-carrying capacity (i.e., decreased hemoglobin concentration) can occur due to postoperative bleeding. Although a decrease in hemoglobin can lead to tissue hypoxia, the measured PaO2 (indicative of dissolved oxygen content rather than hemoglobin-bound oxygen content) will not be affected Choice C= Increased tissue oxygen extraction would cause venous oxygen levels to decrease but would not typically affect the arterial oxygen content Choice D= Hypoxemia due to a primary defect that impairs gas diffusion between the alveoli and pulmonary capillaries can occur in chronic lung diseases, including pulmonary fibrosis and emphysema

-Age: 26 years woman -Evaluated for joint pain affecting the elbows, knees and ankles for the past month. She also has experienced a dry cough and mild shortness of breath over the last 6 months. She is sexually active and takes oral contraceptives. Temp. (37.2C), BP (120/70), pulse (84), RR (16) -On physical examination, the lungs are clear to auscultation. There is mild swelling and tenderness of the elbows, knees and ankles. The lower extremities are tender to palpation and have scattered erythematous nodules. Chest x-ray reveals lung nodules and hilar fullness. Transbronchial biopsy shows large epithelioid cells, occasional giant cells, and no areas of necrosis. Which of the following pharmacotherapies is most appropriate for the initial treatment of this patient? a)Ceftriaxone b)Doxycycline c)Etanercept d)Penicillin e)Prednisone

Answer: Choice E (prednisone) -This young patient has arthralgias, dyspnea, cough, and erythema nodosum (tender, subcutaneous, lower extremity nodules). This, in conjunction with lung nodules and hilar fullness on x-ray, is highly suggestive of sarcoidosis, an inflammatory disease characterized by granuloma formation in multiple tissues. Diagnosis is confirmed by biopsy showing noncaseating granulomas composed of epithelioid cells (activated macrophages), and giant multinucleated cells without central necrosis. Typical manifestations include hilar adenopathy, pulmonary infiltrates (e.g., nodules, interstitial lung disease) skin (e.g., erythema nodosum) and ocular (e.g., anterior uveitis) involvement, polyarthritis, and constitutional symptoms (e.g., fatigue, weight loss) -Many patients with sarcoidosis do not require treatment; however, those with significant symptoms (e.g., dyspnea, chest pain) or progressive pulmonary disease (i.e., worsening opacities/fibrosis or pulmonary function tests) should be treated. Oral glucocorticoids (e.g., prednisone) are the initial treatment of choice Choice A= Ceftriaxone is appropriate treatment for disseminated gonococcal infection, which causes arthralgias and tenosynovitis. However, patients typically have a pustular, painless dermatitis; lung involvement would be unexpected Choice B= Doxycycline is used to treat Lyme disease, which presents with an expansive erythematous rash with central clearing (erythema migrans), fatigue, and arthralgias; pulmonary involvement is unexpected. Histology demonstrates an inflammatory with spirochetes Choice C= Etanercept is a TNF-alpha inhibitor that is used in rheumatoid arthritis, a condition that can cause arthralgias, skin nodules (rheumatoid nodules) and interstitial lung disease; however, it typically involves the metacarpophalangeal and proximal interphalangeal joints. Pathology shows a periarticular inflammatory infiltrate with joint destruction; noncaseating granulomas would not be seen Choice D= Penicillin is used to treat syphilis, but the rash associated with this disease is typically maculopapular and involves the palms and soles. Histopathology may demonstrate lymphoplasmacytic inflammation and granulomas, and Treponema pallidum is visible on darkfield microscopy

-Age: 62 years man -Comes to the clinic due to a 6-month history of progressive exertional dyspnea. The patient has an occasional cough but has no palpitations, orthopnea, chest pain, or lower extremity swelling. Past medical history is insignificant. -He works as an attorney, does not use tobacco, and drinks alcohol only on social occasions. His symptoms continue to progress, and the patient dies of respiratory failure 3 years after the initial clinic visit. Autopsy findings include heterogenous lung parenchyma with predominantly subpleural areas of dense collagen deposition, lymphocyte infiltration and fibroblast proliferation intermixed with areas of normal lung tissue. Which of the following is the most likely diagnosis? a)Alpha-1 antitrypsin deficiency b)Asbestosis c)Chronic bronchitis d)Chronic hypersensitivity pneumonitis e)Cryptogenic organizing pneumonia f)Idiopathic pulmonary fibrosis

Answer: Choice F (Idiopathic pulmonary fibrosis) The described histopathologic findings are consistent with idiopathic pulmonary fibrosis (IPF). Microscopic findings are heterogenous and include: -Patchy areas of interstitial fibrosis with chronic interstitial inflammation intermixed with normal lung -Early lesions consist of fibroblastic foci that become increasingly collagenous with time -Honeycomb pattern with fibrotic walls and cystic spaces lined by bronchial epithelium -Fibrosis most prominent in the subpleural and perilobular regions Importantly, there should also be no other findings consistent with another disease process (e.g., granulomas) The precise etiology of IPF is unclear, but risk factors include cigarette smoking, environmental pollutants, chronic aspiration, older age, and certain genetic factors (e.g., telomerase mutations). Patients typically have dyspnea, nonproductive cough, finger clubbing, and inspiratory crackles. The onset is insidious, prognosis poor, and no curative treatment exists Choice A= Alpha-1 antitrypsin deficiency causes alveolar wall destruction, resulting in panacinar emphysema. Patients tend to present at a young age with dyspnea, productive cough, and wheezing. Emphysematous changes (e.g., bullae) are seen predominantly at the lung bases, and histologic findings include large alveoli with thin septa Choice B= Asbestosis usually occurs in patients with occupational exposures (e.g., shipbuilding). Microscopy may show fibrosis and honeycombing, but asbestosis can be differentiated from IPF by the presence of asbestos bodies (brown, beaded rods coated with iron-containing material) and ferruginous bodies (inorganic particles with a similar ferrous material). Pleural plaques are also common Choice C= Chronic bronchitis causes significant sputum production over a prolonged period and is often related to smoking. Histologic findings include mild lymphocytic infiltrates and mucous gland hyperplasia with mucus-filled bronchioles Choice D= Chronic hypersensitivity pneumonitis is due to repeated exposure to an inciting agent (e.g., birds, fungi, chemicals). Cough and dyspnea occur shortly after exposure to the antigen and may progress to respiratory failure. Histologic findings include noncaseating granulomas, peribronchial fibrosis, and patchy lymphocytic infiltrates Choice E= Cryptogenic organizing pneumonia is another interstitial lung disease of unclear etiology that typically presents with dry cough, dyspnea, and fatigue. However, histologic findings include fibroblastic plugs in the alveolar sacs and ducts, often extending into adjacent alveoli in a characteristic butterfly pattern

-Age: 23 years man -Undergoes exercise physiology testing at sea level. He jogs on a treadmill to achieve a moderate-intensity physical activity based on a target heart rate of 50-70% of his estimated maximum heart rate. An increase in which of the following is expected at the peak of his exertion? a)Arterial blood mean CO2 content b)Arterial blood mean O2 content c)Hemoglobin affinity for O2 d)pH of the arterial blood e)Physiologic dead space f)Venous blood mean CO2 content g)Venous blood mean O2 content

Answer: Choice F (Venous blood mean CO2 content) -Long-distance running and other forms of physical exercise cause increased oxidative metabolism of glucose and fatty acids in the skeletal muscle. This markedly increases the rates of O2 consumption and CO2 production, which must be balanced by increases in skeletal muscle perfusion and alveolar ventilation -The cardiovascular response to exercise involves vasoconstriction in the splanchnic circulation and vasodilation in skeletal muscle, shunting blood toward exercising muscle. The vasodilation is predominant, resulting in overall decreased systemic vascular resistance. There is also an increase in heart rate and stroke volume that markedly increases cardiac output and O2 delivery to the tissues. Increased O2 extraction by the tissues leads to a concomitant increase in CO2 production, which is absorbed by the systemic capillaries and transported in high quantities via venous blood to the lungs, where it is expired Choices A and B= Mean values for arterial O2 and CO2 content remain essentially constant during exercise, even during periods of intense exertion. This is likely accomplished via tight regulation of increases in alveolar ventilation and gas exchange efficiency Choices C and D= Tissue pH typically decreases during exercise due to production of carbonic and lactic acid in active skeletal muscle. The acidic pH causes a decrease in hemoglobin affinity for O2 (i.e., right shift in the O2-hemoglobin dissociation curve) to facilitate unloading of O2 in the tissues. Although venous pH may decrease significantly during moderate exercise, there is typically little change in arterial pH due to respiratory compensation Choice E= Physiologic dead space (i.e., the air in the respiratory system that does not participate in gas exchange) is decreased during exercise due to a decrease in pulmonary vascular resistance that allows perfusion of additional pulmonary capillary beds Choice G= The mean O2 content of venous blood remains the same or decreases with exercise because the rate of O2 extraction by the tissues outpaces the rate of oxygen delivery (cardiac output). In most healthy individuals, cardiac output is the major limiting factor to O2 consumption during exercise

-Age: 69 years man -Comes to the office due to progressive pain beneath his right scapula and in his right arm for the last several months. He has taken over-the-counter analgesics, which provide short-term relief, but the pain has become more severe and awakens him at night -The pain is associated with numbness in his right forearm extending up to the tips of the fourth and fifth fingers. The patient has also had persistent nonproductive cough and lately has been coughing up streaks of blood. -He has a history of hypertension, GERD, and osteoarthritis. He has smoked 2 packs of cigarettes daily for more than 40 years but recently cut down to 4 or 5 cigarettes a day. Which of the following is the most likely cause of this patient's symptoms? a)Abnormal hormone secretion b)Airway obstruction c)Anterior mediastinum mass d)Autoimmune disease e)Pericardial effusion f)Pleural effusion g)Superior sulcus tumor

Answer: Choice G (Superior sulcus tumor) This patient with an extensive smoking history who now has hemoptysis and shoulder pain likely has lung cancer, which can be associated with symptoms due to local or distant spread. Pancoast syndrome is caused by a tumor at the lung apex. Such tumors often arise in the superior sulcus (groove formed by the subclavian vessels). The apical location allows for extensive local tumor spread. Invasion of surrounding structures can cause the following: -Shoulder pain radiating toward the axilla and scapula (most common presenting symptoms). It occurs due to involvement of the lower brachial plexus. Other associated symptoms include arm paresthesia, weakness, and muscle atrophy -Horner syndrome occurs due to involvement of the cervical sympathetic ganglia. Symptoms include ipsilateral ptosis, miosis and anhidrosis -Upper extremity edema may be due to compression of subclavian vessels -Spinal cord compression and paraplegia can result from tumor extension into the intervertebral foramina Choice A= Squamous cell carcinoma is associated with parathyroid-related peptide secretion and hypercalcemia. Cushing syndrome and syndrome of inappropriate antidiuretic hormone secretion can accompany small cell carcinoma of the lung Choice B= Centrally located lung cancers (squamous cell and small cell carcinomas) frequently obstruct the main bronchi, producing dyspnea and cough. Airway obstruction would not cause shoulder pain or muscle atrophy Choice C= Anterior mediastinal masses include thymoma, teratoma, thyroid cancer, and (terrible) lymphoma ("terrible Ts"). They are less likely to lead to hemoptysis or brachial plexus involvement Choice D= Lung cancers are associated with a number of autoimmune syndromes, most commonly Lambert-Eaton (progressive proximal muscle weakness, often in the legs) Choice E= Pericardial effusions are characterized by dyspnea, distended neck veins, distant heart sounds, and -particularly if tamponade is present- pulsus paradoxus (decrease in SBP >10mmHg during inspiration) Choice F= A pleural effusion can lead to dyspnea and irritate the mediastinal and diaphragmatic pleura, causing referred shoulder pain (via the phrenic nerve); however, it would not cause hemoptysis


Set pelajaran terkait